You are on page 1of 87

Contents

數列與級數 ................................................................. 2
1 1-1 數 列 ......................................................................... 2

1-2 級 數 ....................................................................... 10

數據分析 .................................................................. 17
2 2-1 一維數據分析........................................................... 17

2-2 二維數據分析 ......................................................... 25

排列組合與機率 .................................................. 32

3-1 計數原理 ................................................................... 32


3 3-2 排 列 ....................................................................... 38

3-3 組 合 ....................................................................... 44
3-4 機 率 ....................................................................... 47

三角比 .......................................................................... 55

4 4-1 直角三角形的邊角關係 .................................... 55


4-2 廣義角與極坐標 .................................................. 61


4-3 面積公式與正餘弦定理 .................................... 68
模擬試題

第一次段考模擬試題 ..................................................... 78
第二次段考模擬試題 ..................................................... 80
第三次段考模擬試題 ..................................................... 82

- 1 -
2 互動式教學講義.數學(2) 解答篇
頁碼 頁碼
類題2

第1章 -
5
a2 9 5 27
1 公比 r= = =- × =-3
a1 5 9 5
1-1 數 列
27
5 範例 1 5
1 將 n=1,2,3,4,5 分別代入 an=3n-2 ∴a6=a1r6-1= ×(-3) 5
=-45
27
得前五項為 1,4,7,10,13 5
2 n 2 一般項 an=a1rn-1= ×(-3) n-1
,n 為正整數
2 將 n=1,2,3,4,5 分別代入 an=4×
3 () 5
27
n-1
8 16 32 64 128 3 ∵an= ×(-3) =-3645
得前五項為 , , , , 27
3 9 27 81 243 c(-3)n-1=-19683=(-3)9c n-1=9 c n=10
n(n-3)
3 將 n=1,2,3,4,5 分別代入 an= 7 範例 3
2 1 設首項為 a1,公差為 d
得前五項為-1,-1,0,2,5 a10=a1+9d
類題 則
a25=a1+24d
1 將 n=1,2,3,4 分別代入 an=3n+1 a1+9d=23 .....................1
得前四項為 4,7,10,13 c
a1+24d=-22 ...............2
2 將 n=1,2,3,4 分別代入 an=4×3n-1 解1、2得 a1=50,d=-3
得前四項為 4,12,36,108 〔另解〕
2n-1 ∵a25=a10+15d c-22=23+15d
3 將 n=1,2,3,4 分別代入 an=
n2 c d=-3
3 5 7 又 a10=a1+9d c 23=a1+9×(-3)
得前四項為 1, , ,
4 9 16 c a1=50
範例 2 2 第 40 項 a40=a1+39d=50+39×(-3)=-67
1 a1=7 3 設從第 n 項開始為負,則 50+(n-1) (-3)<0
a2=a1+d=7+(-2)=5 53
a3=a2+d=a1+2d=7+2×(-2)=3 c 50-3n+3<0 c n>
3
a4=a3+d=a1+3d=7+3×(-2)=1 又 n 為正整數,故取 n=18
a5=a4+d=a1+4d=7+4×(-2)=-1 類題
故此等差數列的前五項為 7,5,3,1,-1 設首項為 a1,公差為 d
一般項 an=an-1+d a9+a10+a11=45 ................ 1
=a1+(n-1)d 則
a17+a18+a19=165 ............. 2
=7+(n-1)×(-2) 由2-1得(a17-a9)+(a18-a10)+(a19-a11)=165-45
=9-2n,n 為正整數 c 8d+8d+8d=120 c 24d=120 c d=5
2 a1=5 代入1得(a1+8×5)+(a1+9×5)+(a1+10×5)=45
a2=a1r=5×3=15 c 3a1=-90 c a1=-30
a3=a2r=a1r2=5×32=45 故 a12=-30+(12-1)×5=25
a4=a3r=a1r3=5×33=135
範例 4
a5=a4r=a1r4=5×34=405 ∵數列〈an〉為等差數列,令公差為 d
故此等比數列的前五項為 5,15,45,135,405 A ○:∵bn-bn-1=(an+3)-(an-1+3)
一般項 an=an-1r=a1rn-1=5×3n-1,n 為正整數 =an-an-1=d
6 類題1
∴〈bn〉為等差數列
1 公差 d=a2-a1=7-4=3 1 1 1 1
∴a12=4+(12-1)×3=37 B ○:cn-cn-1= an- an-1= (an-an-1)= d
3 3 3 3
2 一般項 an=a1+(n-1)d ∴〈cn〉為等差數列
=4+(n-1)×3 C ×:反例:設〈an〉:-5,-2,1,4,7
=3n+1,n 為正整數 則〈dn〉=〈|an|〉:5,2,1,4,7 不是等差
3 ∵an=3n+1=88 c 3n=87 c n=29 數列
互動式教學講義.數學(2) 解答篇 3
頁碼 頁碼
D ×:承C, 9 範例 6
1 1 1 1 1 設〈an〉的公比為 r1,〈bn〉的公比為 r2
〈en〉=
an 〈 〉
: ,
-5 -2
,1, ,
4 7
A ○:∵
2an

an
=r1
不是等差數列 2an-1 an-1
E ○:fn-fn-1=(an+3n+5)-〔an-1+3(n-1)+5〕 ∴〈2an〉為等比數列
=(an-an-1)
+3=d+3 為等差數列 an2 an 2
故選ABE
B ○:∵
an-1 2=
( )
an-1
=r12

8 類題 ∴〈an2〉為等比數列
-1 1 C ×:設〈an〉:1,3,9,27,81,……
A ×:令 d= ,則 a100= >0,
100 100 則〈2an〉:21,23,29,227,281,……不是等
-899 比數列
但 a1000= <0
100 D ×:設〈an〉:1,3,9,27,
B ○:∵a100<0 c d<0 〈bn〉:1,4,16,64
∴a1000=a100+900d<0 則〈an+bn〉:2,7,25,91 不是等比數列
C ○:∵a1000=1+999d>0 an×bn an bn
E ○:∵ = × =r1×r2
-1 an-1×bn-1 an-1 bn-1
∴d >
999 ∴〈an×bn〉為等比數列
-1 故選ABE
得 a100=1+99d>1+99
999
>0
( ) 類題1

-1 -899 A ○:a9=a5r4 c 2r4=8 c r4=4


D ×:令 d= ,則 a1000= <0,
100 100 c r2=2 c r=a2(∵r>0)
1 1
但 a100= >0 B ×:a5=a1r4 c a1×4=2 c a1=
100 2
E ○:a1000-a10=(a1+999d)-(a1+9d)=990d C ○:a6=a5r=2a2
a100-a1= (a1+99d)-a1=99d D ×:由等比中項性質知 a8×a10=a92=82=64
∴a1000-a10=10(a100-a1) E ○:由等比中項性質知 a6×a12=a92=82=64
故選BCE 故選ACE
範例 5 類題2

1 .................... A ×:若 a1<a2=a1+d,則 d>0


ar2= 1
設首項為 a,公比為 r,則 2 ∴a3=a2+d >a2,故不可能同時成立
ar4=2 ......................2 B ○:取 b1=-1,b2=2,b3=-4,
2 則-1<2 且 2>-4
由 得 r2=4
1 C ×:取 a1=-1,a2=0,a3=1,
1 則-1+0<0,但 0+1>0
∵r<0,取 r=-2 代入1得 a=
8 D ○:若 b1b2<0 c b1(b1r)<0 c b12r<0
1 ∴r<0,則 b2b3=b2(b2r)=b22r<0
∴a6=ar5= ×(-2)
5
=-4,故第 6 項為-4
8 E ×:取 b1=4,b2=6,b3=9,但 4 無法整除 6
類題 故選BD
∵a2+a4=10 10 範例 7
∴a1r+a1r3=10 設三數為 a-d,a,a+d
c a1(1+r
r 2
)=10 ............................ 1 (a-d)+a+(a+d)=15

10 10 (a-d)×a×(a+d)=80
∵a3+a5= ∴a1r2+a1r4=
3 3 3a=15 .............................1
c
10 ........................ 2 a(a2-d 2)=80 ................2
c a1r2(1+r2)=
3 由1得 a=5,代入2得 5(25-d 2)=80
2 1 c d 2=9 c d=±3
得 r= ,代入1得 a1=27
1 3 1 當 d=3 時,此三數為 2,5,8
1 2 當 d=-3 時,此三數為 8,5,2
故首項 a1=27,公比 r=
3 故此三數為 2,5,8 或 8,5,2
4 互動式教學講義.數學(2) 解答篇
頁碼 頁碼
類題 2 a1=4,a2=2a1+3×2=2×4+6=14
a a3=2a2+3×3=2×14+9=37
設此三數為 ,a,ar,
r a4=2a3+3×4=2×37+12=86
a a5=2a4+3×5=2×86+15=187
+a+ar=39 ................... 1
r 故此數列的前五項為 4,14,37,86,187
則 ,由2得 a3=1000
a 3 a1=2,a2=3,a3=a2+a1=3+2=5,
×a×ar=1000 ............... 2
r a4=a3+a2=5+3=8,a5=a4+a3=8+5=13
10 故此數列的前五項為 2,3,5,8,13
∴a=10 代入1得 +10+10r=39
r 12 類題2

c 10+10r+10r2=39r c 10r2-29r+10=0 ∵a2=a1+1,a3=a2+4=a2+22


2 5 a4=a3+9=a3+32,a5=a4+16=a4+42
c(5r-2)(2r-5)=0 ∴r= 或
5 2 ∴at+1=at+t2,t=1,2,3,4
2 故選C
1 當 r= 時,此三數為 25,10,4
5 範例 9
5 1 實際畫畫看
2 當 r= 時,此三數為 4,10,25
2 可得 a1=6,a2=10,a3=14,a4=18
故此三數為 25,10,4 或 4,10,25 2 如下圖,可看出第 n-1 圖加上 1 塊黑色地磚與
11 範例 8 4 塊白色地磚就得到第 n 圖,
1 由初始值與遞迴關係依序代入可以得到 如以下虛線所表示
a1=5
a2=a1+9=5+9=14,a3=a2+9=14+9=23
a4=a3+9=23+9=32,a5=a4+9=32+9=41
故此數列的前五項為 5,14,23,32,41 第 n-1 圖 第n圖
b:此數列〈an〉表示首項為 5,公差為 9 的等差數列 因此 an=an-1+4,其中 nM2
2 a1=4,a2=3a1=3×4=12,a3=3a2=3×12=36 3 承2,可得數列〈an〉的遞迴式為
a4=3a3=3×36=108,a5=3a4=3×108=324 a1=6
故此數列的前五項為 4,12,36,108,324 an=an-1+4,其中 nM2
b:此數列〈an〉表示首項為 4, 4 〈an〉是一個首項為 6,公差為 4 的等差數列
公比為 3 的等比數列 因此,一般項 an=6+(n-1)×4=4n+2,n 為
2 2 5 正整數
3 a1=4,a2=3- =3- = ,
a1 4 2 13 類題
2 2 4 11 1 如右圖
a3=3- =3- =3- =
a2 5 5 5 ∵A1A2:A2B1=3:4
2 且 A1B1=1
2 2 10 23 3
a4=3- =3- =3- = , ∴A1A2= ,
a3 11 11 11 7
5 4
2 2 22 47 A2B1=A1D2=
a5=3- =3- =3- = 7
a4 23 23 23 3 2 4 5
c A2D2=zx
( ) ( )= 7
2

11 +
7 7
5 11 23 47 5
故此數列的前五項為 4, , , , ∴S2 邊長是 S1 邊長的 倍
2 5 11 23 7
類題1
5
1 由初始值與遞迴關係依序代入可以得到 c S2 周長是 S1 周長的 倍
7
a1=1,a2=a1+2×22=1+8=9 5 20 5 20 100
a3=a2+2×32=9+18=27 故 a1=4,a2= ×4= ,a3= × =
7 7 7 7 49
a4=a3+2×42=27+32=59 2 同理,第 n 個正方形的周長是第 n-1 個正方形
a5=a4+2×52=59+50=109 5 5
故此數列的前五項為 1,9,27,59,109 周長的 倍,故 an= an-1,nM2
7 7
互動式教學講義.數學(2) 解答篇 5
頁碼 頁碼
a1=4 15 類題1

3 數列〈an〉的遞迴式為 5 a1= 5
an= an-1,nM2
7 a2= a1 +(2×1+1)
5 a3= a2 +(2×2+1)
4 〈an〉是一個首項為 4,公比為 的等比數列
7 a4= a3 +(2×3+1)
5
( )
n-1 : :
故一般項為 an=4× ,n 為正整數
7 +)an=an-1+〔2(n-1)+1〕
範例 10 ∴an=5+2〔1+2+3+……+(n-1)〕+(n-1)
設第 n 區的首項為 an,末項為 bn (n-1)〔1+(n-1)〕
=5+2× +(n-1)
〈an〉:1,2,5,10,17,…… 2
〈bn〉:1,4,9,16,25,…… =5+(n-1)n+(n-1)=n2+4,n 為正整數
1 每一區的個數依序為 1,3,5,7,…… 類題2

∴第 10 區有 2×10-1=19 個數 a1 =1
2 第 9 區的末項為 b9=92=81 2
a2 = a1
∴第 10 區的首項為 82 1
3 所求為 82+83+……+100 3
a3 = a2
(82+100)×19 2
= =1729
2 4
a4 = a3
4 第 19 區的末項為 b19=192=361 3
∴365 是第 20 區內的第 4 個數 : :
14 類題1 n
×)an = an-1
1 第 1 組到第 9 組共有 n-1
1+2+3+……+9=45(個) 2 3 4 n
∴an=1× × × ×……× =n,n 為正整數
∴第 10 組內第 1 個數為 1+(46-1) ×2=91 1 2 3 n-1
10 16 範例 12
2 所求為 ×〔2×91+(10-1)×2〕=1000
2 1 a1=3
3 2021=1+ (n-1)×2 c n=1011 a2=3+2×3=3×(1+2)=9
∴2021 為第 1011 個數 a3=3+2×3+3×3=3×(1+2+3)=18
設 2021 在第 k 組 a4=3+2×3+3×3+4×3=3×(1+2+3+4)=30
則 1+2+……+kM1011 c ( k k+1)M2022 2 由題圖及 a1,a2,a3,a4 觀察可得
取 k=45 an=an-1+3n,nM2
∴2021 在第 45 組 a1=3
3 數列〈an〉的遞迴式為
類題2 an=an-1+3n,nM2
第 1 列有 1 個數,第 2 列有 2 個數, 4 由3知 a1= 3
第 3 列有 3 個數,……,第 98 列有 98 個數 a2= a1 +3×2
98(98+1) a3= a2 +3×3
∴1+2+3+……+98= =4851
2 a4= a3 +3×4
又第 99 列有 99 個數且第 99 列為從右至左算 : :
∴從左至右算第 67 個數字就是從右至左算的第 +)an=an-1 +3×n
99-67+1=33 個數字 ∴an=3×(1+2+3+……+n)
即為 4851+33=4884,故所求為 4884 n(n+1) 3
=3× = n(n+1)
範例 11 2 2
1 ∵an-an-1=3 3
故 an= n(n+1),n 為正整數
∴數列〈an〉為公差 3 的等差數列 2
故 an=5+ (n-1)×3=3n+2,n 為正整數 17 類題

an 1 a1=1
2 ∵ =3
an-1 a2=a1+4=1+4=5
∴數列〈an〉為公比 3 的等比數列 a3=a2+7=5+7=12
故 an=a1rn-1=5×3n-1,n 為正整數 a4=a3+10=12+10=22
6 互動式教學講義.數學(2) 解答篇
頁碼 頁碼
2 由題圖及 a1,a2,a3,a4 k
2 假設 n=k 時成立,即 ak=
觀察可得 an=an-1+ (3n-2),其中 nM2 k+2
3 a1=1,a2=a1+4,a3=a2+7, 則當 n=k+1 時,
a4=a3+10,…… k 2k+2
1+
∴〈an〉的遞迴式為 1+ak k+2 k+2
ak+1= = =
a1=1 3-ak k 2k+6
3-
an=an-1+ ,其中 nM2
(3n-2) k+2 k+2
4 a1= 1 2k+2 k+1 k+1
= = =
a2= a1 +4 2k+6 k+3 (k+1)+2
a3= a2 +7 ∴n=k+1 時亦成立
a4= a3 +10 n
故由數學歸納法可知,an= 對於所有正整
: : n+2
+)an= an-1+(3n-2) 數 n 都成立
∴一般項 an=1+4+7+……+(3n-2) 範例 14
n 1×(1+1)
= 〔1+(3n-2)〕 1 當 n=1 時,左式=1,右式= =1
2 2
3n2-n ∵左式=右式 ∴n=1 成立
= ,n 為正整數
2 k k+1)

2 設 n=k 時等式成立,即 1+2+3+……+k=
18 範例 13 2
1 a2=4a1+3=4×3+3=15 則當 n=k+1 時,
a3=4a2+3=4×15+3=63 左式=1+2+3+……+k+(k+1)
a4=4a3+3=4×63+3=255 k k+1)
( k k+1)+2(k+1)

= +(k+1)=
2 由於 a1=41-1,a2=42-1,a3=43-1,a4=44-1 2 2
∴推測 an=4n-1,n 為正整數 (k+1)(k+2)
= =右式
3 用數學歸納法證明如下: 2
1 當 n=1 時,a1=3=4-1 ∴n=k+1 時亦成立
∴n=1 時成立 故由數學歸納法可知,原式對所有正整數 n 均成立
2 假設 n=k 時成立,即 ak=4k-1 20 類題1

則當 n=k+1 時, 1 當 n=1 時,
ak+1=4ak+3=4(4k-1)+3=4k+1-1 1×(4×12-1)
左式=12=1,右式= =1
∴n=k+1 時亦成立 3
故由數學歸納法可知 ∵左式=右式 ∴n=1 時成立
an=4n-1 對於所有正整數 n 都成立 2 假設 n=k 時等式成立,
19 類題 k 4k2-1)

即 12+32+……+(2k-1) 2

1 3
1+
3 4 2 則當 n=k+1 時,
1 a2= = = ,
1 8 4 左式=12+32+……+(2k-1) 2
+(2k+1) 2

3-
3 k 4k2-1)
( 2
= +(2k+1)
2 3 3
1+ 1+ 2
4 6 3 5 8 4 k 2k-1)
( (2k+1)+3(2k+1)
a3= = = ,a4= = = =
2 10 5 3 12 6 3
3- 3- 2
4 5 (2k+1) (2k -k+6k+3)

1 2 3 4 3
2 由於 a1= ,a2= ,a3= ,a4=
3 4 5 6 (2k+1) (k+1) (2k+3)

n 3
∴推測 an= ,n 為正整數 2
n+2 (k+1) 〔4(k+1) -1〕
= =右式
1 1 3
3 1 當 n=1 時,a1= =
1+2 3 ∴n=k+1 時亦成立
∴n=1 時成立 故由數學歸納法可知,原式對所有正整數 n 均成立
互動式教學講義.數學(2) 解答篇 7
頁碼 頁碼
類題2 22 範例 16
2 2
A ×:n=41 時,41 -41+41=41 不為質數 令 ak,bk 分別表示第 k 步時,剩下的線段數與總長
B ×:n=3 時,53>33+43 度,則
C ×:n=11 時,113-11>1000 a1=2,a2=4,a3=8,……
○:28n+1=
(24) 2n
×2=(16)2n
×2 1 1 2 2 1 3 3
() ()
D
2n b1= ×2,b2= ×2 ,b3= ×2 ,……
∵(16) 的個位數字必為 6 3 3 3
∴28n+1 的個位數字必為 2 1 9 9 29
又 24n=(24) n
=16n 個位數字必為 6
故 a9=29,b9=
()
3
×2 = 9
3
∴28n+1+24n 的個位數字必為 2+6=8 29

E ×:n=3 時,23<32 ( )
c數對(m﹐n)= 29﹐ 9
3
故選D 類題1

21 範例 15 ∵跳動 100 次後,共跳動(3+1)×50=200(片)


3
1 a1=1 -1+3=3 且號碼 6 個一循環
a2=23-2+3=9 ∴所在蓮葉的號碼為(1+200)÷6 之餘數為 3
a3=33-3+3=27 故跳跳蛙在跳動 100 次後,其所在蓮葉的號碼為 3 號
a4=43-4+3=63 23 類題2

a5=53-5+3=123 1,2,……,9:9 個
推測 an=n3-n+3 必為 3 的倍數,其中 n 為正整數 10,11,……,99:2×90=180(個)
2 1 當 n=1 時,13-1+3=3 為 3 的倍數 100,101,……,699:3×600=1800(個)
∴n=1 時成立 700,701,……,709:3×10=30(個)
2 假設 n=k 時推測成立, 共 9+180+1800+30=2019(個)
即 k3-k+3=3t,t 為正整數 故自左算起的第 2020 個數字為 7
則當 n=k+1 時, 類題3
3
(k+1) -
(k+1)+3 1 ∵從小到大排列後,女生所認識的男生人數為首
3 2
=k +3k +3k+1-k-1+3 項為 8,公差為 2 的等差數列
=(k3-k+3)+(3k2+3k) ∴第 1 位女生認識 8 位男生
=3t+3(k2+k)=3(t+k2+k)為 3 的倍數 第 2 位女生認識 8+2=10 位男生

∴n=k+1 時亦成立
故由數學歸納法可知,推測對所有正整數 n 都成立 第 20 位女生認識 8+(20-1)×2=46 位男生
類題 故選C
1 當 n=1 時,f(1)=33+23=35=7×5 為 p 的倍數 2 設參加舞會的女生人數為 n 人
當 n=2 時,f(2)=35+24=259=7×37 為 p 的倍數 由題意可知,最後一位女生認識的男生數為
∴p 為 7×5 和 7×37 的公因數 8+2(n-1)=2n+6
c(7×5﹐7×37)=7 即所有男生總數為 2n+6 人,又男女生共有 300 人
又 p 為正質數 ∴猜測 p 為 7 所以可知 n+(2n+6)=300
2 證明“對一切的正整數 n,f(n)恆為 7 的倍數” c 3n=294
=35 c (1)
1 當 n=1,f(1) f 為 7 的倍數 ∴n=98
∴n=1 時成立 故參加舞會的女生人數共有 98 人
2 假設 n=k 時成立,即 (k) f 為 7 的倍數
c令 32k+1+2k+2=7t(t 為正整數) 1-1
則當 n=k+1 時,
基礎題
f
(k+1) =32(k+1)+1+2(k+1)+2
=9×32k+1+2×2k+2 24 1 1 將 n=1,2,3,4,5 分別代入 an=-2n+3
=7×32k+1+2(32k+1+2k+2) 得前五項為 1,-1,-3,-5,-7
=7×32k+1+2×7t 2 將 n=1,2,3,4,5 分別代入
1 n-1
=7(32k+1+2t)為 7 的倍數
∴n=k+1 時亦成立
an=3×
2 ()
故由數學歸納法可知,對於所有正整數 n, 3 3 3 3
得前五項為 3, , , ,
f 恆為 7 的倍數都成立 ∴p=7
(n) 2 4 8 16
8 互動式教學講義.數學(2) 解答篇
頁碼 頁碼
2 1 公差 d=1-5=-4,又首項 a1=5 8 由 an=3an-1 可得數列〈an〉為公比 3 之等比數列
∴一般項 an=a1+(n-1)d ∴an=a1×3n-1=5×3n-1,n 為正整數
=5+(n-1)×(-4) 9 1 a1=1,a2=a1+6,a3=a2+12,a4=a3+18
=-4n+9,n 為正整數 a1=1
∴〈an〉的遞迴式為
-12 an=an-1+6(n-1),nM2
2 公比 r= =-3,又首項 a1=4
4 2 a2 =a1+6×1
∴一般項 an=a1rn-1=4×(-3)n-1,n 為正整數 a3 =a2+6×2
3(a2+a4+a6+a8+a10)-(a1+a3+a5+a7+a9) a4 =a3+6×3
=5d=220-240=-20 : :
c d=-4 +)a10=a9+6×9
又 a1+a3+a5+a7+a9=5a1+2d+4d+6d+8d=240 ∴a10=a1+6(1+2+3+……+9)
c 5a1+20d=240 c 5a1-80=240 ∴a1=64 9(1+9)
=1+6×
9=ar3 .......................1 2
25 4
81=ar5 ....................2 =1+6×45=271
2 3 a2= a1 +6×1
得 r2=9 ∵a<0 ∴r<0 c r=-3
1 a3= a2 +6×2
1 1 a4= a3 +6×3
代入1得 a=- ,b=ar= - (-3)=1
3 ( ) 3 : :
1 2 +)an=an-1+6(n-1)
故 a+b=- +1=
3 3 ∴an=a1+6×〔1+2+3+……+(n-1)〕
5 ∵三數成等差數列 n(n-1)
=1+6× =3n2-3n+1
∴設此三數為 a-d,a,a+d 2
c(a-d)+a+ (a+d)=15 c 3a=15 ∴a=5 故 an=3n2-3n+1,n 為正整數
故三數為 5-d,5,5+d 27 0 (1)
f =3×53+24=391=17×23 為 p 的倍數
又新三數為 5-d,4,12+d 成等比數列 f
(2)=3×55+27=9503=13×17×43 為 p 的倍數
c 42= (5-d) (12+d) ∴p 為 17×23 和 13×17×43 的公因數
c 16=60+5d-12d-d 2 c(17×23﹐13×17×43)=17,而 p 為質數
c d 2+7d-44=0 c(d+11) (d-4)=0 ∴p=17,故選D
∴d=-11 或 4 進階題
1 當 d=-11 時,此三數為 16,5,-6 q 1 由觀察知
2 當 d=4 時,此三數為 1,5,9 1,2,3,4,……c 1,2,3,4,……
故原三數為 16,5,-6 或 1,5,9 2,3,4,5,……
1 c 1+1,2+1,3+1,4+1,……
6 a1=
3 c a1=1×(1+1),a2=2×(2+1),
1 2 a3=3×(3+1),a4=4×(4+1),……
1-
1-a1 3 3 2 ∴一般項 an=n(n+1),n 為正整數
a2= = = =
3-4a1 1 4 5 2 由觀察知
3-4× 3-
3 3 分子:2,4,8,16,……
2 3 c 21,22,23,24,……
1-
1-a2 5 5 3 分母:3,7,11,15,……
a3= = = =
3-4a2 2 8 7 c 4×1-1,4×2-1,4×3-1,4×4-1,……
3-4× 3-
5 5 正負號:-,+,-,+,……
n c(-1) 1
,(-1) 2
,(-1)3 4
,(-1),……
c an= 1
2n+1 2
c a1=(-1) 1
× ,
2020 4×1-1
故 a2020= c p=4041,q=2020
4041 2 22
a2=(-1)× ,
26 7 d=an-an-1=5 4×2-1
∴an=a1+(n-1)×d=3+(n-1)×5 3 23
a3=(-1) × ,
=5n-2,n 為正整數 4×3-1
互動式教學講義.數學(2) 解答篇 9
頁碼 頁碼
4 24 t 由題意知 a2=2+d,(2+d)r=12 ..................... 1
a4=(-1)×
4×4-1 a3=2+2d,(2+2d)r2=54 ................ 2
2n 12 (2+d)2 2
r 122 (2+d) 2
8
∴一般項 an= n
(-1) × ,n 為正整數 得 2 = c =
4n-1 2 (2+2d)r 54 2+2d 3
1 2 1 3 2 1 c 8(2+2d)=3(2+d)2
28 w 1 觀察
1( )(
1 2 )(
, , , , , ,
1 2 3 ) c 3d 2-4d-4=0
4 3 2 1 c(3d+2) (d-2)=0
( , , , ,……
1 2 3 4 ) 2
c d=- 或 2
7 3
∵ 之分子、分母和為 17
10 ∵c1<c2 c d>0 ∴取 d=2
7 代回1得 r=3
∴ 在第 16 組的第 10 項
10 故 d4=(2+3d)r3=8×27=216
即在此數列的第 1+2+3+……+15+10 30 y a1=6
15×(15+1) a2=a1+9=a1+3×3
= +10
2 a3=a2+12=a2+3×4
=120+10=130(項) a4=a3+15=a3+3×5
2 第 1 組至第 n 組共有 : :
n(n+1) +)a15=a14+3×16
1+2+3+……+n= (項)
2 a15=6+3(3+4+5+……+16)
(n+1)
n (n+1)(n+2) (3+16)×14
令 <100N , =6+3× =405(片)
2 2 2
則 n=13 歷屆試題
13×14 u 設 a1,a2,a3,a4 之公差為 d
又 =91
2 則 b1=2a1,b2=2a1+d=2a1.2d,
∴第 100 項為第 14 組(此組分子、分母之和 b3=2a1+2d=2a1.(2d)2
,b4=2a1+3d=2a1.(2d)3
為 15)之第 9 項 A ○:公比 r=2d
6 B ○:∵0<a1<2 且 a3=a1+2d=4 ∴1<d<2
故為
9 c 2<2d<4,即 2<r<4 ∴b2=b1r>b1
e 1 由題意知 a2-a=3(a1-a) b3 16
C ○:b3=2a3=24=16,b2= =
c 13-a=3(5-a)c 2a=2 c a=1 r r
2 承1,an-1=3(an-1-1),nM2 又 2<r<4 ∴b2>4
令 bn=an-1 ∴bn-1=an-1-1 D ○:b4=b3r=16×r,又 2<r<4 ∴b4>32
bn E ○:b2×b4=(b3)2=162=256
即 bn=3bn-1 c =3
bn-1 故選ABCDE
c〈bn〉為首項 b1=a1-1=4, 31 i ∵f(x)為二次多項式 ∴令 (x) f =ax2+bx+c
公比 r=3 之等比數列 又 an=an-1+f(n-2)
故 b8=b1×r7=4×37=8748 a2=a1+f(0) 2=1+c ........................... 1
c a8-1=8748 c a3=a2+f(1)c 5=2+a+b+c ............... 2
c a8=8749 a4=a3+f(2) 12=5+4a+2b+c ......... 3
3a1-1 2 解1、2、3得 a=b=c=1
29 r 1 a1=1,a2= = ,
4a1-1 3 又 a5=a4+f(3)=12+9a+3b+c=25
3a2-1 3 3a3-1 4 o ∵a1,a2,……,a9 為等差數列
a 3= = ,a4= =
4a2-1 5 4a3-1 7 ∴設公差為 d
2 3 4 a1 x-a2 y+2a3 z=k+1 .................. 1
∴序組 (a2﹐a3﹐a4)

= ﹐ ﹐
3 5 7 ) a4 x-a5 y+2a6 z=-k-5 .............. 2
2 3 4 a7 x-a8 y+2a9 z=k+9 .................. 3
2 ∵a1=1,a2= ,a3= ,a4=
3 5 7 2-1得 3dx-3dy+6dz=-2k-6
n 3-2得 3dx-3dy+6dz=2k+14
∴觀察得 an= ,n 為正整數
2n-1 ∵方程組有解 ∴-2k-6=2k+14 c k=-5
10 互動式教學講義.數學(2) 解答篇
頁碼 頁碼
32 p A ○:〈an〉為等差數列且公差為正 f 設 30 分鐘內有 x 次 A 廣告,則有 x-1 次 B 廣告
∴a1<a2<a3<…… c〔x+(x-1)〕.T=30
又 bn=-an ∴b1>b2>b3>…… c(2x-1).T=30,且 2x-1 為奇數
B ×:反例:若〈an〉 :-2,-1,0,1,2,…… 2x-1 1 3 5 15
2
∵cn=an ∴〈cn〉:4,1,0,1,4,…… T 30 10 6 2
C ×:反例:若〈an〉:1,2,3,4,5,…… 故選BD
(公差為 1)
∵dn=an+an+1 1-2 級 數
∴〈dn〉:3,5,7,9,……(公差為 2) 35 範例 1
D ○:∵en=an+n 1 a1=S1=20-5-12=3
∴〈en〉:a1+1,a2+2,a3+3,a4+4,…… 2 當 nM2 時,
公差為 en+1-en an=Sn-Sn-1
= (an+1+n+1)-(an+n) =(20n2-5n-12)-〔20(n-1)2-5(n-1)-12〕
=an+1-an+ (n+1-n)=a+1 =40n-25
E ×:反例:若〈an〉:1,2,3,4,5,…… 3,n=1
(公差為 1) ∴an=
40n-25,nM2
a1+a2+……+an b:此數列非等差數列
∵fn=
n 類題
3 5 1 1
∴〈fn〉:1, ,2, ,3,…… 1 a1=S1=1- =
2 2 2 2
1
( 公差為
)2
2 當 nM2 時,
1 1
故選AD (
an=Sn-Sn-1= 2n-1-
2 )(
- 2n-2-
2 )
a 原售價為 200×5=1000(元) n-1 n-2 n-2
=2 -2 =2×2 -2 =2 n-2 n-2

此時利潤為 1000-200=800(元) 由1、2得 an=2n-2,n 為正整數


1 b:此數列為等比數列
第一次降價,利潤為 800× =400(元)
2
範例 2
1 1 ∵首項 a1=2,公差 d=3
第二次降價,利潤為 400× =200(元)
2 又 an=a1+(n-1)d c 35=2+(n-1)×3
1 c n=12
第三次降價,利潤為 200× =100(元)
2 12×(2+35)
故調降三次後該模型的售價為 200+100=300(元) 故等差級數的和為 =222
2
33 s 由題意可知,每秒位置如下 1
4,8,12,16,20,24,24,24, 2 ∵首項 a1=8,公差 d=- ,項數 n=14
2
28,32,36,40,44,48,48,48 故等差級數的和為
由上述可知每 8 秒(一週期) 移動 24 單位長 14 1
又 116÷24=4……20 2
× 2×8+(14-1)× -
2 ( )
即經過 4 個週期再加 20 個單位長 13 133
又移動 20 個單位長需要 5 秒 (
=7× 16-
2

2 )
故移動 8×4+5=37 秒後會抵達數線上坐標為 36 類題1
116 的位置 設此等差數列的首項為 a1,
d 每一循環:「藍-白-紅-白」 (-2)=-8 c a1=-6
則 a2=a1+(2-1)
共 5+2+6+2=15(秒) 又 an=a1+(n-1)d c-38=-6+(n-1)
(-2)
由 99÷15=6……9 c(n-1)(-2)=-32 c n-1=16 c n=17
知六個循環後共 15×6=90(秒) 17×〔(-6)+(-38)〕
又藍:91∼95 秒,白:96∼97 秒, 前 17 項和 S17=
2
紅:98∼103 秒,白:104∼105 秒 17×(-44)
∴啟動後第 99 至 101 秒之間皆為紅燈 = =-374
2
故選C 故等差級數的和為-374
互動式教學講義.數學(2) 解答篇 11
頁碼 頁碼
類題2 200 207
c n-1< c n< ~29.6
設此等差數列的首項為 a1,公差為 d,則 7 7
a1+d=10 ∴取 n=29,即前 29 項之和為最小
〔a1+
3 (a1+2d)〕 10〔a1+(a1+9d)〕 29×〔2×(-200)+(29-1)×7〕
= 又 S29=
2 2 2
a1+d=10 29×(-204)
c = =-2958
a1+6d=0 2
解得 a1=12,d=-2 類題3

故前 13 項的和 (a1+a51)×51
A ○:S51=0,又 S51=
13
〔2×12+(13-1)×(-2)〕 2
S13= =0
2 ∴a1+a51=0
〔另解〕 c a1+(a1+50d)=0 c a1+25d=0 c a26=0
∵前 3 項和=前 10 項和 B ×C ○:∵a26=0 且 a41=41 c d>0 且 a1<0
∴a4+a5+a6+a7+a8+a9+a10=0 D ○:由等差中項性質知 a2+a50=2a26=0
由等差中項性質知 a4+a10=a5+a9=a6+a8=2a7 E ×:承D,由等差中項性質知 a3+a49=0,
c 7a7=0 c a7=0 又〈an〉為遞增數列
又 S13=13×a7=0 ∴a46<a49 c a3+a46<0
∴前 13 項之和為 0 故選ACD
範例 3 38 範例 4
1 首項為 6,公差為 4, 設首項為 a1
第 n 項為(4n+2)的等差級數 1 a18=a1+(18-1)×2=56 c a1+34=56
其前 n 項和 c a1=22
Sn=6+10+14+……+(4n+2) ∴a25=22+(25-1)×2=70(個)
n×〔6+(4n+2)〕 n×(4n+8) 25×〔2×22+(25-1)×2〕
= = 2 S25=
2 2 2
=(2n+4)×n=2n2+4n,n 為正整數 25×(44+48)
= =1150(個)
1 2
2 首項為 8,公差為- 的等差級數
2 類題1

其前 n 項和 每層積木的個數為一個首項 a1=1,公差 d=2 的等


1 差數列

S n=
n× 2×8+(n-1)× -
( )2 ∴第 50 層 a50 有 1+(50-1)×2=99 個積木
2 堆 50 層需要 1+3+5+7+……+99
n 33 50×(1+99)



n× - +
2 2 ) =
2
=2500 個積木

2 類題2

-n(n-33) -n2+33n 令第 1 排到第 20 排的座位數量分別為


= = ,n 為正整數
4 4 a1,a2,……,a20 個
37 類題1 且 an+1=an+k,n=1,2,……,19
首項 a1=60,公差 d=57-60=-3 即〈an〉為等差數列
n〔2×60+(n-1)×(-3)〕 ∵a10+a11=235
∴前 n 項和 Sn= =600
2 ∴紫色區共有
cn(-3n+123)=1200 c-3n2+123n-1200=0 a1+a2+……+a19+a20
c n2-41n+400=0 c(n-16)
(n-25)=0 =(a1+a20)+(a2+a19)+……+(a10+a11)
∴n=16 或 25 =10(a10+a11)=2350 個座位
類題2 故選B
∵公差 d=7>0, 39 範例 5
故欲使 Sn=a1+a2+……+an 為最小 1 首項 a1=3,公比 r=2,項數 n=10
則 a1,a2,……,an 均須為負數 ∴an 必為負數 3(210-1)
故前 10 項的和 S10= =3069
∴an=-200+(n-1)×7<0 2-1
12 互動式教學講義.數學(2) 解答篇
頁碼 頁碼
1 類題1
2 首項 a1=7,公比 r=-
2 設項數為 n,又首項 a1=2,公比 r=3
1 n-1 7 2(3n-1)
( )
又 an=a1r n-1 c 7× -
2
=-
128
則 Sn=
3-1
=2186 c 3n=2187 ∴n=7

1 n-1 1 1 7 故末項 a7=2×37-1=1458


( )
c -
2
=-
( )
128
= -
2 類題2

c n-1=7 c n=8 設其公比為 r,項數為 n,又首項為 3


故等比級數的和為 c an=3×rn-1=-384
1 8 1 c rn-1=-128 ............................... 1
7× 1- -
( ) 2


7× 1-
256 ) =
595

3(1-rn)
=-255 ................... 2
1 3 128 1-r
1- -
( )2 2 由1得 rn=rn-1×r=-128r
類題 1-(-128r)
代入2得 =-85
1 首項 a1=1,公比 r=2 1-r
又 an=a1r n-1 c 1×2n-1=1024 c 2n-1=210 c-85+85r=1+128r
c n-1=10 c n=11 ∴r=-2 代入1得(-2)n-1=-128=(-2)
7

故等比級數的和為 故 n-1=7 c n=8


1×(211-1) 11 41 範例 7
=2 -1=2048-1=2047
2-1 點 P7 的坐標為
-81 1 1 1 2 1 3 1 4 1 1
( )( )( )( )( )
5 6
2 首項 a1=243,公比 r= =- 1- + - + - +
243 3 2 2 2 2 2 2
1 1 1 7
( ) ( )
n-1
又 an=a1r n-1 c 243× - = 1× 1- -
3 3 2 43
6
= =
1 1 1 64
( ) ( ) ( )
n-1
c - = - c n-1=6 c n=7 1- -
3 3 2
故等比級數的和為 類題

1 7 A1B 3
243× 1- -
( ) 3

AB

4
1 3
1- -
( ) 3
∴下一個三角形面積為上一個的
4

3 1 7 3 1 1
= ×243× 1- -
4 ( ) 3 4 (
= × 243+
9 ) 又△ABC= ×4×4a3 =8a3
2
3 2188 547 3 3
= × = 1 △A1BC= ×△ABC= ×8a3 =6a3
4 9 3 4 4
40 範例 6 3 3
2 同理,△A2BC= ×△A1BC= ×6a3
2 4 4
1 首項為 1,公比為- 的等比級數
3 3 3 2
2 n
△A3BC= ×△A2BC=
4 4
×6a3
( )
其前 n 項和 Sn=
1× 1- -
( )
3 3
∴Rk 為公比 r= 的等比數列,其中 R1=6a3
2 4
1- -
3( ) 3 4
3 2 n
6a3 × 1-
4 ()
= × 1- -
5 ( )
3
,n 為正整數 c R1+R2+R3+R4=
1-
3
2 由題意知 a1=9,r=3 4
9×(3n-1) 525 a3
c Sn= =3276 =
3-1 32
c 9×(3n-1)=6552 42 範例 8
c 3n-1=728 80×(80+1)
1 1+2+3+……+80= =3240
c 3n=729 c n=6 2
互動式教學講義.數學(2) 解答篇 13
頁碼 頁碼
2 12+22+32+……+302 44 範例 10
30×(30+1)×(2×30+1) 30×31×61
= =
6 6
=9455
3 83+93+103+……+203
=(13+23+33+……+203)-(13+23+33+……+73) 所求本利和為
20×21 2 7×8 2
( )( )
7 6
30000(1+3 %) +30000(1+3 %) +……
= - =44100-784=43316
2 2 +30000(1+3 %)
類題 =30000×1.037+30000×1.036+……
令 n×n 方格中的數字總和為 an +30000×1.03
1 2 3 4 1.03(1.037-1)
1 2 3 =30000×
1 2 1 2 3 3 1.03-1
1 1 2 2
1 1 1 2 2 2 30000×1.03×(1.2299-1)
1 1 1 ~
1 1 1 1 0.03
∵1+2+1=4=22 ~236797
1+2+3+2+1=9=32 故本利和為 236797 元
1+2+3+4+3+2+1=16=42 類題1

∴a10=12+22+……+102
10
(10+1)
(2×10+1)
= =385
6
43 範例 9
1×2+2×3+3×4+……+99×100 年利率 1.8 %相當於月利率 1.8 %÷12=0.15 %
=1×(1+1) +2×(2+1)+3× (3+1)+…… 小如每月存 4500 元,
+99×(99+1) 30 個月的本利和為
30 29
=(12+22+32+……+992) +(1+2+3+……+99) 4500(1+0.15 %) +4500(1+0.15 %) +……
99×(99+1) ×(2×99+1) 99×(99+1) +4500(1+0.15 %)
= + 1.0015(1.001530-1)
6 2 =4500×
99×100×199 99×100 1.0015-1
= + 1.0015(1.046-1)
6 2 ~4500×
=328350+4950=333300 1.0015-1
類題 ~138207
令 n×n 方格中的數字總和為 an 故 30 個月的本利和為 138207 元
類題2
1 2 3 4
1 2 3
1 2 2 2 3 4
2 2 3
2 2 3 3 3 4
3 3 3
4 4 4 4
∴a15=1×1+2×3+3×5+4×7+……+15×29
=1×
(2×1-1)+2×(2×2-1) 設每年年底應還 x 元
+3×(2×3-1)+4×(2×4-1)+…… x 1+10 %)2 元
則第一年年底還的錢經複利計算會有 (
+15×
(2×15-1) 第二年年底還的錢經複利計算會有 (x 1+10 %)元
(1 +22+32+……+152)
=2× 2 第三年年底還的錢經複利計算會有 x 元
-(1+2+3+……+15) c 66200(1+10 %)3
2
15×(15+1)×(2×15+1) 15×(15+1) =x(1+10 %) +x(1+10 %)+x
=2× - c 66200×1.331=x(1.12+1.1+1)
6 2
15×16×31 15×16 c 3.31x=66200×1.331
=2× - c x=26620
6 2
=2480-120=2360 故每年年底應還 26620 元
14 互動式教學講義.數學(2) 解答篇
頁碼 頁碼
45 範例 11 2 1 1
∴第二層的體積為 p×(a2 ) × =p×2×
1 10 10
∵首項 a1=1,公比 r=
3 1 1
第三層的體積為 p×12× =p×1×
1 10 10
( ) = 3 × 1- 1
n
1× 1-
3 故可推得結婚蛋糕的總體積為
(3)
n
∴前 n 項和 Sn=
1 2 1 1 1 1
1-
3 (
p× 4+2+1+ + + ×
2 4 8 )10
3 3 3 1 n 3 1 1 1 6
() () ()
n
-Sn= - × 1- = × < 4× 1-
2 2 2 3 2 3 1000 2 1
=p× ×
1 2 1 10
()
n
c < c 3n>1500 1-
3 3000 2
又 36=729,37=2187 c n>6.…… 4p 63
= ×
故最小自然數 n 為 7 10 32
類題 63
= p 立方公尺
1 5 1 1 1 80
()
2
由遞迴式可知,a1= ,a2= 2 - = =
4 4 4 16 4 47 類題1

5 1 1 1 1 P1P2+P2 P3+P3 P4+……+P19 P20


() ()
3 n
a3= 3 - = = ,可得 an=
4 16 64 4 4 =1+a2 +(a2 )2+(a2 ) 3 18
+……+(a2 )
19
1 1 n 1×〔(a2 ) -1〕
4
1-
() 4 1 1 1 n

a2 -1
∴前 n 項和 Sn=
1
= -
3 3 4 ( ) (a2 ) 19
-1
a2 -1
1- =
4
1 1 1 1 n 1 1 1 n 〔(a2 ) 19
-1〕 (a2+1)
|
c Sn- = -
3 | | ( ) | | ( )|
3 3 4
- = -
3 3 4

(a2-1)
20
(a2+1)
19
1 1 1 (a2 ) +(a2 ) -a2 -1
()
n
= N =
3 4 3000 2-1
1 n 1 =1024+512a2 -a2 -1
()
c
4
N
1000
c 4nM1000
=1023+511a2
又 44=256,45=1024,故最小正整數 n 為 5 類題2

46 範例 12
內切圓直徑為其所切正方形的邊長,
又恰為其內接正方形的對角線長
1
a2
可推得每層蛋糕半徑為下層蛋糕半徑的 倍
設每年年底應還 x 萬元
2 1 1 則第 1 年年底還的錢經複利計算會有
a2 a2 a2
1 設首項 a1=2,a2= = a1,公比 r= 19
x 1+2 %)
( 萬元
a1=2 第 2 年年底還的錢經複利計算會有
c 1 ,故選D x 1+2 %)18 萬元

a2
an= an-1,nM2

2 ∵第一層的半徑為 2 公尺, 第 20 年年底還的錢經複利計算會有 x 萬元


20
1 1000(1+2 %)
又每一層高均為 公尺 19 18
10 =x(1+2 %) +x(1+2 %) +……+x
1 1 =x×(1+1.02+1.02 +……+1.0219)
2

∴第一層的體積為 p×22× =p×4×


10 10 1×(1.0220-1)
c 1000×1.49=x×
2 1.02-1
a2
同理第二層的半徑為 =a2 公尺,
0.49x
a2
c 1490=
0.02
a2
第三層的半徑為 =1 公尺
∴x~60.8163~61
故每年年底應還 61 萬元
互動式教學講義.數學(2) 解答篇 15
頁碼 頁碼
進階題
1-2 50 9 x 坐標=(1+2-3-4)+(5+6-7-8)+……
+(17+18-19-20)
基礎題
=(-4)+(-4)+……+(-4)
a1+3d=-9 a1=-15
48 1 c =(-4)×5=-20
a1+9d=3 d=2
y 坐標=(1-2-3+4)+(5-6-7+8)+……
a99=-15+98×2=181
+(17-18-19+20)
c a1+a3+a5+……+a99
=0+0+……+0=0
50×(a1+a99) 50×〔(-15)+181〕
= = =4150 ∴走完第 20 次所停在的坐標為(-20﹐0)
2 2
0 A ○:∵S2016=a2016+S2015 且 S2016=a2016
2 a2+a3+a10+a11
∴S2015=0
=(a1+d)+ (a1+2d)+(a1+9d)+(a1+10d)
c S2015=a1+a2+……+a2015
=4a1+22d=36 c 2a1+11d=18
a1+a2015 a2+a2014
又 a1+a12=a1+ (a1+11d)=2a1+11d=18
(a1+a12)
×12
=2×
( 2

2
故前 12 項的和為 =108 a1007+a1009

〔另解〕
2 +……+
2 ) +a1008

=2015a1008=0
由等差中項性質知 a2+a11=a3+a10=a1+a12
c a1008=0
又 a2+a3+a10+a11=36 c a1+a12=18
又 a2016=a1008+(2016-1008)d=2016
(a1+a12) ×12
故前 12 項的和為 =108 c 1008d=2016 c d=2>0
2
B ×:∵a1008=a1+1007d=0 ∴a1=-2014
n
〔10+ (n-1)×3〕
3 =55 c 3n2+7n-110=0 C ×:S1008=S1007+a1008=S1007
2
D ○:由A知 S2015=a1008=0
c(n-5) (3n+22)=0 c n=5(項)
E ○:∵a1,d 皆為偶數
4 10×5+9×4+8×3+7×2+6×1=130(個)
∴每項皆為偶數,即 Sn 必為偶數
故選C
故選ADE
49 5 首項 a1=1,公比 r=-2,項數 n=10
BD 1
a2
10
1×〔1-(-2) 〕 1-1024 51 q ∵ =
∴S10= = =-341 BC
1-(-2) 3
1
a2
6 首項 a1=9,an=a1rn-1=2187 ∴下一個正三角形邊長為上一個的 倍

a3
a1(1-rn) a1-a1rn 9-2187r
Sn= = = =3276 1 2 1
1-r 1-r
c 9-2187r=3276-3276r
1-r 1 a 1=
4
×162=64a3 ,公比 r=
( ) a2

2
1 1
c 1089r=3267 c公比 r=3 an=an-1× c an= an-1
2 2
∵9×3n-1=2187 c 3n-1=243=35 c n=6
a1=64a3
7 1 ∵T1 邊長為 1 ∴周長為 1×4=4
∴遞迴定義式為 1
2 ∵T2 邊長為 1+1=2 ∴周長為 2×4=8 an= an-1,nM2
2
3 由1,2推得 Tn 周長=2Tn-1 周長且 T1=4
1
()
5
∴前 8 個正方形周長總和為 2 a6=a1× =2a3
2
4(28-1)
4+8+16+……+4×27= =1020 1 6

8 設年利率為 r
2-1
3 a1+a2+……+a6=
64a3 × 1-
() 2
2 1
由題意知 12000×(1+r) =5800×(1+r)+9200 1-
2
c 12000×(1+r)2-5800×(1+r)-9200=0
=126a3
c 60
(1+r)2-29(1+r)-46=0
w 設阿雅每個月要還 x 元,可得
c〔20(1+r)-23〕 〔3(1+r)+2〕=0 10
20000×(1+0.01)
23 2
c 1+r= =1.15 或 1+r=- (不合) =x〔(1+0.01)9+(1+0.01)8+……+(1+0.01)+1〕
20 3 10
x×〔(1.01) -1〕
c r=0.15=15 %,故年利率為 15 % c 20000×(1.01)10

1.01-1
16 互動式教學講義.數學(2) 解答篇
頁碼 頁碼
10
200×(1.01) 200×1.1046 220.92 y A ○:29=1+(15-1)×2 ∴共有 15 列
c x= 10 ~ =
(1.01) -1 0.1046 0.1046 B ○:第 8 列由左至右算第 8 個數為 8+7=15
~2112 15〔2+(15-1)×2〕
C ×:共有 =225 個數
故所求為 2112 元 2
52 e 1 ∵a2=a1+3×60=0+3=3 D ×:a=15
a3=a2+3×61=3+18=21 ∵第 15 列有 1+(15-1)×2=29 個數
∴a4=a3+3×62=21+108=129 ∴b=15+28=43 c b-a=28
2 由1可推得 an=an-1+3×6n-2,nM2 不為 3 的倍數
a 1= 0 E ○:由題意知
a2= a1 +3×60 第 1 列總和為 1
a3= a2 +3×61 第 2 列總和為 2+3+4=9=32
a4= a3 +3×62 第 3 列總和為 3+4+5+6+7=25=52
第 4 列總和為 4+5+6+7+8+9+10

+)an=an-1+3×6n-2 =49=72
an=3×(60+61+……+6n-2) ∴第 15 列總和為 292=841
1×(6n-1-1) 故選ABE
=3×
6-1 歷屆試題
3 54 u 前 n 項和 Sn=2n+1×(n2-2n)
= × (6n-1-1),n 為正整數
5 1 a1=S1=21+1(1-2)=-4
13×(13+1) 2 當 nM2 時,
r ∵1+2+3+……+13= =91
2 an=Sn-Sn-1
∴所求為 1+(2+2)+ (3+3+3)+…… =2n+1(n2-2n)-2(n-1)+1〔(n-1)2-2(n-1)〕
+(13+13+……+13)+(14+14+……+14) =2n+1(n2-2n)-2n(n2-4n+3)
13 個 9個 =2n(2n2-4n-n2+4n-3)=2n(n2-3)
2 2 2 2 2
=1 +2 +3 +4 +……+13 +14×9 由1、2得 an=2n(n2-3),n 為正整數
13(13+1)
(2×13+1) i 圖 E1 有兩層共有 1+(1+2)=4(個焊接點)
= +14×9
6 圖 E2 有三層共有
=819+126 1+(1+2)+(1+2+3)=10(個焊接點)
=945 圖 E3 有四層共有
53 t 1 1×2+2×3+3×4+……+n(n+1) 1+(1+2)+(1+2+3)+(1+2+3+4)
=(12+1)+(22+2)+
(32+3)+…… =20(個焊接點)
(n2+n)
+ 以此類推,圖 E5 有六層共有
=(12+22+……+n2) +(1+2+……+n) 1+(1+2)+(1+2+3)+(1+2+3+4)
(n+1)
n (2n+1) n(n+1) +(1+2+3+4+5)+(1+2+3+4+5+6)
= +
6 2 =56(個焊接點)
(n+1)
n 〔 (2n+1)+3〕 o 113+123+……+203

6 =(13+23+……+203)-(13+23+……+103)
(n+1)
n (n+2) 20×(20+1) 2 10×(10+1) 2

3

( 2 )( -
2 )
故選D 20×21 2 10×11 2
2 共需 1+(1+2)+(1+2+3)+……

( 2 )(-
2 ) =44100-3025=41075

+(1+2+……+15) 故選A
1×2 2×3 3×4 15×16 55 p 設首項為 a1,公差 d=2
= + + +……+
2 2 2 2 25
則 S25= ×(a1+a25)
1 2
= (1×2+2×3+……+15×16)
2 25
= ×(a1+a1+24d)
1 15×16×17 2
由1知,所求為 × =680(顆)
2 3 =25(a1+12d)=25a13=25×64=1600
故共需要 680 顆柳丁 故共有 1600 個座位
互動式教學講義.數學(2) 解答篇 17
頁碼 頁碼
a(a1+2)+(a2+4)+……+(a10+20)=240
c(a1+a2+……+a10)+(2+4+……+20)=240
第2章
10
c a1+a2+……+a10=240- (2+20)
2
2-1 一維數據分析
=240-110=130
故選C 58 範例 1
s 設此等比數列首項為 a1,公比為 r 算術平均數
∵前 10 項的和為 80 1
m= (79+85+50+56+78+63+65+73+46+49)
c a1+a2+a3+……+a10=80 10
a(1-r 10
) 644
c
1
=80 ..................... 1 = =64.4(分)
1-r 10
59 類題1
又 a1+a3+a5+a7+a9=120
a1〔1-(r2) 5
〕 76×15-92-45-55 948
c =120 比賽成績為 = =79(分)
1-r 2 12 12
10 類題2
a(1-r )
=120 ................... 2
1
c 2 算術平均數
1-r
1 80 2 1 1
m= (5+12+10+13+5+8+12+15+9+10+10+11)
得 1+r= = c r=- 12
2 120 3 3
1 10 4 320 120
代入1得 a1 1- -
( )3
=80× =
3 3

類題3
12
=10(人)

1 320
()
10
c a1 1- = c a1~106 平均營業額
3 3
c 100Na1<110 1
m= (8+16+12+10+6+8+12+18+8+8+10+4)
故選D 12
56 d 設甲、乙兩人起薪 (第一個月) 均為 x 元 120
= =10(百萬元)
12
∴去年每個月的平均營業額是 1000 萬元
範例 2
85+82+73
平時成績為 =80(分)
3
∴學期成績 W 為
80×30 %+86×20 %+79×20 %+90×30 %
A ×:第 9 個月,甲9:x+200×2 =84(分)
60 類題1
B ×:第 13 個月
甲13:x+200×4,乙13:x+1000 設她期末考至少需考 x 分,則
C ○:第 19 個月 48×0.2+45×0.2+85×0.1+55×0.2+x×0.3M60
甲19:x+200×6,乙19:x+1000 c 38.1+0.3xM60
D ×:滿 18 個月 c 0.3xM21.9
甲共領 18x+200×3(1+2+3+4+5) c xM73
=18x+9000 ∴她期末考至少需考 73 分才可以及格
類題2
乙共領 18x+1000×6=18x+6000
E ○:甲25∼甲27 領 x+8×200 加權平均數 W 為
甲28∼甲30 領 x+9×200 6×70+6×50+5×x+4×85+4×50 1260+5x

甲31∼甲33 領 x+10×200 6+6+5+4+4 25
甲34∼甲36 領 x+11×200 1260+5x
∵60< <65
乙25∼乙36 領 x+2×1000 25
∴在第 3 年的 12 個月中,恰有 3 個月甲 ∴1500<1260+5x<1625
的月薪比乙的月薪高 c 240<5x<365 c 48<x<73
故選CE 故選BCD
18 互動式教學講義.數學(2) 解答篇
頁碼 頁碼
範例 3 D ○:∵70 分為第 60 百分位數
1 幾何平均數 G=al2×3×4×8×9×27
=6al2×3×22×23×32×33
6
∴至少有 40 %同學的成績大於或等於 70 分

=6al26×36 =6
E ○:由題圖可得選取成績較好的 20 %,則累積
相對次數需在 80 %以上,其對應的成績在

r=4al(1+44 %)
2 近四年的每年平均成長率為 90 分以上

=al1.44×1.5×0.8×1.2 -1
(1+50 %) (1-20 %)(1+20 %)-1 故選ACDE
4
63 類題2

144 150 80 120 1 由題圖可知,大約落在第 97 百分位數


=4zx × × × -1

al24×32×2×3×52×24×5×23×3×5
100 100 100 100 2 由題圖可知,2 歲的時候,女孩頭圍落在第 50
4
百分位數時,大約為 47 公分
= -1

al212×34×54
100 5 歲的時候,女孩頭圍落在第 50 百分位數時,
4
8×3×5 大約為 50 公分
= -1= -1
100 100 故這段期間頭圍約增加 50-47=3(公分)
=1.2-1=0.2=20 % 範例 5
故每年平均成長率為 20 % 1 由小而大:31,42,45,48,49,50,53,55,
61 類題 58,59,60,72,75,共 13 項
令平均成長率為 r,則 13+1
∴第 =7 項為中位數
(1+r)3 c 1.8=(1+r)3
18=10× 2
1
c 1+r=
(1.8)3 ~1.22 c r~0.22=22 % 故中位數為 53(分)
62 範例 4 2 由小而大:40,49,49,54,60,63,66,66,
將 350 個數據由小到大排序為 x1,x2,……,x350 69,79,共 10 項
25 10
1 因為 350× =87.5 不是整數 ∴第 =5 項及第 6 項的平均數為中位數
100 2
此時第 25 百分位數 P25=x88=6(小時) 1
故中位數為 (60+63)=61.5(公斤)
50 2
2 因為 350× =175 是整數 64 類題
100
x175+x176 1 若 kM79,則中位數為 79
所以第 50 百分位數 P50= 65+77+79+91+k
2 ∴ =79 c k=83
又 x175=7,x176=7 5
7+7 2 若 77<k<79,則中位數為 k
故 P50= =7(小時) 65+77+79+k+91
2 ∴ =k c k=78
70 5
3 因為 350× =245 是整數 3 若 kN77,則中位數為 77
100
x245+x246 65+77+79+k+91
所以第 70 百分位數 P70= ∴ =77 c k=73
2 5
又 x245=7,x246=8 故 k 之所有可能值為 83,78,73
7+8 65 範例 6
故 P70= =7.5(小時) 86+95+73+80+66
2 1 算術平均數 m= =80
75 5
4 因為 350× =262.5 不是整數 變異數 s 2
100
此時第 75 百分位數 P75=x263=8(小時) 1 2 2 2
= ×〔(x1-m) +(x2-m) +(x3-m)
類題1 5
2 2
A ○:∵累積相對次數為 50 %者為中位數 +(x4-m) +(x5-m) 〕
∴中位數為 60 分 1 2 2 2
= ×〔(86-80) +(95-80) +(73-80)
B ×:第 60 百分位數即為累積相對次數為 60 %者 5
2 2
∴P60=70 分 +(80-80) +(66-80) 〕
C ○:第 1 四分位數即為第 25 百分位數,所對應 1
= ×(36+225+49+0+196)=101.2
的成績落在 50 分∼60 分之間 5
故標準差 s =f101.2(~10.06)
互動式教學講義.數學(2) 解答篇 19
頁碼 頁碼
1 2 若標準差最小,此時資料要集中在 m 處
2 算術平均數 m= (1+2+3+4+5+7)
6 則此 8 筆資料應為 2,6,6,6,6,6,6,10
22 11 此時 m=6
= =
6 3 1
c s =zx ×〔(-4)2+02+02+02+02+02+02+42〕
1 8
變異數 s 2= × (x12+x22+x32+x42+x52+x62)-m2
6 =a4 =2
1 11 2 67 7
( )
範例
= ×(12+22+32+42+52+72)-
6 3 X-5
1 ∵Y=
1 121 70 35 -4
= ×104- = =
6 9 18 9 mX-5 12-5 7
s35
∴mY= c mY= =-
故標準差 s =zx
35 -4 -4 4
= (~1.97)
9 3 2 sY=s( X-5 )=s(- 1 X+ 5 )
-4 4 4
b:試自行比較兩種解法,歸納使用時機 1 1 3
類題1 | |
= - sX= ×3=
4 4 4
設第六科的成績為 x 分, 類題1
1 已知 mX=42,s X=7,mY=64,s Y=10
則 (68+80+80+80+86+x)=80 c x=86
6 ∵y=ax+b
∴標準差 s ∴s Y=|a|s X
1
=zx ×〔(x1-m) 2
+ 2
(x2-m)+……+ 2
(x6-m)〕 10
c 10=|a|×7 c a=± (負不合)
6 7
1
=zx ×〔(-12)2
+02+02+02+62+62〕 又 mY=amX+b c 64=42a+b
6 10 10
=s36 =6
(分) a= 代入得 64=42× +b c b=4
7 7
66 類題2
10
a 故數對(a﹐b)=
7 (
﹐4

類題2

b A ○:次序不影響標準差
B ○:將原資料加 11,不影響標準差
C ×:將原資料乘 2,則標準差變為 2 倍
c
D ○:將原資料減 7,不影響標準差
E ×:將原資料平方,則標準差會改變
故選ABD
d 68 範例 8
令原數據為 xi,將原數據 xi 乘上 100 再減去 170 得
新數據 y(即
i yi=100xi-170)為
e 5,8,9,9,19,6,9,11,-2,6
c平均數 my
1
由圖可知,a的資料分散在兩極端的地方 = 〔5+8+9+9+19+6+9+11+(-2)+6〕
10
c的資料最集中在 mc 處 80
∴a的標準差最大,而c的標準差最小 = =8
10
故選A 1
類題3
2
變異數 s y2= ×〔(5-8) +(8-8) 2
+(9-8)2

10
1 若標準差最大,此時資料分散在兩極端的地方 +(9-8) 2
+(19-8) 2 2
+(6-8)
則此 8 筆資料應為 2,2,2,2,10,10,10,10 +(9-8) 2
+(11-8) 2
+(-2-8)2

此時 m=6 +(6-8)〕2

1
c s =zx ×〔(-4)2+(-4)2+(-4)2+(-4)2+42+42+42+42〕 1
變異數 s y2= ×250=25
8 10
=s16 =4 c標準差 s y=s25 =5
20 互動式教學講義.數學(2) 解答篇
頁碼 頁碼
又 my=amx+b=100mx-170=8 D ○:∵調整後的分數不低於原始分數
178 x x
c mx= =1.78(公尺) ∴xNy= +45 c N45 c xN90
100 2 2
5 c沒有人的原始分數超過 90 分
s y=|a|s x=100s x=5 c s x= =0.05(公尺)
100 故選ABCD
故1 身高的平均數為 1.78 公尺 70 範例 10
2 身高的標準差為 0.05 公尺 10×56+10
1 m真= =57(分)
類題 10
設 x 是原始分數,y 是調整後的分數,y=a+bx 1
2 ∵4=zx ×(x12+x22+……+x92+602)-562
a+24b=50 2450 50 10
∴ c a= ,b=
a+97b=100 73 73 c x1 +x22+……+x92+602
2

2450 50 =10×(42+562)=31520
∴f(x)= + x c (64)
f ~77.4
73 73 c x12+x22+……+x92=27920
1
∴s 真=zx ×(27920+702)-572
調整後的平均分數變大了,同理,中位數也變大了
50 10
又由標準差的性質 s a+bx=|b|s x= sx
73 =s33(~5.74)
(分)
∴標準差變小,故選C 類題

69 範例 9 21×55-60
1 m新= =54.75
設原始成績 x1,x2,……,x50,算術平均數 M 21-1
調整後成績 10alx1 ,10alx2 ,……,10alx50 1
2 ∵3=zx ×(x12+x22+……+x212)-552
∵調整後的算術平均數為 65,標準差為 15 21
∴6.5 及 1.5 分別為 alx1 ,alx2 ,……,alx50 的算術 c x12+x22+……+x212
平均數及標準差 =21×(32+552)=63714
1 1
c 1.5=zx ×〔(alx1 )2+(alx2 )2+……+(alx50 )2〕-(6.5)2 ∴s 新=zx ×(63714-602)-(54.75)
2

50 20
1 =al3005.7-2997.5625
c 1.52= × (x1+x2+……+x50) -42.25
50 =al8.1375(~2.85)
c x1+x2+……+x50=50×(1.52+42.25) 71 範例 11
=50×44.5 組別 人數 平均成績 標準差
x1+x2+……+x50 50×44.5 第二類組 300 50 10
c M= = =44.5
50 50 第三類組 200 60 12
∴44NM<45,故選E 300×50+200×60
類題 1 ∵m 全= =54
300+200
設全班原始分數為 xi,平均為 mX,標準差為 s X; ∴合併後的平均成績為 54 分
調整後的分數為 yi,平均為 mY,標準差為 s Y 2 設二類組的學生為 x1,x2,……,x300,
xi 與 yi 的關係為 三類組的學生為 x301,x302,……,x500
xi mX sX 1
yi= +45 c mY= +45,且 s Y=
2 2 2 s(二)=10=zx ×(x12+x22+……+x3002)-502
300
A ○:已知調整後全班的平均為 70 分, c x12+x22+……+x3002=300(102+502)=780000
mX 1
則 70= +45 c mX=50<60
2 s(三)=12=zx ×(x3012+x3022+……+x5002)-602
200
B ○:已知調整後的標準差為 6 分, c x3012+x3022+……+x5002
sX =200(122+602)=748800
則 6= c s X=12
2 1
C ○:若調整後的分數為 60, ∴s 全=zx ×(x12+x22+……+x5002)-542
500
x 1
則 60= +45 c x=30
2 =zx
(780000+748800)-542
500
∵調整後仍有 5 人不及格 =al3057.6-2916 =al141.6(~11.9)
∴調整前有 5 人低於 30 分 故合併後的標準差為 al141.6(~11.9) (分)
互動式教學講義.數學(2) 解答篇 21
頁碼 頁碼
類題 73 類題

20×75+30×60 體重的標準化數據為-0.6,1,-0.8,-1.2,0,1.6
1 A= =66
20+30 乘以標準差 5 後得-3,5,-4,-6,0,8
2 設甲組學生為 x1,x2,……,x20, 再加上算術平均數 54 後得 51,59,50,48,54,62
乙組學生為 x21,x22,……,x50 故原來 6 位同學的體重分別為
1 51,59,50,48,54,62 公斤
52=s(甲)2= × (x12+x22+……+x202)-752
20 範例 13
c x12+x22+……+x202=113000 13-10
1 英文的標準化分數為 =1
(aB )2=s(乙)2 3
1 12-9
= × -602
(x212+x222+……+x502) 數學的標準化分數為 =1.5
30 2
c x212+x222+……+x502=108000+30B 2 ∵英文的分數比平均多 1 個標準差,
又 102=s 全2 數學的分數比平均多 1.5 個標準差
1 ∴相對於全班琳雅數學表現比較突出
= × 〔113000+(108000+30B)〕-662
50 類題

c B=60 1 期中考的算術平均數 m中=80,標準差 s 中=5


72 範例 12 期末考的算術平均數 m末=72,標準差 s 末=4
1 算術平均數 mx 84-80
期中考試成績的標準化分數為 =0.8
1 5
= (163+165+167+171+174)=168(公分)
5 77-72
期末考試成績的標準化分數為 =1.25
變異數 s x2 4
1 2 2 2 2 ∵期中考試成績比平均多 0.8 個標準差,
= 〔 (163-168) +(165-168) + (167-168)
5 期末考試成績比平均多 1.25 個標準差
2 2
+(171-168) +(174-168) 〕 ∴期末考的班級排名較佳
1 74 範例 14
= × (25+9+1+9+36)=16
5 甲得票數 0.4x+0.55y
標準差 s x=s16 =4(公分) 乙得票數 0.6x+0.45y
將各身高的數據減去算術平均數,再除以標準差 若甲當選,則甲>乙
即 xi 163 165 167 171 174 c 0.4x+0.55y>0.6x+0.45y
xi-mx c 0.2x<0.1y
xi'= -1.25 -0.75 -0.25 0.75 1.5 x 1
sx
c <
故得出身高的標準化數據為 y 2
-1.25,-0.75,-0.25,0.75,1.5 x 1
反之,若乙當選,則 >
2 算術平均數 my y 2
1 A ×:x+y 確定,但 x,y 比例未知
= (48+54+62+66+70)=60(公斤) ∴不可決定當選人
5
變異數 s y2 x 1
B ○: < c甲當選
1 y 2
2 2 2
= 〔 (48-60) +
(54-60) +
(62-60) x 1
5 C ○:x>y c >1> c乙當選
+(66-60) 2
+(70-60) 2
〕 y 2
1 x 55 1
= × (144+36+4+36+100)=64 D ○:0.4x>0.55y c > > c乙當選
5 y 40 2
標準差 s y=s64 =8(公斤) x 45 3
E ×:0.45y>0.6x c < =
將各體重的數據減去算術平均數,再除以標準差 y 60 4
即 yi 48 54 62 66 70 x 1
但無法確定 是否大於或小於
yi-my y 2
yi'= -1.5 -0.75 0.25 0.75 1.25 ∴不能決定當選人
sy
故得出體重的標準化數據為 故選BCD
-1.5,-0.75,0.25,0.75,1.25
22 互動式教學講義.數學(2) 解答篇
頁碼 頁碼
75 類題1 88
77 4 134495× =118355.6,可知 x=12
1 設最高族群所得平均為 61x, 100
最低族群所得平均為 x,其中 x>0 50
134495× =67247.5,可知 y=6
61x+5000 300000 100
A ×:甲案c =61- <61
x+5000 x+5000 故整數數對(x﹐y)=(12﹐6)
61x.(1+10 %) 5 1 算術平均數為
B ×:乙案c =61
x.(1+10 %) 10×1+25×2+20×3+20×4+10×5+15×6
61x 183000 10+25+20+20+10+15
C ○:丙案c =61- <61
x+3000 x+3000 340

D ○:∵標準差的改變只與伸縮有關 100
∴標準差不變 =3.4
E ×:承D 2 將擲 100 次所得的點數由小而大排列,
∴標準差增加 10 % 則第 50 次與第 51 次的算術平均數即為中位數
故選CD 3+3
∴中位數為 =3
2 設發放 k 元救濟金,則 2
61.22000 5+7+8+10+11+13
=50 6 平均數 m= =9
22000+k 6
c 61.22000=50.22000+50k 變異數 s 2
c 50k=11.22000 1 2 2 2
= 〔(x1-m) +(x2-m) +……+(x6-m) 〕
c k=4840 6
所以應發放 4840 元,可使貧富差距縮小至 50 倍 1 2 2 2 2
= 〔(5-9) +(7-9) +(8-9) +(10-9)
類題2 6
由題意 1988 至 2018 經過了 30 年 +(11-9)2+(13-9) 2

c 400(1+r)3=1638.4 1
= ×(16+4+1+1+4+16)=7
1638.4 4096 163 16 6
c 1+r=3zx =3zx =3zx 3 = =1.6
400 1000 10 10 標準差 s =a7 =aa
2018 至 2028 經過了 10 年 故 a=7
c 1638.4
(1+r) =1638.4×1.6=2621.44 78 7 設原來 mX=50,s X=10 且售價 y=1.5x
∴x=2621.44 則 mY=1.5mX=1.5×50=75
最接近的數值為 2600 s Y=1.5s X=1.5×10=15
故選B 故平均售價每個 75 元,標準差為 15 元
8 乙=甲+3,丙=2×甲,丁=2×甲-1
2-1 A ×:m乙=m甲+3
B ○:Me乙=Me丙=6
基礎題
30
76 1 算術平均數為此數列的第 26 項 C ○:5× =1.5,故 P30丙=4
100
a26=5+
(26-1)×3=80 30 4+4
2 設期末考考 x 分 10× =3,故 P30戊= =4
100 2
3.70+4.46+3.x D ○:丙=丁+1 c s 丙=s 丁
c M60
3+4+3 E ○:s 丁=s 丙,又 s 丙=s 戊,故 s 丁=s 戊
c 394+3xM600 故選BCDE
c 3xM206 12-10
2 9 1 英文的標準化分數為 =1
c xM68 2
3 11-7 4
故期末考要考 69 分 數學的標準化分數為 =
3 3
3 設平均成長率為 x 2 ∵英文的分數比平均多 1 個標準差

c x=al(1+20 %)
3

(1+x) = (1+40 %)
(1+20 %) (1+50 %) 4
3
(1+40 %)
(1+50 %)-1 數學的分數比平均多 個標準差
3
故選E ∴相對於全班,輊翔的數學科表現比較突出
互動式教學講義.數學(2) 解答篇 23
頁碼 頁碼
進階題 E ×:6×0.67=4.02
17×10+21+25 c P67戊 為由小到大的第 5 個數值
79 0 新平均數 m'= =18
12 即 P67戊=-6
1
又 zx (x12+x22+……+x102)
故選AC
-172 = 2
10 5×30+4×30+3×20+2×10+1×10
r 1 m=
c x12+x22+……+x102=2930 100
2930+212+252
故新標準差 s '=zx
360
-182 =
12 100
=a9 =3 =3.6(分)
q 設這 5 年人數為 a+20,a+10,a,a-10,a-20 1
2 s =zx (x12+x22+……+x1002) -m2
則平均人數 100
1 1 2
m= (a+20+a+10+a+a-10+a-20)=a =zx (5 ×30+42×30+32×20+22×10+12×10)-(3.6)2
5 100
變異數 s 2 1
=zx ×1460-(3.6) 2

1 2 2 2 100
= × 〔(a+20-a) +(a+10-a) +(a-a)
5 =d1.64(~1.28) (分)
+(a-10-a)2+ (a-20-a) 2
〕 81 t 設原數據為變量 X,中位數為 Me(X),
1 新數據為變量 Y,中位數為 Me(Y),
= × (202+102+02+102+202)
5 則 Y=100X-240
=200 3+6+1+5+4+8+6+7+5
∴mY= =5
標準差 s =d200 =10a2(~14.14) (人) 9
w 1 此群數值共 1
s Y2= 〔(-2) 2
+12+(-4) 2
+02+(-1) 2
+32
100×101 9
1+2+3+……+100= =5050(個)
2 +12+22+02〕
5050÷2=2525 =4
設此群數值由小到大的第 2525 項為 n,則 s Y=a4 =2,Me(Y)=5
1+2+3+……+(n-1)<2525N1+2+3+……+n Y+240
∵Y=100X-240 c X=
c(n-1)n<5050Nn(n+1) 100
∴n=71,亦即此數列的第 2525 項與第 2526 mY+240 5+240
故 mX= = =2.45
項均為 71 100 100
故中位數為 71 s X=s
( Y+240
100 )=s( Y

240
100 100 )
2 算術平均數 1 1
12+22+32+……+1002 = s Y= ×2=0.02
m= 100 100
1+2+3+……+100 Me(Y)+240 5+240
100×101×201 Me(X)= = =2.45
100 100
6 1 故選ABCE
= = ×201=67
100×101 3 y A ○:高於 25(mg/m3)有 12+4=16(天)
2 低於 20 天
80 e A ○:∵乙=甲+2 B ○:100×40 %=40
∴s甲=s乙 由題圖可知 P40 在 5∼15 之間
139 9 2
B ×:m丙=
6

()
2
=(m甲)2 C ○:夏季全距約為 30∼45
冬季全距約為 50∼65
C ○:∵Q1甲=3,Q1乙=5,Q1丙=9, D ○:由題圖可知夏季 PM2.5 之濃度分布較集中
Q1丁=a3 ,Q1戊=-12 故冬季的標準差較大
∴Q1丙 最大 E ○:高於 35(mg/m3)的天數有
D ×:∵Me甲=4.5,Me乙=6.5,Me丙=20.5,
a4 +a5
4+8+1+2=15(天)
Me丁= ,Me戊=-9 15
2 所占之百分比為 ×100 %<10 %
200
∴Me戊 最小 故選ABCDE
24 互動式教學講義.數學(2) 解答篇
頁碼 頁碼
歷屆試題 D ×:最低月所得平均
82 u A ○:40∼44 歲的失業率為 13.17 %為最大 0×30 %+10000×10 %
B ×:單由比率無法得知兩範圍之人數 +20000×30 %+40000×30 %
C ×:∵40∼44 歲和 45∼49 歲之勞動力人數 =19000(元)
不一定相等 ∴最低月所得平均小於中位數
∴失業率並不是平均數 E ○:假設社區月所得平均為 m,則
D ○:令 35∼39 歲之勞動力人數為 a, m×1000+200000
-m
40∼44 歲之勞動力人數為 b 1001
0.098a+0.1317b 200000-m
則 =0.1266 = <200
a+b 1001
c 0.0286a=0.0051b 故選ABE
∴a<b p 若全校老師與學生的人數分別為 A 人與 B 人
E ×:由題目表中顯示不出有如此的因果關係 則可推得全校師生玩過「寶可夢」的比率為
故選AD Ar1+Br2
r=
i A ○:令 x=9 代入得 A+B
9+1 A B
40 log

10
+60
) =
A+B
r1+
A+B
r2

=40×0+60=60(分) A B
可知只要知道全校老師與學生比率 ,
B ○:令 x=20 代入得 A+B A+B
20+1 即可判定全校師生玩過「寶可夢」的比率
40 log
(10
+60
) 故選A
=40×log 2.1+60 75+80+85+75+80
84 a A ×:m=
>40×0.3010+60>70(分) 5
C ×:∵分數不為線性調整 =79<80
∴兩次分數的全距無法比較 75+80+85+80+80
B ○:m=
D ○:∵大小順序不變 5
∴中位數不變 =80M80
1
s =zx (52+02+52+02+02)
E ×:∵每個分數都會改變
∴平均會改變 5
則小美的新成績不一定等於全班成績的 =s10 <5
平均 C ×:若自然與社會分別為 95 分與 75 分,則
故選ABD 75+80+85+95+75
m=
83 o 月所得 少於 10,000 10,000∼20,000 5
比例 30 % 10 % =82
1
月所得 20,000∼40,000 40,000∼80,000 s =zx 〔(-7)2+(-2)2+32+132+(-7)2〕
比例 30 % 30 % 5
A ○:中位數在 50 %, =s56 >5
所以介於 20,000 元及 40,000 元間 D ×:若自然與社會分別為 90 分與 80 分,則
B ○ 75+80+85+90+80
m=
C ×:假設月所得 5
月所得 9,999 19,999 =82
1
比例 30 % 10 % s =zx 〔(-7)2+(-2)2+32+82+(-2)2〕
月所得 39,999 79,999 5
比例 30 % 30 % =s26 >5
則月所得平均為 E ○:80NmN80.8
1
9999×30 %+19999×10 % s Nzx (5.82+0.82+4.22+1.22+1.22)
+39999×30 %+79999×30 % 5
=40999>40000 =al10.96 <5
故選BE
互動式教學講義.數學(2) 解答篇 25
頁碼 頁碼
x-180 50-100 1 x-180 1 g n=29
s = × c =-
180 100 2 180 4 P25:29×25 %=7.25,7+1=8
c x-180=-45 c x=135 ∴由高分數來第 29-8+1=22 名的成績
y-135 90-50 1 y-135 2 c不調,仍是 41 分c a=41
又 = × c =
135 50 2 135 5 P50:29×50 %=14.5,14+1=15
c y-135=54 c y=189 ∴由高分數來第 29-15+1=15 名的成績,調整
c 120<x<180<y,故選E 後為 60+5=65 分
85 d A ○ c b=65
B ×:50 歲至 64 歲之男性農業就業人口先增後 P75:29×75 %=21.75,21+1=22
減再增 ∴由高分數來第 29-22+1=8 名的成績,調整
C ○:男性農業就業人口皆小於 500 千人,且 後為 74+5=79 分
總就業人口皆大於 10000 千人 c c=79
500 P95:29×95 %=27.55,27+1=28
∴其所占比率皆小於 =5 %
10000 ∴由高分數來第 29-28+1=2 名的成績,調整
D ×:皆多於 49 歲以下之男性農業就業人口 後為 92+5=97 分
E ×:2018 年比 2011 年增加了 c d=97
79.4-69.1=10.3 千人>1 萬人 ∴數組(a﹐b﹐c﹐d)=(41﹐65﹐79﹐97)
故選AC 故選C
86 f A ○:x1N0.8x1+20 c 0.2x1N20 c x1N100
即原始分數 100 分以下,調整後的分數 2-2 二維數據分析
均不低於原始分數 88 範例 1
40 1 散布圖如下所示
B ○:0.8mx +20=60 c mx = =50
1
0.8 1

35 2
0.75mx +25=60 c mx = =46
2
0.75 2
3
∴mx >mx 1 2

16
C ×:0.8sx =16 c sx = =20
1
0.81
2 由散布圖可知,扣除(96﹐94)與(84﹐86)兩點
15 後,約略呈現負相關
0.75sx =15 c sx = =20
2
0.75 2
類題

∴sx =sx 1 2
1 散布圖如下所示
D ○:0.8x1+20>0.75x2+25
15 25
c x 1> x2+
16 4
15 25
c x1-x2> x2+ -x2
16 4
1 25 2 由散布圖可知為負相關
=- x2+
16 4 91 範例 2
1 1
= (100-x2) M0
16
(∵x2N100)
∴x1>x2
E ×:0.8x1+20<60 c x1<50
2
0.75x2+25<60 c x2<46
3
只可知甲班原始分數未滿 50 分的人數多 1
2 2 mx= ×(192+180+180+182+178+178+175
於乙班原始分數未滿 46 分的人數 9
3 +177+178)
故選ABD mx=180
26 互動式教學講義.數學(2) 解答篇
頁碼 頁碼
1 類題
my= ×(13+11+9+10+11+10+8+9+9)
9 A ○:相關係數 r 必滿足-1NrN1
=10 1
B ×:∵所有點都在直線 y= x-20
身高(公分)xi (yi-my)(xi-mx)2 (yi-my)2
鞋號 yi xi-mx yi-my (xi-mx) 2
192 13 12 3 36 144 9 ∴必為完全正相關,亦即 r=1
180 11 0 1 0 0 1 C ×:相關係數為 0,其散布圖可能為鉛垂線,水
180 9 0 -1 0 0 1 平線或上下左右均成對稱狀態
182 10 2 0 0 4 0
D ○:相關係數不受單位的影響
178 11 -2 1 -2 4 1
故選AD
178 10 -2 0 0 4 0
175 8 -5 -2 10 25 4 範例 5
177 9 -3 -1 3 9 1 1 m3x+5=3mx+5=3×8+5=29
178 9 -2 -1 2 4 1 2 s 3x+5=3s x=3×6=18
合計 49 194 18 3 r(3x+5﹐5y-2)=r(x﹐y)=0.7
49 49 49 95 類題1
d194 ×s18
r= = ~ ~0.83
6s97 6×9.85 1 設英文成績為 x,數學成績為 y
即相關係數約為 0.83 調整後英文成績為 x',數學成績為 y'
92 類題 得 mx=60,s x=5,my=50,s y=10,r(x﹐y)=0.6
x1+x2+……+x10 60 由題意知 y'=0.5×y+40
1 mx= = =6
10 10 c my'=m0.5y+40=0.5my+40=0.5×50+40=65
y1+y2+……+y10 70 s y'=s 0.5y+40=0.5s y=0.5×10=5
2 my= = =7
10 10 2 r(x'﹐y')=r(1.3x-8﹐0.5y+40)=r(x﹐y)=0.6
1
sx=zx ×〔(x1-mx)2+(x2-mx)2+……+(x10-mx)2〕
類題2
3
10 ∵z=2x-5,w=15-5y
1
=zx ×36 =al3.6
∴r(z﹐w)=r(2x-5﹐-5y+15)=-r(x﹐y)=-R,故選B
10 97 範例 6
1
sy=zx ×〔(y1-my)2+(y2-my)2+……+(y10-my)2〕
1+2+3+4 5 3+5+6+10
4 mx= = ,my= =6
10 4 2 4
1 (yi-my)(xi-mx)2
xi yi xi-mx yi-my (xi-mx)
=zx ×25 =al2.5
10 3 9 9
1 3 - -3
-18 -18 2 2 4
s36 ×s25
5 相關係數 r= = 1 1 1
30 2 5 - -1
=-0.6 2 2 4
1 1
範例 3 3 6 0 0
A:水平線c相關係數為 0 2 4
3 9
BC:直線的斜率為負c相關係數為-1 4 10 4 6
2 4
DE:直線的斜率為正c相關係數為 1
合計 11 5
故選DE
令 y 對 x 的最適直線
93 類題1
方程式為 y=a+bx
C:直線斜率為正c相關係數為 1
11
ABDE:為鉛直線或水平線或對稱圖形 ∴b=
5
c相關係數為 0
a=my-bmx
故選ABDE
11 5
類題2 =6- ×
5 2
∵A,B,C,E 四筆資料的關係約呈現直線相關
1
∴選擇去掉資料 D =
2
故選D
故 y 對 x 的最適直線
94 範例 4
方程式為
由題意知 y=1.5x-10 ∴r(x﹐y)=1
1 11
故選E y= + x
2 5
互動式教學講義.數學(2) 解答篇 27
頁碼 頁碼
類題1 2 將身高的單位改為公尺,得另一變量 x',則
∵x1+x2+……+x10=60 mx'=1.65 公尺
60 s x'=0.08 公尺
∴mx= =6
10 而相關係數 r 不變
∵y1+y2+……+y10=100 sy 4
∴b=r× =0.8× =40,
100 s x' 0.08
∴my= =10
10 a=my-bmx'=58-40×1.65=-8
由公式知,y 對 x 的最適直線方程式為 故體重 y 對身高 x' 的最適直線方程式為
36 y=-8+40x'
y-10= (x-6)c y=-2+2x
18 100 範例 8
98 類題2 1
∵mx=3
4+1+5+3+a
∴ =3 c a=2
5
又最適直線 23x-10y+11=0 必過(mx﹐my)
∴23×3-10my+11=0 c my=8
11+3+12+8+b
∴ =8 c b=6
5 1
2 mx= (163+165+167+171+174)=168
故數對 (a﹐b)=(2﹐6) 5
範例 7 1
my= (48+54+62+66+70)=60
1 設數學成績為 x,英文成績為 y 5
c mx=60,s x=12,my=69,s y=10,r(x﹐y)=0.6 1
s x=zx 〔(-5)2
+(-3)2
+(-1) 2
+32+62〕=4
又 y 對 x 最適直線的斜率 5
sy 10 1
m=r(x﹐y)× =0.6× =0.5 s y=zx 〔(-12) 2
+(-6) 2
+22+62+102〕=8
sx 12 5
又 y 對 x 的最適直線必過(mx﹐my) 標準化後的數據如下
∴y 對 x 的最適直線方程式為 xi-mx
xi'= -1.25 -0.75 -0.25 0.75 1.5
y-69=0.5(x-60)c y=39+0.5x sx
2 今數學成績為 66 yi-my
yi'= -1.5 -0.75 0.25 0.75 1.25
∴令 x=66 代入得 y=39+0.5×66=72 sy
故此生英文成績預測是 72 分 故得散布圖如下
99 類題1
1 y 對 x 的最適直線方程式為
sy 3
y=my+r× (x-mx)
=42+0.6× (x-60)
sx 4
=15+0.45x
2 x 對 y 的最適直線方程式為
3 直接計算得
sx
x=mx+r× (y-my) 1
sy r= ×〔(-1.25)×(-1.5)
5
4
=60+0.6× (y-42) +(-0.75)×(-0.75)+(-0.25)×0.25
3
+0.75×0.75+1.5×1.25〕
=26.4+0.8y
1
b:y 對 x 的最適直線與 x 對 y 的最適直線不同 r= ×(1.875+0.5625-0.0625+0.5625+1.875)
5
類題2
1
sy 4 = ×4.8125
1 b=r× =0.8× =0.4, 5
sx 8
=0.9625
a=my-bmx=58-0.4×165=-8
即相關係數為 0.9625
故體重 y 對身高 x 的最適直線方程式為
4 y' 對 x' 的最適直線方程式為 y'=0.9625x'
y=-8+0.4x
28 互動式教學講義.數學(2) 解答篇
頁碼 頁碼
101 類題 1
2 mx= (60+70+80+90+100)=80
1 5
1
my= (14+16+19+20+21)=18
5
xi (yi-my)(xi-mx)2 (yi-my)2
yi xi-mx yi-my (xi-mx)
60 14 -20 -4 80 400 16
70 16 -10 -2 20 100 4
80 19 0 1 0 0 1
90 20 10 2 20 100 4
1 10021 20 3 60 400 9
2 mx= (0+1+2+3+4+5+6)=3 合計 180 1000 34
7
180
f1000 ×s34
1 r=
my= (87+83+79+75+71+67+63)=75
7
180
1
s x=zx 〔(-3)2+(-2)2+(-1)2+02+12+22+32〕

20s85
7
180
=2 ~
20×9.22
1
s y=zx 〔122+82+42+02+(-4)2+(-8)2+(-12)2〕 ~0.98
7
3 設最適直線方程式為 y=a+bx
=8
180 9
標準化後的數據如下 則 b= =
1000 50
xi-mx
xi'= -1.5 -1 -0.5 0 0.5 1 1.5 9 18
sx a=my-bmx=18- ×80=
50 5
yi-my
yi'= 1.5 1 0.5 0 -0.5 -1 -1.5 18 9
sy 故最適直線方程式為 y= + x
5 50
故得標準化數據的散布圖如下
4 當 x=85 時,
18 9
y= + ×85=18.9(公里/公升)
5 50
103 類題

168+169+170+171+172
1 mx= =170,
5
3 直接計算得 59+63+64+66+68
my= =64
1 5
r= ×〔(-1.5)×1.5+(-1)×1
7 xi (yi-my) (xi-mx)2
yi xi-mx yi-my (xi-mx)
+(-0.5)×0.5+0×0+0.5×(-0.5) 168 59 -2 -5 10 4
+1×(-1)+1.5×(-1.5)〕 169 63 -1 -1 1 1
1 170 64 0 0 0 0
r= ×(-7)=-1
7 171 66 1 2 2 1
即相關係數為-1 172 68 2 4 8 4
4 y' 對 x' 的最適直線方程式為 y'=-x' 合計 21 10
102 範例 9 21
b= ,
1 10
21
a=my-bmx=64- ×170=-293
10
21
故 y 對 x 的最適直線方程式為 y=-293+ x
10
2 x=180 代入1得
21
y=-293+ ×180=85(公斤)
10
互動式教學講義.數學(2) 解答篇 29
頁碼 頁碼
3 若身高改為公尺,
則 mx'=1.7, 2-2
0.21
b= =210, 基礎題
0.0010
106 1 ∵y 值不受 x 影響
a=my-bmx'=64-210×1.7=-293
且其散布圖為水平線
故 y 對 x' 的最適直線方程式為
故相關係數為 0
y=-293+210x'
2 ∵15 筆資料皆在直線
104 範例 10
y=4-3x 上
A ×:∵標準化後迴歸直線斜率為相關係數
且斜率為負
∴Y' 對 X' 的迴歸直線斜率為 0.0314
∴相關係數為-1
B ×:y=4×35-20=120>100,超過上限
3 設甲、乙、丙、丁評的分數分別為 ai,bi,ci,di
9
C ○:∵ ×1>0 1(ai﹐bi)皆落在直線 x-y=0 上
5
且直線 x-y=0 的斜率為正
∴相關係數不變
∴甲與乙評分的相關係數為 1
D ○:斜率為正的線性變換不影響數據相對位置
2(ai﹐ci)皆落在直線 x-y=1
E ×:∵相關係數接近 0
且直線 x-y=1 的斜率為正
∴不適用
∴甲、丙評分的相關係數為 1
故選CD
3(ai﹐di)皆落在直線 x=2y 上
105 類題1
且直線 x=2y 的斜率為正
5×18+18
1 A ×:mx'= ∴甲、丁評分的相關係數為 1
6
由1、2、3可知 a=b=g,故選B
=18
107 4 ∵r(x﹐y)=0.8
5×40+148
B ○:my'= =58 ∴r(3x-7﹐-4y+9)=-r(x﹐y)=-0.8
6
5 由題意明顯可知,
>40
5 筆資料(xi﹐yi)皆在 x+y=12 直線上
s x2×5+0
C ×:s x'=zx ∴y 對 x 的最適直線為 y=12-x
6
sy 3 2
<s x 6 ∵r× =1.2 c r× =1.2 c r=1.2× =0.8
sx 2 3
s y2×5+(148-58) 2

D ○:s y'=zx 2+3+5+5+10


6 7 mx= =5,
5
>s y
1+8+2+4+5
(∵y1,y2,y3,y4,y5 介於 20∼60 之間 my= =4
5
∴s yN20)
∴去掉 D(5﹐4)後相關係數不變,故選D
故選BD
5
s y' s x' 108 8 迴歸直線方程式為 y-62=0.8× (x-170)
2 bnew=r'× c r'=bnew× , 10
s x' s y'
整理得 y=0.4x-6
sy sx
同理 b=r× c r=b× 9 C ×:y 對 x 的最適直線必過(mx﹐my)
sx sy
故選C
又 s x'<s x,s y'>s y,bnew=b c r>r'
sy
類題2 0 由 y=my+r (x-mx)
sx
A ○:my=2×25+15=65
sy
s y=2s x=18 且 mx=5,my=3,r=1 c y=3+ (x-5)
sx
B ×:s z 不一定為 10als x =30
sy
C ×:∵原成績與方案甲、方案丙皆成正比 ∵過(0﹐2) ∴2=3+ (-5)c 5s y=s x
sx
∴相關係數皆為 1
sy
D ○:方案甲與方案丙的相關係數為 1,故原成績 ∴最適直線為 y=3+ (x-5)
5s y
與方案乙的相關係數必不大於方案甲與方案
1 1
丙的相關係數 即 y=3+ (x-5) ∴y=2+ x
5 5
故選AD
30 互動式教學講義.數學(2) 解答篇
頁碼 頁碼
A ×:最適直線不過(0﹐0) 1
E ×:y 對 x 的迴歸直線為 y-5=- (x-6)
1 2
B ○:y 對 x 的最適直線為 y=2+ x
5 x=p+3
p=x-3
C ×:最適直線不過 (10﹐0) 又 c 1
q=-2y+1 y=- (q-1)
D ○:∵s x=5s y ∴s xMs y 2
1 1 1
E ×:y 對 x 的最適直線的斜率為 c- (q-1)-5=- (p+3-6)
5 2 2
故選BD c q-1+10=p-3
進階題 c q=p-12
109 q 由圖知 E 同學轉去第二類組會使兩類組 x,y 相 故選AC
關係數都提高,故選E 1+2+3+2 a+5+4+b
112 t mx= =2,my=4=
w A ○:由題圖可知為正相關 4 4
B ○:由氣溫低至高數點,第 10、11 個點皆在 c a+b=7 c b=7-a……………(*)
20∼25 之間 (yi-my)(xi-mx)2 (yi-my)2
xi yi xi-mx yi-my (xi-mx)
2
C ×:冰淇淋全距不到 30,標準差必小於 30 1 a -1 a-4 4-a 1 (a-4)
D ×:由題圖可知為正相關且必大於 0.01 2 5 0 1 0 0 1
3 4 1 0 0 1 0
E ○:由正相關可知斜率為正 2
2 7-a 0 3-a 0 0 (3-a)
故選ABE 2
合計 4-a 2 2a -14a+26
x x

a2 ×al2a2-14a+26
110 e A ○:xM30 c M10 c +50M60 1 4-a
3 3 又 r= =
2
1 2
B ○:s y= s x c s x=3s y=3×5=15(分) 1 (4-a)
3 平方得 =
4 2(2a2-14a+26)
C ×:原始分數 54 分,
c a -8a+16=a2-7a+13 c a=3
2

54
則調整後分數為 y= +50=68 分 代入(*)得 b=4
3
故數對(a﹐b)=(3﹐4)
∴刪去一筆調整後 68 分的數據後,
y ∵最適直線必過(mx﹐my)
剩下的人調整後的平均為 68 分
若 y 對 x 的最適直線為 y=22+0.6x
x 2
D ○: +50Mx c xN50 c xN75 將(a﹐b)代入得 b=22+0.6a ...................1
3 3
若 y 對 x 的最適直線為 y=24+0.8x
yi 68
E ○:∵zi= - ∴相關係數為 1 將(b﹐a)代入得 a=24+0.8b ...................2
5 5
由1、2得 a=80,b=70
故選ABDE
故數對(a﹐b)=(80﹐70)
111 r ∵mx=6,my=5
113 u 1 由題圖知 x,y 之對應關係如下:
由迴歸直線方程式得 y-5=m (x-6)
x 3 5 7 7 8
sy sy
其中 m=r× =-0.8× y 6 5 6 8 10
sx sx
1
A ○:迴歸直線過
(2﹐7) 可得 mx= (3+5+7+7+8)=6,
5
1
代入可得 7-5=m(2-6)c m=- 1
2 my= (6+5+6+8+10)=7
5
1 4 sy sy 5
B ×:m=- =- × c = <1 xi yi xi-mx (yi-my)(xi-mx)2 (yi-my)2
yi-my (xi-mx)
2 5 sx sx 8
3 6 -3 -1 3 9 1
c sy<sx 5 5 -1 -2 2 1 4
C ○:∵p=x-3,q=-2y+1 7 6 1 -1 -1 1 1
∴sp=sx,sq=2sy 7 8 1 1 1 1 1
sq 2sy 10 8 10 2 3 6 4 9
c = = >1 c sq>sp
sp sx 8 合計 11 16 16
p=x-3 1 16
D ×:∵ ,且 1×(-2)<0 s x=zx (9+1+1+1+4)=zx
q=-2y+1 5 5
∴rpq=-rxy=0.8 =al3.2(~1.79)
互動式教學講義.數學(2) 解答篇 31
頁碼 頁碼
11 115 a A ×:第十公里平均心率為 188,無法得知最高
s16 ×s16
2 相關係數 r=
心率為多少
11 B ○:總步數為
= =0.6875
16 990+1000+1005+995+1015+1005
3 設最適直線方程式為 y=a+bx, +1050+1050+1050+1100=10260
11 10000
則 b= , ∴ <1(公尺)
16 10260
11 23 C ×:由數據可看出,完成時間愈少,平均心
a=my-bmx=7- ×6=
16 8 率愈大,兩者應為負相關
23 11 D ○:由數據可看出,步數愈多,平均心率愈
∴最適直線方程式為 y= + x
8 16 大,兩者應為正相關
i A ○:由散布圖可知 E ○:由數據可看出,完成時間愈少,步數愈
數學成績與英文成績呈現正相關 多,兩者應為負相關
所以相關係數大於 0 故選BDE
B ○:由散布圖可知 116 s 年份 男增加 女增加 人數差距 總人數
英文成績的中位數為 70 2009 1460 5360
數學成績的中位數約為 65 2010 10 50 1420 5420
2011 120 240 1300 5780
C ×:由散布圖可知 2012 170 130 1340 6080
數學成績的全距約為 50 2013 120 280 1180 6480
所以標準差不可能大於 30 2014 90 130 1140 6700
D ○:過點(80﹐80)作斜率為-1 的直線 2015 70 80 1130 6850
合計 580 910 42670
由散布圖可知
A ×:男增加人數小於女增加人數
該直線的右半平面有 4 個點,
B ×:總共增加 580 名
所以有 4 位同學兩科總分大於 160 分
C ×:2012 年差距增加
E ○:平移不改變其相關係數
D ○:男生增加的幅度較小 ∴m男<m女
故選ABDE
E ○:42670÷7~6096,超過 6000 名
歷屆試題
故選DE
114 o A ○:300 位學生的 (X﹐Y)可用散布圖表示
117 d 原坐標約
B ×:∵r=0.016,相關程度不高
x -13 -9 -8 -3 1 3
∴不適合 y 5 4 6 9 9 12
C ○:r(X+5﹐Y+5)=r(X﹐Y) x 7 10 15 17 19 20
=0.016 y 18 21 22 24 27 27
D ○:r(10X﹐10Y)=r(X﹐Y) 新坐標
x' 18 13 14 12 8 9
=0.016
y' 5 4 6 9 9 12
E ○:r(X'﹐Y')=r( X-m
s
X Y-mY
﹐ s ) x' 11 11 7 7 8 7
X Y

=r( sX - sm Y mY y' 18 21 22 24 27 27
X
X

X
﹐s - s
Y Y

=r(X﹐Y) 散布圖略圖如右
=0.016 由圖形可知,最高溫與
故選ACDE 溫差為負相關
sy 且它們的相關性比最高
p ∵迴歸直線方程式為 y-my=r(x﹐y) (x-mx) 溫與最低溫的相關性弱
sx
其中 5 組數據中的 mx 與 my 均相同,且 5 組數據 故選D
中的 s x 亦相同,又 5 組數據以A選項 y 的標準 118 f ∵相關係數 r=-0.99,接近-1
差 s y 最大,E選項 y 的標準差 s y 最小 ∴數據幾乎共線,即平均氣溫與平均售出量可看
sy 成線性關係
由題意知,迴歸直線相同c r(x﹐y) 相等 設平均氣溫為 8nC 時賣出 y 杯
sx
由題意知,各資料為負相關c r(x﹐y)<0 437-512 512-y
∵斜率相等,則 =
綜合以上可知,E選項中的相關係數為最小 13-11 11-8
故選E c y=624.5,故選B
32 互動式教學講義.數學(2) 解答篇
頁碼 頁碼
g mX=94,mY=97
(yi-mY)(xi-mX)2 (yi-mY)2
腦容量(xi)IQ(yi)xi-mX yi-mY(xi-mX) BBB
第3章
90 90 -4 -7 28 16 49
95 100 1 3 3 1 9 3-1 計數原理
91 112 -3 15 -45 9 225
88 80 -6 -17 102 36 289 123 範例 1
106 103 12 6 72 144 36 1 1 5 不是自然數
合計 160 206 608 2 本班至少有 11 人數學及格
160 3 本班同學中至少有 1 人會吸菸
d206 ×d608
rX,Y= 4 a,b 至少有一個不是自然數
20 2 “1Nx<3”表示“xM1 且 x<3”,
f1957
= ~0.45 其否定敘述為“x<1 或 xM3”
類題1
故選D
119 h 由題圖可知C為正相關,r>0 1 班上的同學都是女生
2 小明第一次段考至少有一科不及格
且D為完全正相關,即 r=1 為最大
3 本班全部的同學都不會翹課
故選D
120 j 由題圖可知, 4 本班這次段考至多有 9 人數學及格或至少有 11
人數學及格
這組二維數據分布在 y=2x 附近
124 類題2
1
又 y=- x 和 y=2x 垂直 1 2+1≠3 且 7-5=6
2
且過點 (0﹐0) 2 2+1≠3 或 7-5=6
3 3N1 且 5+2=3
4 3N1 或 5+2=3
範例 2

1 a+b>0 且 ab>0 —
←—
—→
— a>0 且 b>0

∴為充要條件
2 2 2 ○
2 AB +AC =BC —←—
—→
— △ABC 是直角△
×
(△ABC 為直角三角形時,∠A 不一定是直角)
∴為充分條件
1 ×
若將二維數據投影至 y=- x上 3 AC⊥BD —
←—
—→
— ABCD 為菱形
2 ○
如上圖所示 (如右圖,四邊形 ABCD 的對角線
這些新的數據會聚集在 (0﹐0)附近 AC⊥BD,但此圖形不是菱形)
此時所得到的變異數會是最小 ∴為必要條件
故選E 類題1

k 取圖上兩點大約為 ○
1 a=b=0 —
←—
—→
— a+b=0
1 1 ×
( -1﹐- , 1﹐
2 )(2 ) (a+b=0 時,可令 a=2,b=-2,
1 則不滿足 a=b=0)
可得斜率 m= ∴為充分條件
2
1 ×
c L:y= x ——
2 abc=0 ←——
→ a=0
2 ○
1 (abc=0 時,可令 a=5,b=3,c=0,
將 x=s,y=log t 代入 L 得 log t= s 則不滿足 a=0)
2
s
∴為必要條件
c t=10 2 c t2=10s

故選D ——
3 x=1 或 x=2 ←—→ (x-2)=0
— (x-1)

∴為充要條件
互動式教學講義.數學(2) 解答篇 33
頁碼 頁碼
125 類題2 類題1

× 1 A-B={1﹐2}
1 ∠A>∠B —←—
—→
— ∠A>60n
× 2 A={1﹐2﹐3﹐4},B'={1﹐2﹐7}
(若∠A>∠B,令∠A=50n,∠B=40n, ∴A∩B'={1﹐2}
∠C=90n,則不滿足∠A>60n 3 由1、2可知
又∠A>60n, A-B=A∩B' 成立
令∠A=70n,∠B=80n,∠C=30n, 127 類題2

則不滿足∠A>∠B) 作圖如下,得
∴為非充分亦非必要條件

——
2 ∠B>90n ←—→ △ABC 是鈍角三角形

×
(若△ABC 是鈍角三角形,令∠A=120n, 1{x|-5NxN5,xlR}
∠B=30n,∠C=30n,則不滿足∠B>90n) 2{x|-2NxN4,xlR}
∴為充分條件 3{x|-5Nx<-2,xlR}
範例 3 4{x|4<xN5,xlR}
1 1{1﹐3﹐5﹐15} 5{x|x<-2 或 x>4,xlR}
2{-3﹐-2﹐-1﹐0﹐1﹐2} 128 範例 5
2 ∵A={1﹐2﹐3} 利用樹狀圖分析如下
∴A 的子集合有 ∅,{1} ,{2},{3},{1﹐2},
{1﹐3},{2﹐3} ,{1﹐2﹐3},共 8 個
b:集合 A 若有 n 個元素,則其子集合共有 2n 個
類題1

ACD
126 類題2

1 不含元素的—∅,共 1 個
2 含有 1 個元素的— {a},{b},
{c}, ,共 4 個
{d } ∴共有 10 種走法
3 含有 2 個元素的— {a﹐b}, {a﹐c},{a﹐d }, 129 類題1

{b﹐c},{b﹐d }, ,共 6 個
{c﹐d } y 0 1
4 含有 3 個元素的— {a﹐b﹐c}, {a﹐b﹐d }, x 0,1,2,3,4,5 0,1,2
{b﹐c﹐d },共 4 個
{a﹐c﹐d },
由上表可知
5 含有 4 個元素的— {a﹐b﹐c﹐d },共 1 個
滿足 x+3yN5 的數對(x﹐y)共有 6+3=9(種)
∴共有 1+4+6+4+1=16(個) 類題2
範例 4
作樹狀圖如下
1 1 ∵A∩B=
{3﹐4}
∴(A∩B) '=
{1﹐2﹐5﹐6﹐7}
2 A'={5﹐6﹐7},B'=
{1﹐2﹐7}
∴A'∪B'=
{1﹐2﹐5﹐6﹐7}
3 由1、2可知 (A∩B)'=A'∪B' 成立
2 作圖如下,得

由上圖可知共有 10 種情形
範例 6
1 S∪T={x|-1<xN7,xlR} ∵骰子點數有 1,2,3,4,5,6,共 6 種情形
2 S∩T= {x|2NxN5,xlR} ∴點數和為 2,3,4,……,12,共 11 種情形
3 S-T={x|5<xN7,xlR} 這些點數和為 4 的倍數者有 4,8,12,共 3 種情形
4 T-S= {x|-1<x<2,xlR} 點數和 4 8 12
5 S∪T'={x|xN-1 或 xM2,xlR} 阿珠 1 2 3 2 3 4 5 6 6
6 S'∩T'=
{x|x>7 或 xN-1,xlR} 阿花 3 2 1 6 5 4 3 2 6
由以上討論可得共有 3+5+1=9 種情形
34 互動式教學講義.數學(2) 解答篇
頁碼 頁碼
130 類題1 類題2

1 若買丙,可送甲或乙共 2 種選擇; 4 人比賽共需 6 場(甲乙、甲丙、甲丁、乙丙、乙


同理,買丁或戊亦有 2 種選擇 丁、丙丁)
2 若買己,可送甲或乙或丙或丁或戊共 5 種選擇; ∴4 人的總勝場數為 6 場
同理,買庚或辛亦有 5 種選擇 1 若甲、乙、丙三人皆勝 2 場
由1、2可得共有 2+2+2+5+5+5=21 種選擇 則三人勝場數為 6,此時丁勝 0 場
〔另解〕 2 若甲、乙、丙三人皆只勝 1 場
買的款式 甲、乙 丙、丁、戊 己、庚、辛 則丁勝 6-3=3 場,但甲勝丁
送的款式 0 甲、乙 甲、乙、丙、丁、戊 ∴不合
搭配種數 0 3×2=6 3×5=15 故得丁的勝場為 0 場
由上表可得共有 6+15=21 種選擇 133 範例 9
類題2 1 依 A → B → C → D → E 的順 A B C D E
與甲同行的號碼為 序塗之
2,6,10,14,18,22,26,30,34,38,46,50 得 3×2×2×2×2=48(種)
又乙、丙的號碼和為 90-42=48 2 由相鄰區域最多者開始:
且 48=46+2=38+10=34+14=30+18=26+22 C→D→A→B→E
而乙、丙可以互換座位 ∴共有 5×2=10(種) 得 5×4×3×4×4=960(種)
131 範例 7 類題

1 1 由 C → D → E → F → A → B 順序塗之得
7×6×5×4×6×6=30240(種)
2 由 A → B → C → D → F → E 順序塗之得
7×6×5×5×5×5=26250(種)
共有 3×3×2=18(種) 134 範例 10
2 300
1∼300 中 5 的倍數有 =60 個
5
300
而 =37.……
8
如上圖,展開式共有 3×2×4=24(項)
類題1 ∴8 的倍數有 37 個
300
4×2×2=16
(種) 又 =7.5
40
∴40 的倍數有 7 個
1 n(A)=60
2 n(B')=300-n(B)=300-37=263
類題2

1 依序選擇 a,選擇 b,選擇 c


則由乘法原理知共有 4×4×4=64(種)
2 2×4×4=32(種)
132 範例 8 3 (n A∩B)=7
1 若穿裙子則有 5×6×4=120(種) 4 n(A∪B)=n(A)+n(B)-n(A∩B)
2 若穿長褲則有 4×6×3=72(種) =60+37-7=90
由1、2可得 120+72=192(種) n A∩B')=n(A)-n(A∩B)=60-7=53
5 (
類題1 6 n((A∪B)')=300-n(A∪B)
1 當 b=2 時,a=1,c=0,1,有 1×2 個 =300-90=210
2 當 b=3 時,a=1,2,c=0,1,2,有 2×3 個
……

8 當 b=9 時,a=1∼8,c=0∼8,有 8×9 個


∴共有 1×2+2×3+……+8×9=240(個)
故選A
互動式教學講義.數學(2) 解答篇 35
頁碼 頁碼
類題1
範例 12
設 A 表示解出第一題者所成的集合 先考慮第三天
B 表示解出第二題者所成的集合 1 若第三天訪查的店為 A
則n
(A)
=32, 一 二 三 四 五
(B)
n =24, A A A
(A∩B)
n =15 1 × 5 × 1 × 5 × 1 =25(種)
作圖填空格,依序填入 2 若第三天訪查的店家不是 A
(A∩B)
n (A-B)
,n , 一 二 三 四 五
(B-A)
n 可得右圖 A 非A A
故兩題都解不出者有 1 × 4 × 5 × 4 × 1 =80(種)
50-(17+15+9)=9(人) 由1、2可得所求為 25+80=105(種)
類題2
故選B
∵學生 100 人,而愛好音樂者有 53 人,愛好體育者 136 類題1
有 72 人 1 ∵每車只能載 4 人
∴53+72-xN100 c xM25 ∴ 甲 A 與 乙 B 必不同車
但 xN53 且 xN72 c xN53 (若同車則造成 C,D 亦同車)
即 M=53,m=25 則有以下情形:
c M-m=28
135 範例 11 1 — 2 —
設 A2、A3、A5 分別表 2 的倍
數、3 的倍數、5 的倍數所成 3 — 4 —
的集合 故共有 4 種乘車方式
則n(A2) =500, 2 A ×:情形3有 A,D 同車
(A3)
n =333, B ○: 甲 A 與 乙 B 必不同車
(A5)
n =200 C ×:情形1,2,乙丙不同車
(A2∩A3)
n =166, D ×:情形1,2,丙 B 不同車
(A3∩A5)
n =66, E ○:情形1,4皆符合
(A5∩A2)
n =100, 故選BE
(A2∩A3∩A5)
n =33 類題2
∴n
(A2∪A3∪A5) 全部的點餐方式共有 5×2×3=30(種)
=n (A2)+n(A3)
+n(A5)-n(A2∩A3)
-n(A3∩A5) 超過 1050 大卡的點餐方式有
(A5∩A2)
-n +n(A2∩A3∩A5) (雞腿堡﹐奶昔﹐奶茶)、(雞腿堡﹐奶昔﹐可樂)、
=500+333+200-166-66-100+33 (雞腿堡﹐中薯﹐奶茶)、(牛肉堡﹐奶昔﹐奶茶)、
=734(個) (牛肉堡﹐奶昔﹐可樂)
類題
共5種
設 Ak 表示 k 的倍數所成的集合 故小華的點餐方式共有 30-5=25(種)
則n(A6)=166
(A10)
n =100 3-1
(A8)
n =125
(A6∩A10)
n =n(A30)=33 基礎題
(A10∩A8)
n =n(A40)=25 137 1 1NxN3 表示 xM1 且 xN3,
(A8∩A6)
n =n(A24)=41 其否定敘述為 x<1 或 x>3
(A6∩A10∩A8)
n =n(A120)=8 故選D
∴所求為 n (A6∪A10∪A8) -n(A8) 2 1 x2+x=0
=n (A6)+n(A10)+n(A8) c( x x+1)=0
(A6∩A10)
-n -n(A10∩A8)
-n(A8∩A6) c x=0 或 x=-1
+n(A6∩A10∩A8)-n(A8) ×
x2+x=0 —
←—
—→
— x=0
=166+100+125-33-25-41+8-125 ○
=175(個) ∴為必要條件
36 互動式教學講義.數學(2) 解答篇
頁碼 頁碼
2 圓內接平行四邊形必是矩形 進階題
而矩形可以中心為圓心作外接圓 w 上層磁磚可排
∴為充要條件 “紅—紅—紅”或“紅—綠”
3 ∵BfA 或“綠—紅”共 3 種方式,
∴5lA c x=5 下層磁磚同樣的也可排 3 種方式,
得 A={2﹐4﹐5},B= {2﹐5﹐y} 共有 3×3=9 種排列方式
由 BfA 得 y=2,4 或 5 e A ○:小君符合條件2
故數對 (x﹐y)=(5﹐2), (5﹐4)或(5﹐5) B ×:密切接觸者也可能是老師
4 C ×:也有可能是連續 10 個停課 1 日所造成
D ×:若該校班級數為 15 班以下,則符合三分
之一以上停課,得實施全校停課
故 A'∪B'=
{x|xlR 且 xN1 或 x>4}
E ×:若三人同班,則只需 1 個班級停課 1 日
5 設洋基隊以 Y 表示,太空人隊以 H 表示
即可
由樹狀圖得知共有 4 種可能情形
故選A
140 r 三位數共有 9×10×10=900(個)
而不滿足「cMa 或 cMb」者即為「c<a 且 c<b」
138 6 1 1∼9 共有 9×1=9(個) 若 c=0,則(a﹐b)有 92 種
2 10∼99 共有 90×2=180(個) 若 c=1,則(a﹐b)有 82 種
3 100 共有 1×3=3(個)
∴共需寫 9+180+3=192 個字 ……
故選D 若 c=8,則(a﹐b)有 12 種
7 阿雅選洋裝、外套有 5×4=20(種) 故滿足所求之序組(a﹐b﹐c)有
阿寶選洋裝、外套有 4×3=12(種) 900-(12+22+……+92)
9×10×19
由乘法原理得 20×12=240(種) =900-
6
8 1 y=0 時,x=0,1,2,3,4,5,共 6 種
=900-285
2 y=1 時,x=0,1,2,共 3 種
=615(個)
由1、2可得 6+3=9(種)
t 在(A﹐B﹐C﹐D)分店數為(3﹐3﹐2﹐2)的基
9 由 C → A → D → B 的順序塗之,
礎下設立 14 家分店,且每個城市至少有 3 家分
有 5×4×3×3=180(種)
店,則分店數有兩類:
0 設 A 表數學及格者之集合 ∴n (A)=50-30=20
1(5﹐3﹐3﹐3):
設 B 表物理及格者之集合 (B)
∴n =50-23=27
快遞路線有
5×(3+3+3)+3×(3+3)+3×3
=45+18+9
=72
2(4﹐4﹐3﹐3):
將集合個數填入上圖之文氏圖中,得 快遞路線有
1 兩科均不及格者有 4×(4+3+3)+4×(3+3)+3×3
50- (8+12+15)=15(人) =40+24+9
2 數學及格而物理不及格有 8 人 =73
139 q 設 Ak 表示 k 的倍數所成的 ∴最少 a=72,最多 b=73
集合 故數對(a﹐b)=(72﹐73)
歷屆試題
由文氏圖可得,所求為
y ∵國文成績已不符合標準
(A2∪A3∪A5)
n -n(A2)
∴若他有資格參選模範生,則必須英文達 70 分
=n(A2)+n(A3)+n(A5)
且數學及格
(A2∩A3)
-n -n(A3∩A5)
亦即小文的英文成績未達 70 分或數學成績不及
(A2∩A5)
-n +n(A2∩A3∩A5)
-n(A2)
=333+200-166-66-100+33 格皆不符合資格參選模範生
=234
(個) 故選E
互動式教學講義.數學(2) 解答篇 37
頁碼 頁碼
141 u 1 若第三排舖 i

則前二排可舖 1 或 3 或 5 個

1 若舖 1 個 當有手機者皆有平板時,A 最大值為 24
而 A 的最小值為 35+24-45=14
則可分為
∴14NAN24 ................................... 1
B=35-A
∴11NBN21 ................................... 2
C=24-A
∴0NCN10 ..................................... 3
由取捨原理知
45-35-24+A=D
c D=A-14
其餘位置舖 ,共 3 種 ∴0NDN10 ..................................... 4
由1、2、3、4得正確答案為BCD
2 若舖 3 個 142 o A ×:由題意可知文氏圖如下

則可分為

得 34-x=y
c x+y=34
B ○:∵英文不及格有 11 人而數學及格但英文
不及格的有 y 人
其餘位置舖 ,共 4 種 ∴0NyN11
由A x+y=34 得 23NxN34
3 若舖 5 個 ,則共 1 種 C ×:∵英數都及格的有 x 人且英文及格的學
生國文也都及格
∴國英數都及格的有 x 人
故至少一科不及格的有 50-x 人
D ×:由BC知 16N50-xN27
∴三科中至少有一科不及格的學生最少
2 若第三排舖 2 個 蓋住 有 16 人,三科中至少有一科不及格的
學生最多有 27 人
E ○
則最右邊 2×3 格可放 1 個或 3 個
故選BE
p 由大考中心公布的各科標準知小華成績為
國文均標,英文前標,數學均標,自然均標,社
會前標
A ○
B ×:國文不符規定
C ×:數學,自然皆未達前標不符規定
D ○
E ×:自然未達前標不符規定
其餘舖 ,共 3 種
故選AD
由1、2得共有 3+4+1+3=11(種)
38 互動式教學講義.數學(2) 解答篇
頁碼 頁碼
143 a
3-2 排 列
145 範例 1
1 6!=6×5×4×3×2×1=720(種)
2 5!=5×4×3×2×1=120(種)
如上圖 類題
x+y=765-224=541 .............................. 1 1 7!=7×6×5×4×3×2×1=5040(種)
y+z=648-224=424 .............................. 2 2 6!=6×5×4×3×2×1=720(種)
z+x=537-224=313 .............................. 3 3 5!=5×4×3×2×1=120(種)
1+2+3 146 範例 2
將 得 x+y+z=639 ...............4
2 5! 5!
1 P 52= = =5×4=20
將4-2得 x=215,故同時領甲、乙兩案但沒有 (5-2)! 3!
領丙案公投票者有 215 人 2 5P n+2 n
3 =12P 3

s A ○:1,2,3 滿足(B)條件, c 5(n+2) (n+1)n=12n(n-1)(n-2)


但不滿足 (A)條件,得獎金 100 元 ∵n≠0 ∴兩邊約掉 n 得
B ○:1,3,3 為三奇數,滿足(A)條件, 5(n2+3n+2)=12(n2-3n+2)
但不滿足(B) 條件,得獎金 100 元 c 7n2-51n+14=0 c(7n-2)
(n-7)=0
C ×:1,3,4 皆不滿足(A)與(B)
條件, 2
c n= (不合)或 7,故 n=7
得獎金 0 元 7
D ×:1,3,5 同時滿足(A)與(B)
條件, 類題1

得獎金 200 元 6! 6!
P 64= = =6×5×4×3=360
E ×:1,3,6 皆不滿足(A)與(B)
條件, (6-4)! 2!
得獎金 0 元 類題2

故選AB 4n(n-1)
(n-2)=5(n-1)
(n-2)(n-3) ∵nM4
144 d 1 恰連續兩個 0,計數 1 次 ∴兩邊約掉(n-1) (n-2)得 4n=5n-15 c n=15
1 :有 2×(3×3-1) 範例 3
1 P 53=5×4×3=60(個)
=2×8=16 次
2 個位數字為 1 或 3 或 5 有 3×4×3=36(個)
2 :有 2×2×3=12 次 偶數有 60-36=24(個)
3 :同2,有 12 次 3 1 百位數字為 1 有 4×3=12(個)
百位數字為 2 有 4×3=12(個)
4 :同1,有 16 次 百位數字為 3,十位數字為 1 有 3 個
以上共有 16+12+12+16=56 次 百位數字為 3,十位數字為 2 有 3 個
2 有兩組 00,計數 2 次 ∴第 30 個為 325
:有 2 種,計數 2×2=4 次 2 第 31 個為 341,第 32 個數為 342
147 類題
3 連續三個 0,計數 2 次
不含 0 時,數字和最小為 1+2+3+4+5=15>12
1 :有 2×3=6 種 ∴必有 0
2 :有 2×2=4 種 ∴可分為 0,1,2,3,6 與 0,1,2,4,5 等兩類討論:
1 由 0,1,2,3,6 所組成時,
3 :有 3×2=6 種 共有 P 55-P 44=120-24=96(個)
以上共有
(6+4+6)×2=32 次 2 由 0,1,2,4,5 所組成時,
4 連續四個 0,計數 3 次 共有 P 55-P 44=120-24=96(個)
1 :有 2 種 由1、2知共有 96×2=192(個)
範例 4
2 :有 2 種 先排有規定棒次者,甲只有 1 種選擇
以上共有
(2+2)×3=12 次 乙、丙、丁三人的棒次可以互換而有 3!種方法
5 連續五個 0,計數 4 次 剩下的 5 人任意排入所餘的五個棒次,有 5!種方法
總計 56+4+32+12+4=108 次 由乘法原理得 1×3!×5!=720(種)
互動式教學講義.數學(2) 解答篇 39
頁碼 頁碼
類題1 2 由取捨原理,
第一、三、五三場由特定的 3 人排列 (全部排法數)-(甲坐最前面之排法數)
排法共有 3!=6
(種) -(丁坐最後面之排法數)
第二、四兩場由剩下 7 人中選 2 人排列 +(甲坐最前面且丁坐最後面之排法數)
排法共有 P 72=42(種) =4!-3!-3!+2!
∴排法共有 6×42=252(種) =24-6-6+2
類題2 =14(種)
(一)
(二)
(三)(四)
(五)
(六)
× ×
先排第二與第三志願的方法有
P 52=5×4=20(種)
再排其他四個志願的方法有 類題1
P 54=5×4×3×2=120(種) (b-2)≠0 c a≠1 且 b≠2
(a-1)
∴共有 20×120=2400 種排法 ∴所求為(全部序組數)-(a=1 之序組數)
148 範例 5 -(b=2 之序組數)+(a=1 且 b=2 之序組數)
1 7 人任意排成一列 =5!-4!-4!+3!
方法有 P 77=7!=5040(種) =120-24-24+6
2 三人須完全相鄰,則先將甲、乙、丙三人綁起來 =78(種)
與其他四人進行直線排列,排完之後再鬆綁
共有 5!×3!=720(種)
3 甲、乙、丙三人須完全分開,則先排其他四人,
然後甲、乙、丙再從此四人排好之後的 5 個空隙
中找 3 個空隙排入
類題2
○○○○
↑↑ ↑ ↑ ↑ 所求排法數為
甲、乙、丙插空隙 (全部排法數)-(甲、乙相鄰之排法數)
共有 4!×P 53=1440(種) -(丙、丁相鄰之排法數)
n 任意排列)-n(三人完全相鄰)-n(三人完全分開)
4 ( +(甲、乙相鄰且丙、丁相鄰之排法數)
=5040-720-1440 =7!-6!×2!-6!×2!+5!×2!×2!
=2880
(種) =5040-1440-1440+480
類題1
=2640(種)
1 甲校先選星期一到星期日中連續的兩天,有 6 種
選法
2 另兩校再由剩下的五天中任選兩天,方法數為 P 52
由1、2可得共有
6×P 52=6×
(5×4)=120 種方法
故選D 150 範例 7
類題2 (4+2) !
1 =15(種)
4!2!
1 P 73=7×6×5=210(種)
2 這 8 字中“阿”有 2 個,“雄”有 2 個,
2 想像成排成一列的 4 個○,
○○○○ “棒”有 2 個,“帥”有 2 個
↑↑ ↑ ↑ ↑ 8!
∴排法數為 =2520(種)
產生的 5 個空隙中,選 3 個排入 3 輛車 2!2!2!2!
即有 P 53=5×4×3=60(種) A
3 □□□□□□□□□
149 範例 6 將 A 放正中間,再將 AABBBBCC 作排列,
1 所求為 8!
有 =420 種排法
(全部排法數) 2!4!2!
-(甲坐最前面之排法數) 類題1
=4!-3! 10!
=24-6=18(種) =210(種)
4!6!
40 互動式教學講義.數學(2) 解答篇
頁碼 頁碼
類題2 類題2

1 先由國、英、物、化、體五科中取兩科排第四、 設每次移動 1 格者有 x 次,2 格者有 y 次,


五節 3 格者有 z 次,
有 P 52=5×4=20 種方法 則 x+2y+3z=6,其中,x,y,z 為非負整數
2 再排第一、二、三、六、七節課, x 6 4 2 0 3 1 0
5! c y 0 1 2 3 0 1 0
計有 =60 種排法(數學有兩節)
2! z 0 0 0 0 1 1 2
由1、2可得共有 20×60=1200 種不同的安排 6! 5! 4! 3! 4! 2!
151 範例 8 ∴共 + + + + +3!+
6! 4! 2!2! 3! 3! 2!
∵A 排需依 1,2 的順序擊落,B =1+5+6+1+4+6+1=24(種)
排需依 3,4,5 的順序擊落,C 153 範例 10
排需依 6,7,8,9 的順序擊落 1 由 A 到 B 走捷徑,須向右走 5 次,向上走 4 次
∴所求方法數相當於 1 在 2 之 而右、右、右、右、右、上、上、上、上的排列
前;3 在 4 之前,4 在 5 之前; 中的每一種順序即是一種走法,例如“右右上上
6 在 7,7 在 8,8 在 9 之前的排列 右上右右上”即是下圖中的走法
9!
即共有 =1260 種方法
2!3!4!
類題1

6!
1 ×1=360(種)
2! 9!
6! 故走法有 =126(種)
2 ×1=120(種) 5!4!
3! 2! 7!
6! 2 如圖1, × =70(種)
3 ×2!=240(種) 1!1! 4!3!
3! 5! 4!
6! 3 如圖2, × =60(種)
4 ×1×1=180(種) 3!2! 2!2!
2!2!
6!
5 ×2!×3!=72(種)
5!
類題2

將原本 5 個節目視作一物與其他 3 個新節目排列而 圖! 圖@

8! 2! 3! 4!
有 =336
(種) 4 如圖3, × × =36(種)
5! 1!1! 1!2! 2!2!
152 範例 9
設長鳴有 x 次,短鳴有 y 次,則其間隔有 x+y-1 次
∴每一信號須時 3x+y+2(x+y-1)=30(秒)
x 4 1
c 5x+3y=32,x,y 為非負整數c 圖#
y 4 9
5 不經 C 點且不經 D 點的走法為
8! 10!
∴共有 + =70+10=80(種) (全部走法)-(必經 C 的走法)
4!4! 1!9!
-(必經 D 的走法)+(必經 C 且必經 D 的走法)
類題1
=126-70-60+36=32(種)
觀察蜂房號碼得知相鄰蜂房數字相差 1 或 2
因此設蜂房號碼 1 次增加 1 者有 x 次,1 次增加 2 者
有 y 次,則 x+2y=10,x,y 為非負整數
x10 8 6 4 2 0

y 0 1 2 3 4 5
類題1
10! 9! 8! 7! 6! 5!
∴共有 + + + + + 1 必過原點的走法,即是走 A → O → B 路線
10! 8!1! 6!2! 4!3! 2!4! 0!5!
5! 7!
=1+9+28+35+15+1 ∴共有 × =210(種)
3!2! 5!2!
=89 (種)
互動式教學講義.數學(2) 解答篇 41
頁碼 頁碼
2 如下圖,必經過第二象限的走法只有 2 大類(經 155 範例 12
過 P 或經過 Q) 1 英文字母有 26 字,數字 0∼9 有 10 個,且最後
一位數字不用 4
車牌: ,

但沒有 者

A → P 及 A → Q 皆是 5 格 ∴共有 26×26×(10×10×10×9-1)
∴任何通過第二象限的走法,必過 P 或 Q 故選D
即1 A → P → B 2 8 位數共有 9×107 個門號,
5! 7! 而 7 位數共有 9×106 個門號
得 × =70(種) ∴共增加 9×107-9×106=(90-9)×106
3!2! 6!1!
2 A→Q→B =81×106=81000000(個)
5! 類題1
得 ×1=5(種)
4!1! 車牌為: ,
∴共有 70+5=75(種)
154 類題2
且23兩格不得同時為 4
∵PT+TQ>PQ ∴共 25×(103-10)=25×990,故選D
類題2
∴由 A 到 B 中間過程中,一定要走一次斜線會比較
1 5 件獎品每件都有 4 種分法

故所求為 4×4×4×4×4=45=1024(種)
2 A 未得時,每件獎品都有 3 種分法,
故 A 至少得 1 件的分法為
(全部分法)-(A 未得的分法)=45-35=781(種)
3 先將 5 件獎品選 1 件給 A,
如上圖所示,捷徑走法可分為 剩下的 4 件獎品再分給 B,C,D 三位小朋友
1 A→P→Q→B ∴所求為 5×34=405(種)
4! 3! 4 A 至少得 2 件的分法為
得 ×1× =12(種)
3!1! 2!1! (全部分法)-(A 得 0 件的分法)
2 A→R→S→B -(A 得 1 件的分法)
4! 3! =45-35-5×34=376(種)
得 ×1× =18(種)
2!2! 2!1! 156 範例 13
故共有 12+18=30(種) 1 設 5 人為甲、乙、丙、丁、戊,
範例 11 而船為 A,B,C,則每個人都有 3 種選船的方法
1 第 1 封信有 4 個郵筒可供選擇, ∴所求為 3×3×3×3×3=35=243(種)
第 2、3、4、5 封信也一樣 2 設 6 人為甲、乙、丙、丁、戊、己,
∴4×4×4×4×4=45=1024(種) 而船為 A,B,C
2 第 1 個酒杯可以倒入 5 種不同的酒, 則任意搭乘時有 36 種方法
第 2、3、4 個酒杯也一樣 但 6 人全在 A 船會沉,6 人全在 B 船或 C 船時也
∴5×5×5×5=54=625(種) 會沉
類題 ∴所求為 36-3=726(種)
1 每一個選項都有“選”與“不選”兩種選擇 3 設三船為 A,B,C,任意搭乘時有 37 種方法
∴共有 2×2×2×2×2=32 種選擇方式 但1 7 人同船會沉,有 3 種
但扣除 A,B,C,D,E 都不選的一種 2 6 人同船,另 1 人在其他船也會沉,此時有
故共有 25-1=31(種) 7×P 32 種方法
3 船中選出 2 船與(6 人),(1 人)作直線排列
2 第 1 次可能出現 6 種點數,第 2,3,4 次也一樣
將 7 人分成(6 人),(1 人)兩組,有 7 種方法
∴6×6×6×6=64=1296(種)
∴共有 37-3-7×P 32=2187-3-42=2142(種)
42 互動式教學講義.數學(2) 解答篇
頁碼 頁碼
類題 2 x+2y=5
將小春與冬冬綁起視為一人, ∴(x﹐y)有(5﹐0),(3﹐1),(1﹐2)等 3 組解
與阿秋、冬冬乘船渡河, 故由第一階到第五階的走法有
共有 33-3=24 種方法 5! 4! 3!
四人同船 + + =1+4+3=8(種)
5! 3!1! 1!2!
範例 14
同理,由第六階到第十階時亦有 8 種方式
下列兩種為不同的排法
由乘法原理可得上樓方式共有 8×8=64(種)

3-2
基礎題
158 1 4!=24(種)
2 ∵4P n3=5P n-1
3
∴本題即為 2 組, 3 組, 4 組所成之直線
∴4n(n-1)
(n-2)=5(n-1)
(n-2)
(n-3)
排列 c 4n=5n-15
9! c n=15
故所求為 =1260(種) 3 1 首位數字不排 0
3!2!4!
157 類題1 ∴共有 5×5×4=100(個)
1 將「內、外、夾」綁在一起,與「弓」排列, 2 c有 5×4=20(個)
有 2!=2 種排法
c有 4 個

內外夾 △弓△
c有 4 個
再將「大、立、腕」排入△產生的空隙中,
有 3!=6 種排法 ∴第 28 個數字為 215,
最後「內、外、夾」可互換位置, 第 29 個為 230,
有 3!=6 種排法 第 30 個為 231
∴共有 2×6×6=72 種排法 3 可被 5 整除,個位數字必為 0 或 5
故選B c有 4×4=16(個)
2 不考慮「弓」的排法,
c有 5×4=20(個)
將「大、立、腕」視為相同物,
6! 720 故共有 16+20=36 個
故有 = =120 種排法 4
3! 6 c 4×3=12
類題2

1 設一次上 1 階的有 x 次,一次上 2 階的有 y 次


則 x+2y=10,x,y 為自然數或 0 c 4×3=12

x 10 8 6 4 2 0
c
y 0 1 2 3 4 5 c 4×3=12
10!
而 x=10,y=0,表示有 =1(種)
10!
x=8,y=1,表示共需走 9 次,其中有 8 次 1 12+12+12=36(種)
階的,1 次 2 階的 159 5 1
9! 2!×3!×3!=2×6×6
∴共有 =9(種)
8!1! =72(種)
同理,可得總數為 2 男生先排列,女生再插空隙
10! 9! 8! 7! 6! 5! 有 3!×P 43=6×(4×3×2)
+ + + + +
10! 8!1! 6!2! 4!3! 2!4! 5! =144(種)
=1+9+28+35+15+1 6 P 4-P 3-P 3+P 52=840-120-120+20
7 6 6

=89
(種) =620(種)
互動式教學講義.數學(2) 解答篇 43
頁碼 頁碼
5! u A ○:每位評審對三位小朋友的評分方式均有
7 =10(種)
3!2! 33 種
5! 共有 33×33×33×33×33=315(種)
8 1 =60(種)
2! B ○:若 5 位評審均給小雄雄 3 分
2 將甲、乙、丙視作相同物,但乙、丙可對調 則小雄雄得 15 分
5! C ×:每位評審對三位小朋友的評分方式
而有 ×2!=40(種)
3! 有 3!=6(種)
9 設一次移動 1 格有 x 次,一次移動 2 格有 y 次 共有 3!×3!×3!×3!×3!=65(種)
則 x+2y=7,x、y 為非負整數 D ○:小雄雄最少得 5 分,最多得 15 分
x 7 5 3 1 E ×:共有 5∼15 等 11 種得分可能
y 0 1 2 3 故選ABD
7! 6! 5! 4! i(A□□□□□A 之排列數)
∴共有 + + +
7! 5!1! 3!2! 1!3! -(A□□□□□A,但 B,C 相鄰之排列數)
=1+6+10+4 得 5!-4!×2=120-48=72(種)
=21(種) 6!
o 4× =4×360=1440(種)
6! 3! 2!
160 0 × =15×3=45(種) 捏陶土與洗澡視為相同物排列
4!2! 1!2!
早上選 1 小時算數學
162 p 1 若前 8 次共撿走 12 顆石子:
設前 8 次中有 x 次撿走 1 顆石子,y 次撿走 2
顆石子
x+2y=12
q 3×3×3×3=34=81(種) 則 c x=4,y=4
x+y=8
w 25-2=32-2=30(種) 8!
有 1 人都沒分到 方法數為 =70(種)
e 1(任意坐)- (5 人同船) 4!4!
5
=3 -3=243-3 2 若前 8 次共撿走 11 顆石子:
=240(種) 設前 8 次中有 x 次撿走 1 顆石子,y 次撿走 2
2(任意坐) -(6 人同船)-(5 人同船) 顆石子
6 3
=3 -3-6×P 2 x+2y=11
則 c x=5,y=3
=729-3-36 x+y=8
=690(種) 8!
方法數為 =56(種)
進階題 5!3!

5! 由1、2可得共有 70+56=126(種)
r =60
(種) a 設會開車者為甲、夫婦 Aa、夫婦 Bb
2!
t 設一次取 2 枚有 x 次,一次取 3 枚有 y 次 三組司機與車進行直線排列有 3!種
則 2x+3y=10,x,y 為非負整數 剩下 3 人
x 5 2 1 每車 1 人c 3!=6(種)
y 0 2 2 一車 2 人,一車 1 人c 3×3!=18(種)
(2﹐1﹐0)排列
5! 4! 3 人中選 2 人一組
∴共有 + =1+6=7 種取法 3 3 人同車c只能坐甲所開之車,共 1 種
5! 2!2!
161 y 取 3 個相鄰攤位的方法有 5 種 ∴共有 3!×(6+18+1)=6×25=150(種)
歷屆試題
s 1 ∵杜鵑、山茶都被選到
5×4
∴再從剩下的 5 盆中選 2 盆:=10
2
2 四盆花排一列且杜鵑、山茶相鄰:
3!×2!=12
杜鵑、山茶可對調
杜鵑、山茶視為一體與其他兩盆排列
∴5×3!=5×6=30 種 ∴共有 10×12=120 種
44 互動式教學講義.數學(2) 解答篇
頁碼 頁碼
163 d 1 若牛肉麵(大滷麵)點兩次, 3 3r+1=r+2 或(3r+1)+(r+2)=19
則“牛牛大咖排”排成一列, 1
c r= (不合)或 r=4,故 r=4
其中“牛牛大”不相鄰 2
3! 類題2

有 2 × 2!×
2! )
=2×6=12(種)
設選手人數有 n 人,則 C n2=78 c
n(n-1)
=78
牛,大 2
2 若咖哩飯 (排骨飯) 點兩次, c n2-n-156=0 c(n-13)
(n+12)=0
則“牛大咖咖排”排成一列, c n=13 或-12(不合),故選手有 13 人
其中“牛大”不相鄰且“咖咖”不相鄰 範例 2
5! 4! 13×12×11×10
有 2×

- ×2!-4!+3!×2!
2! 2! ) 1 C 13
4 =
1×2×3×4
=715(種)

咖,排 2 從男生 7 人中選出 2 人的方法數有 C 72 種


=2×24=48(種) 從女生 6 人中選出 2 人的方法數有 C 62 種
由1、2知共有 12+48=60(種),故選B 由乘法原理知共有 C 72×C 62=21×15=315(種)
f 3 道菜中有 1 道含豬肉,1 道含雞肉,1 道含牛 3(全部方法數)-(沒有女生的方法數)
肉,而白菜、豆腐、香菇各自可選擇與哪種肉搭配 =C 13 7
4 -C 4=680
(種)
∴共有 3×3×3=33=27(種),故選E 類題1

g 設曾有過不愉快的兩人為甲男乙女 組隊方式可能有
1 若甲男乙女相鄰,則有 2 男 6 女,3 男 5 女及 4 男 4 女等 3 種
另一側 3 人排列
一共有 C 42C 76+C 43C 75+C 44C 74=42+84+35=161
(種)
4×2×2!×2!×3!=4×2×2×2×6=192(種) 166 類題2
甲、乙排列 1 C 22C 63×5!=2400(種)
甲乙與另一人在同側排列
老師的左、右側 2 C 22C 63×4!×2!=960(種)
其他 4 人選 1 位搭甲、乙 範例 3
2 若 3 男在同一側, 8×7
則有 3!×3!×2!=6×6×2=72(種) 1 C 82= =28(個)
1×2
而不可能有甲乙相鄰且 3 男在同一側的情形發生 2 C 12 3 4
2 -C 2-C 2+1=66-3-6+1=58
(個)
∴所求為 (任意排列)-(甲乙相鄰)-(3 男在同一側) 4 線共點所產生的一個交點
類題1
=6!-192-72=720-192-72=456(種)
8×7
1 C 82-8= -8=20(條)
3-3 組 合 1×2
2 “最多”的情形就是“除此情形之外任 2 線不平
164 範例 1
行,且任 3 線不共點”
20!
1 C 20
2 = =190 C 12 5 4
2 -C 2+1-C 2=51
(個)
18!2! 5 線共點所產生的一個交點
2 ∵C 18 18 18
4 =C 14=C n+2 ∴n+2=4 或 14 c n=2 或 12 類題2
n! (n-2)! 1 1 直線共有 C 52=10(條),三角形有 C 53=10(個)
3 3× =10×
3!(n-3) ! 2!
(n-4)! 2 直線有 C 82-C 52+1=28-10+1=19(條)
(n-1)
n (n-2) (n-2) (n-3) 三角形有 C 83-C 53=56-10=46(個)
c 3× =10×
1×2×3 1×2 2 利用(任 3 點決定一個三角形)-(共線所多算的
∵nM3 且 n-2M2 c nM4 三角形)得 C 63-2C 33=20-2=18(個)
∴n-2≠0,左右同時除以 n-2 167 範例 4
得n(n-1)=10(n-3)c n2-11n+30=0 將不停水日視同○,停水日視同×
c (n-5)=0 c n=6 或 5
(n-6) 則週一至週日停水日安排的方法恰為
165 類題1 ○,○,○,○,○,×,×排成一列
28! 且兩個“×”不相鄰的排法數
1 C 28
27= =28
27!1! 想像成將○,○,○,○,○排成一列
2 ∵n=8+12=20 6×5
兩個“×”去插空隙,故共有 C 62= =15(種)
20! 20×19×18 1×2
∴C n17=C 20
17= = =1140
17!3! 1×2×3
互動式教學講義.數學(2) 解答篇 45
頁碼 頁碼
類題1
範例 7
10
1 C =120
3 (種) 1 1(2a-3b)4=C 4(2a)0
4
+C 4(2a)
1
3
(-3b) 1

2 要求三節設置無線網路的車廂兩兩不相連接, +C 42(2a) 2
(-3b) 2

4 1 3
可以想像成在七節車廂間的 8 個空隙中, +C (2a)
3 (-3b) +C 4(-3b)
4
4

任取 3 個空隙插入 3 節車廂 =16a4-96a3b+216a2b2-216ab3+81b4


2(1+x)4=C 40+C 41 x+C 42 x2+C 43 x3+C 44 x4
=1+4x+6x2+4x3+x4
故共有 C 83=56(種) 2(3x2-2y) 7
之展開式之一般項為
r r
類題2 7
C (3x
r
2 7-r
) (-2y) =C 7r 37-r×(-2) ×x14-2ryr
由於兩端的燈不可熄滅,此即相當於中間的 10 盞選 令 r=5 得該項為 C 7532× (-2) 5
×x4y5=-6048x4y5
4 5
擇 3 盞熄滅 (但不可相連) 即 x y 項之係數為-6048
故共有 C 83=56(種) 170 類題1
範例 5 1(3a-b)5=C 5(3a)
0
5 5
+C (3a)
1
4
(-b)1+C (3a)5
2
3
(-b)2
5 2
在照片的左右上下各取一條直線即可包含此照片 +C(3a)
3 (-b)+C 4(3a)(-b)+C 55(-b)5
3 5 1 4

∴C 11C 21C 31C 21=12(種) =243a5-405a4b+270a3b2-90a2b3+15ab4-b5


左 右 上下 2(1-x)6=C 60+C 6(-x) 1
+C 6(-x) 2
+C 6(-x) 3
1 2 3
168 類題1
+C 64(-x) 4
+C 65(-x) 5
+C 66(-x) 6

含 A 之長方形有 C 11×C 31×C 31×C 11=9(個) 2 3


=1-6x+15x -20x +15x -6x +x 4 5 6

含 B 之長方形有 C 31×C 11×C 31×C 11=9(個) 類題2


含 A 且含 B 之長方形有 C 11×C 11×C 31×C 11=3(個) 2
( )
10

∴至少含 A 或 B 兩點之一的長方形有 -x2 之展開式中,一般項為


x
9+9-3=15(個) 2 10-r
類題2 C 10
r
() x
(-x2)r=C 10
r 2
10-r
×(-1)r
×x-10+r+2r
鉛垂線取 2 條,水平線取 2 條, =C 10 10-r r
×x3r-10
r 2 ×(-1)
即可組成一個矩形,如右圖 1 令 3r-10=14 c r=8,得 x 項之係數為 14

故矩形有 C 72C 52=21×10 C 10 2 8


8 ×2 × (-1) =45×4×1=180
=210(個) 17
2 令 3r-10=7 c r= (不合),即 x7 項係數為 0
範例 6 3
1 C 62C 42C 22=90(種)
171 範例 8
C 62C 42C 22 1 ∵C 33=C 44
2 =15(種)
3! ∴C 33+C 43+C 53+C 63+……+C 93
3 C 64C 21C 11=30(種) =C 44+C 43+C 53+C 63+……+C 93
C 64C 21C 11 =C 54+C 53+C 63+……+C 93
4 =15(種)
2! =C 64+C 63+……+C 93=C 10 4 =210
5 C 63C 32C 11=60(種) 3
2 ∵C 0=C 0 4

6 C 63C 32C 11=60(種) ∴C 30+C 41+C 52+……+C 89 86


類題1
=C 40+C 41+C 52+……+C 89 5 5 89
86=C 1+C 2+……+C 86
1 C 10 6 3
4 C 3C 3=4200 (種) =C 62+C 63+……+C 89 90 90
86=C 86=C 4
2 甲、乙兩人住入 4 人房,尚缺 2 名室友, 即 n=90,r=4 或 86,
故有 C 82C 63C 33=560(種) 故數對(n﹐r)=(90﹐4)或(90﹐86)
3 C 21×(C 84C 41C 33)=560(種) 類題1
B、C 兩間先選 1 間住甲、乙
1 C 22+C 32+C 42+……+C 98 2 +C 2
99

169 類題2
=C 3+C 2+C 2+……+C 2 +C 99
3 3 4 98
2
C 63C 33 =C 43+C 42+……+C 98 +C 99
=C 100
1 ×C 31×C 31=90(種) 2 2 3
2! 故數對(n﹐r)=(100﹐3)
C 21C 21C 21 2 C 10+C 21+C 32+……+C k-1k
=C k+1
2 ×C 32×C 11×C 32×C 11=36(種) k-1=190
2! (k+1) !
乙邊 3 人中選 2 人先賽
甲邊 3 人中選 2 人先賽
∴ =190 c k2+k-380=0
(k-1) !2!
平分成甲、乙兩邊
c(k+20) (k-19)=0 c k=19 或-20(不合),故 k=19
46 互動式教學講義.數學(2) 解答篇
頁碼 頁碼
類題2 C 42C 22
n 9 =3(種)
(1+x) =C n0+C n1 x+C n2 x2+……+C nn xn 中, 2!
xk 項係數為 C nk 0 1 C 82C 62C 42C 22=2520(種),故選D
∴所求 x4 項係數為 2 其餘 5 位轉組學生任意編入乙、丙、丁班有
C 44+C 54+C 64+……+C 20 4 35 種,其中 5 位同班不符合編班原則
=C 55+C 54+C 64+……+C 20 6 6 20
4 =C 5+C 4+……+C 4 有 3 種須扣除得 35-3=240(種)
21×20×19×18×17 6!
=C 215 = =20349 q C 64= =15
1×2×3×4×5 4!2!
〔另解〕 故選C
2 3 20
視(1+x) +(1+x) +(1+x) +……+ (1+x) 為首 w 原式為 C 33+C 32+C 42+……+C 10 2
4 4 10
項 1+x,公比 1+x 之等比級數,其和為 =C 3+C 2+……+C 2
20 21
(1+x) 〔(1+x) -1〕 (1+x) -(1+x) =C 10 10 11
3 +C 2 =C 3 =165

(1+x) -1 x 175 e 預賽有 8×C 42=8×6=48(場)
4
欲求 x 項之係數 單敗淘汰賽至產生冠軍隊有 16-1=15(場)
21
則相當於分子 (1+x) -(1+x)中求得 x5 項之係數 另爭三、四名時加賽一場
即 C 21
5 =20349 共需 48+15+1=64(場),故選C
172 範例 9 進階題
1 若只安排 4 天,每天 2 個時段, r(6﹐1﹐0)c C 76C 11×3!=42
則有 C 54×(C 32)4=405(種) (5﹐2﹐0)c C 75C 22×3!=126
2 若 5 天全排,則有 3 天排 2 個時段,2 天排 1 個 (4﹐3﹐0)c C 74C 33×3!=210
時段,則有 C 53× (C 32)3× 2
(C 31) =2430(種) 共有 42+126+210=378(種)
由1、2可得共有 405+2430=2835 種排法 C 93C 63C 33
t 9 人平分成三組的方法有 =280(種)
類題 3!
(C 31C 31×2!)×(C 21C 21×2!)×(C 11C 11×2!)=288(種) C 71C 63C 33
A、B 在同組的方法有 =70(種)
第一排 第二排 第三排
2!
∴A、B 不在同組的方法有 280-70=210(種)
n
y 所求為 (任意取兩數) -n(兩數在同一個括號內)
3-3
=C 2 -(C 2+C 2+C 42+……+C 11
66 2 3
2 )

基礎題 =2145-(C 33+C 32+C 42+……+C 11 2 )


4 4 11
173 1 n=8+14=22 =2145-(C 3+C 2+……+C 2 )
2 千 百 十 個 =2145-C 12 3 =2145-220=1925 (種)
先由個、十、百、千四個位數中選 3 個放 1,剩 u 先將 0,1,2,3,4,5,6 分類
下的再填入其他數字,而有 C 43×5=20(個) 3k 3k+1 3k+2
3 十 個 0,3,6 1,4 2,5
小 大 1 單組取 3 個(只有 3k 這組)有
∵個位數字>十位數字 2×2×1=4(個)
∴只能由 1∼9 等 9 個數字中選 2 個出來,大者 2 每組取 1 個組成三位數
排個位,小者排十位,共有 C 92=36(種) 1 取到 0 c C 21×C 21×(2×2×1)=16(個)
12×11 2 不取到 0 c C 21C 21C 21×3!=48(個)
4 C 12 10
2 ×C 1 = ×10=660(種)
2 故共有 4+16+48=68(個)
5 C 10 5
2 -C 2+1=45-10+1=36 (個) 176 i 1 2 同 2 同 1 異
5 直線的交點 P 5!
6(C 20
2 -19)
×2!=(190-19)×2=342(種) c C 32×C 51× =3×5×30=450(種)
2!2!
相鄰座位有 19 種
20 個位置任選 2 個 2 2同3異
5 4
174 7 C ×C =10×6=60(個)
2 2 5!
c C 31×C 63× =3×20×60=3600(種)
8 1 C 62×C 42×C 22=15×6×1=90(種) 2!
2 C 61×C 52×C 33=6×10×1=60(種) 3 5 異c C 75×5!=21×120=2520(種)
C 62C 42C 22 90 由1、2、3得
3 = =15(種)
3! 6 共有 450+3600+2520=6570(種)
互動式教學講義.數學(2) 解答篇 47
頁碼 頁碼
歷屆試題 2 含第 2 棟: 1 2 3 4 5 6 7 8
o 如右圖,由六點 D、E、F、 × L ×
G、H、I 所圍成之形狀為正 情形同1c C 42=6(種)
六邊形 3 不含第 1,2 棟: 1 2 3 4 5 6 7 8
A ○:左、右、下各取一 × × ×
點,得 相當於在 4∼8 號取三棟互不相鄰,
C 21C 21C 21=8(個) 僅能取 4、6、8 號大廈,有 1 種
B ○:△DHF 及△EGI 為銳角三角形 共有 6+6+1=13(種)
C ×:△DIF 為以∠D 為直角的直角三角形, 故選B
同理 E、F、G、H、I 每一個點皆可找 g
到一個以其為直角頂的直角三角形: ∵前 5 位數字由左至右遞增,
△EIF,△FEH,△GEH,△HGD, 後 5 位數字由左至右遞減
△IGD,共計有 6 個 ∴第 5 位(正中間位)數字必是 9
D ×:鈍角三角形不存在 而前 4 位數字的選法有 C 84 種,
E ×:正三角形有 2 個,即△DHF 及△EGI 排法有 C 84×1=C 8(種)
4

故選AB 同理,後 4 位數字的排法有 C 44×1=1(種)


p 1 若己被選中,則有 C 11C 51=5(種) 8!
∴所求為 C 84×1= (種)
2 若己不被選中,則由右手持拍選手選 1 4!4!
搭左手持拍選手選 1 即可,共有 C 31C 21=6(種) 故選A
由1、2得 5+6=11 種 h 1∼20 取相異 a、b、c 成等差,且 a<b<c,
故選C 則 2b=a+c,可推得 a+c 為偶數
177 a A ○:x24 的係數為 C 12 0 =1, 1 若 a、c 皆為奇數,則有 C 10 2 =45
(種)
10 7 12 10
x y 係數為 C 7 =792 2 若 a、c 皆為偶數,則有 C 2 =45(種)
B ×:x12y6 的係數為 C 12 6 =924, 由1、2可得
10 7 12
x y 係數為 C 7 =792 共有 45+45=90(種)
C ×:x14y5 的係數為 C 12 5 =792
8 8
D ○:x y 係數為 C 8 =495 12 3-4 機 率
故選AD 179 範例 1
s(1+a2 )6 1 以(a﹐b)來表示樣本點,其中 a 表示第一次擲出
=C 6(a2
0 ) 0
+C 6(a2
1 ) 1
+C 6(a2
2
2
)+C 6(a2
3 )3
的點數,b 表示第二次擲出的點數
+C 64(a2 ) 4
+C 65(a2 ) 5
+C 66(a2 )6

+(C 1+2C 63+22C 65)a2


∴樣本空間
6 6 2 6 3 6 6
=(C 0+2C 2+2 C 4+2 C 6) S={(1﹐1)﹐(1﹐2)﹐(1﹐3)﹐(1﹐4)﹐
∴b=C 61+2C 63+22C 65,故選B (2﹐1)﹐(2﹐2)﹐(2﹐3)﹐(2﹐4)﹐
d 若任選兩球不同口味,則有 C n2 種,由 C n2>100 (3﹐1)﹐(3﹐2)﹐(3﹐3)﹐(3﹐4)﹐
14×13 (4﹐1)﹐(4﹐2)﹐(4﹐3)﹐(4﹐4)}
當 n=14 時,C 14 2 = =91<100
2 2 共有 4×4=16 個樣本點
15×14 180 類題
當 n=15 時,C 15 2 = =105>100
2 以(a﹐b)表示樣本點,其中 a 表示硬幣出現正面或
∴n 的最小值為 15 反面,b 表示骰子出現的點數
若任選兩球可為同一口味, 1 樣本空間
即可選擇兩球不同口味或兩球相同口味, S={(正﹐1)﹐(正﹐2)﹐(正﹐3)﹐(正﹐4)﹐
則有 C 15
2 +15=105+15=120
(種) (反﹐1)﹐(反﹐2)﹐(反﹐3)﹐(反﹐4)}
故選D 2 共有 2×4=8 個樣本點
178 f 1 含第 1 棟: 1 2 3 4 5 6 7 8 範例 2
L × × 樣本空間
相當於在 4∼8 號取兩棟不相鄰,可以想像成 S={(正﹐正﹐正)﹐(正﹐正﹐反)﹐
在三棟大廈間的 4 個空隙中,任選 2 個空隙 (正﹐反﹐正)﹐(正﹐反﹐反)﹐
插入兩棟大廈 (反﹐正﹐正)﹐(反﹐正﹐反)﹐
c C 42=6(種) (反﹐反﹐正)﹐(反﹐反﹐反)}
48 互動式教學講義.數學(2) 解答篇
頁碼 頁碼
A={(正﹐正﹐正)﹐(反﹐反﹐反)} 範例 4
B={(正﹐正﹐反)﹐(正﹐反﹐正)﹐ C 51C 21 5×2 10
= =
(反﹐正﹐正)} C 72 21 21
C={
(正﹐正﹐正)﹐(正﹐正﹐反)
﹐ 類題1
(正﹐反﹐正)﹐(反﹐正﹐正)} 四個球標號和為奇數的情形有 3 奇 1 偶和 1 奇 3 偶
1 A 和 C 的和事件為 兩種,又 1 至 7 號中有 4 個奇數,3 個偶數
A∪C={ (正﹐正﹐正)
﹐(反﹐反﹐反)﹐ C 43C 31+C 41C 33 16
故所求機率為 =
(正﹐正﹐反)﹐(正﹐反﹐正)﹐ C 74 35
(反﹐正﹐正)} 類題2

2 A 和 B 的積事件為 A∩B=∅ 4 球之號碼中第二大數目是 7


∴A,B 為互斥事件 即第一大數目為 8,9,10 中選 1 個
3 A 的餘事件為 第三及第四大數目為 1,2,3,4,5,6 中選 2 個
A'={(正﹐正﹐反)﹐(正﹐反﹐正)﹐ C 31C 11C 62 3
故所求機率為 10 =
(正﹐反﹐反)﹐(反﹐正﹐正)﹐ C4 14
(反﹐正﹐反)﹐(反﹐反﹐正)} 183 範例 5
類題1 樣本空間有 24=16 個樣本點
樣本空間 S= {1﹐2﹐3﹐4﹐5﹐6} 1 四枚均為正面
A={1﹐3﹐5} ,B={1﹐2﹐3﹐4﹐5}, 即(正﹐正﹐正﹐正)1 種情形
C={2﹐4﹐6} 1
故所求機率為
1 A 和 B 的和事件為 A∪B= {1﹐2﹐3﹐4﹐5} 16
2 A 和 B 的積事件為 A∩B= {1﹐3﹐5} 2 一枚正面、三枚反面
3 A 的餘事件為 A'= {2﹐4﹐6} 即(正﹐反﹐反﹐反),(反﹐正﹐反﹐反),
4 ∵A∩B≠∅ (反﹐反﹐正﹐反),(反﹐反﹐反﹐正)
∴A,B 沒有互斥 4! 4 1
有 =4 種情形,故所求機率為 =
5 ∵A∩C=∅ 3! 16 4
∴A,C 互斥 3 兩枚正面、兩枚反面
181 類題2 即(正﹐正﹐反﹐反),(正﹐反﹐正﹐反),
1 樣本空間為 (正﹐反﹐反﹐正),(反﹐正﹐正﹐反),
S={ABC﹐ABD﹐ABE﹐ACD﹐ACE﹐ADE﹐ (反﹐正﹐反﹐正),(反﹐反﹐正﹐正)
BCD﹐BCE﹐BDE﹐CDE} 4!
有 =6 種情形
共有 C 53=10 個樣本點 2!2!
2 抽到包含 A 書的事件為 6 3
故所求機率為 =
{ABC﹐ABD﹐ABE﹐ACD﹐ACE﹐ADE}
, 16 8
共有 6 個樣本點 類題

範例 3 樣本空間有 23=8 個樣本點


撲克牌裡 4 點有 4 張,5 點有 4 張 1 一正面、兩反面
4+4 2 即(正﹐反﹐反),(反﹐正﹐反),(反﹐反﹐正)
∴所求機率為 =
52 13 3! 3
有 =3 種情形,故所求機率為
類題1 2! 8
號碼和為 36 之情形有 2 三次都出現同一面的情形有
(1﹐35)
,(2﹐34),……,(17﹐19)共 17 種 (正﹐正﹐正),(反﹐反﹐反)2 種
17 1 2 1
故所求機率為 52 = 故所求機率為 =
C2 78 8 4
182 類題2 184 範例 6
完全立方數之情形有兩大類 樣本空間樣本點個數有 6×6=36 種
1 乘積為 23 者有(1﹐8) ,(2﹐4)兩種 而點數和為 8 的情形有
2 乘積為 33 者有(3﹐9) 一種 (6﹐2),(5﹐3),(4﹐4),(3﹐5),(2﹐6)
2+1 3 1 5 5
故所求機率為 9 = = 共 5 種,故點數和為 8 的機率為 2 =
C2 36 12 6 36
互動式教學講義.數學(2) 解答篇 49
頁碼 頁碼
b:擲一顆骰子兩次,各點數和出現的機率為 類題1

點數和 2 3 4 5 6 7 樣本空間 S 有 72=49 個樣本點


1 2 3 4 5 6 令 C 表示取出 1 顆紅球 1 顆白球的事件,則
機率
36 36 36 36 36 36 n(C)=C 41C 31+C 31C 41=12+12=24 個樣本點
點數和 8 9 10 11 12 n(C) 24
故所求機率 P(C)= =
5 4 3 2 1 n(S) 49
機率
36 36 36 36 36 類題2

類題1 欲選出兩個格子不在同行,可先從 4 行中選 2 行,


點數和 4 5 6 7 8 9 再從每一行的 4 格中選 1 格
1 2 3 4 5 6 C 42C 41C 41 6×4×4 4
機率 所求機率為 16 = = ,故選E
36 36 36 36 36 36 C2 8×15 5
點數和 10 11 12 13 14 187 範例 9
5 4 3 2 1 1 P
(A∪B)=P(A)+P(B)-P(A∩B)
機率 1 1 1 9 3
36 36 36 36 36 = + - = =
6 1 3 2 12 12 4
∴點數和 9 點時機率 = 為最大,故選D 2 P(A'∩B)=P(B)-P(A∩B)
36 6
類題2 1 1
= -
4! 2 12
2! 1 3×3×2 1 2!2! 3 5
P1= 2 = ,P2= = ,P3= = =
2 2 6 2
2 2 4
8 12
類題1
∴P1=P2>P3,故選B
1 P(A∩B)=P(A)+P(B)-P(A∪B)
185 範例 7
1 42×41×40×39×38×37 1 5 3 3
= + - =
r C5 39 42
C6 1×2×3×4×5×6 2 8 4 8
= = 39 = 2 P(A'∩B')=P((A∪B)')=1-P(A∪B)
R 1 C5 39×38×37×36×35
C 42 1×2×3×4×5 3 1
6 =1- =
82 4 4
= ~9 3 P(A'∪B')=P((A∩B)')=1-P(A∩B)
9
故選D 3 5
=1- =
類題 8 8
21×20×19×18×17×16 4 P(A'∩B)=P(B)-P(A∩B)
C 21
1×2×3×4×5×6 5 3 1
所求機率 p=
6
= = - =
C 42
42×41×40×39×38×37 8 8 4
6
類題2
1×2×3×4×5×6
21 20 19 18 17 16 1 1 1
= × × × × × ~ 6 P(A)= ,P(B)=
42 41 40 39 38 37 2 2 3
故選E 1 1
∵P(A∪B)=P(A)+P(B)-P(A∩B)= + -P(A∩B)
2 3
186 範例 8
1 樣本空間 S 有 P72=7×6=42 個樣本點 5
∴p= -P(A∩B)
令 A 表示取出 1 顆紅球 1 顆白球的事件,則 6
(A)
n =C 41C 31+C 31C 41=12+12=24 個樣本點 1 當 A∩B=∅ 時,P(A∩B)=0 為最小
n(A) 24 4 5
故所求機率 P (A) = = = ∴p 最大為
n(S) 42 7 6
7×6 1
2 樣本空間 S 有 C 72= =21 個樣本點 2 當 BfA 時,P(A∩B)=P(B)= 為最大
2×1 3
令 B 表示同時取出 1 顆紅球 1 顆白球的事件,則 5 1 1
∴p 最小為 - =
(B)
n =C 41C 31=12 個樣本點 6 3 2
n(B) 12 4 1 5
故所求機率 P (B) = = = 得 NpN ,故選D
n(S) 21 7 2 6
50 互動式教學講義.數學(2) 解答篇
頁碼 頁碼
188 範例 10 190 類題2

令 A 表示英文及格的事件,B 表示數學及格的事件 全取出後,依取出之先後次序排成一列有


15 12 6 (4+7) !
則P(A)= ,P (B)
= ,P (A∩B)= =330(種)
30 30 30 4!7!
1 P(A∪B) 白球先取完,即為最後一球排紅球,而前面由 4 個
=P(A) +P(B)-P(A∩B) (4+6) !
白球 6 個紅球作排列,有 =210(種)
15 12 6 4!6!
= + -
30 30 30 210 7
故白球先取完之機率為 =
21 7 330 11
= =
30 10 b:袋中有 m 個紅球,n 個白球,每次取一球,取
2 P(A∩B') m
後不放回,則白球先取完之機率為
=P(A) -P(A∩B) m+n
15 6 範例 12
= -
30 30 P12
4
4 人在不同月分出生之機率為
9 3 124
= =
30 10 故 4 人中會有人在相同月分出生的機率為
類題 P12
4 12×11×10×9 55 41
1- 4 =1- =1- =
1 P
(A'∩B) 12 12×12×12×12 96 96
=P(B) -P(A∩B) 類題

12 6 P 12
3
= - 三人皆不同生肖之機率為
30 30 123
6 P 12
3 12×11×10 17
= 所求機率為 1- 3 =1- =
30 12 12×12×12 72
1 191 範例 13

5 C 41C 13
5 33
1 所求機率為 =
2 P
(A'∩B') C 525 16660
=P((A∪B) ') C 13 4 4 11 4
2 C 2C 2C 1 C 1 198
2 所求機率為 52 =
=1-P (A∪B) C5 4165
7 C 13
1 C 4 12 4
C
3 1 C 2 6
=1- 3 所求機率為 =
10 C 525 4165
13 4 12 4 4
3 C 1 C 3C 2 C 1C 1 88
= 4 所求機率為 52 =
10 C5 4165
189 範例 11 類題

袋中共有 12 個球,任取 4 球,有 C 12 4 種取法 C 41C 13


4 44
1 所求機率為 52 =
1 恰為 1 個黑球,3 個白球的方法有 C 31C 53 種 C4 4165
C 31C 53 3×10 2 2 1(x﹐x﹐x﹐y) 三同一異,如(5﹐5﹐5﹐8)
故所求機率為 = =
C 12
4 495 33 有 C 13
1 C 4 12 4
C
3 1 C 1=2496 (種)
2 4 球均為同色的情形為 4 白球或 4 紅球 2(x﹐x﹐y﹐y) 兩同兩同,如(5﹐5﹐8﹐8)
C 54+C 44 6 2 13 4 4
有 C 2 C 2C 2=2808(種)
故所求機率為 = =
C 12
4 495 165 2496+2808 5304 24
∴所求機率為 = =
類題1 52
C4 270725 1225
袋中共有 12 個球,任取 3 球,有 C 12 3 種取法 3(x﹐y﹐z﹐u)四異
3 4 5
C 1C 1C 1 60 3 如(3﹐5﹐8﹐9),且花色各異
1 所求機率為 = =
C 12
3 220 11 C 13
4 ×4! 264
所求機率為 =
C 33+C 43+C 53 15 3 C452
4165
2 所求機率為 12 = =
C3 220 44 192 範例 14
2 黑球
3 黑球 甲、乙各有 10 種選法,而乙的數字與甲相同
C 52C 71+C 53 80 4 10×1 1
3 所求機率為 12 = = 故所求機率為 =
C3 220 11 10×10 10
互動式教學講義.數學(2) 解答篇 51
頁碼 頁碼
類題1 範例 16
5
甲任取 2 數的方法有 C =10 種 2
5
點數 1 2 3 4 5 6
乙任取 2 數的方法有 C =10 種 2
1 1 1 1 1 1
∴樣本空間 (n S)=C 52C =100 種
5 機率
2
6 6 6 6 6 6
又兩人所選數字完全相同之方法有 C 52×1=10 種
1 1 1 1 1
C 52×1 10 1 期望值 E=1× +2× +3× +4× +5×
故所求機率為 5 5 = = 6 6 6 6 6
C 2C 2 100 10
1
類題2 +6×
6
1 甲任取 3 數之方法數為 C 100 3 ,
21
乙任取 3 數之方法數為 C 100 = =3.5(元)
3
100 100
6
∴樣本空間 (n S)=C 3 C 3 類題1
又兩人所選的數完全相同之方法數為
乙與甲所取者均一致 點數和 2 3 4 5 6 7
C 100
3 ×1 1 2 3 4 5 6
故所選之數完全相同的機率為 機率
36 36 36 36 36 36
C 100
3 ×1 1 1 點數和 8 9 10 11 12
100 100 = 100 =
C3 C3 C3 161700 5 4 3 2 1
2 至少有一數相同的機率為 機率
36 36 36 36 36
1-P(三數相異) 1 2 3 4
乙與甲所取者均相異 期望值 E=2× +3× +4× +5×
C C100 97 36 36 36 36
3 3
=1- 100 100 5 6 5 4
C C3 3 +6× +7× +8× +9×
7372 713 36 36 36 36
=1- = 3 2 1
8085 8085 +10× +11× +12×
36 36 36
193 範例 15
3 2 252
期望值 E=50× +100× =70(元) = =7(元)
5 5 36
故選A 195 類題2

類題1 4 4
取 1 枚代幣的期望值為 ×10+ ×50=30(元)
8 8
金額 1810 2 -6
∴取 4 枚代幣總額的期望值 E=30×4=120(元)
(元) (三正)(兩正一反)(一正兩反) (三反)
範例 17
1 3 3 1
機率 存活 死亡
8 8 8 8
5000-400 5000-400-500000
1 3 3 1 利潤(元)
∴期望值 E=18× +10× +2× +(-6)× =4600 =-495400
8 8 8 8
機率 0.995 0.005
48 故所求期望值 E=4600×0.995+(-495400)×0.005
= =6(元)
8 =4577-2477
〔另解〕 =2100(元)
利用期望值=平均值 類題
1 1
∴期望值 E= 6× +(-2)× ×3=6(元) 建大廠之期望利潤為
2 2
50×0.3+10×0.1+5×0.4+(-30)×0.2
194 類題2
=12(百萬元)
設應置入 x 顆其他顏色的球
建中廠之期望利潤為
2 5
則 2000× +1000× =300 40×0.3+30×0.1+10×0.4+(-10)×0.2
7+x 7+x
=17(百萬元)
4000 5000 9000
c + =300 c =300 建小廠之期望利潤為
7+x 7+x 7+x
30×0.3+20×0.1+5×0.4+(-2)×0.2
c 7+x=30
=12.6(百萬元)
c x=23
故選建中廠獲利最佳,其利潤期望值是 17 百萬元
故應置入 23 顆其他顏色的球
52 互動式教學講義.數學(2) 解答篇
頁碼 頁碼
196 範例 18 198 類題2

1 所求期望值為 C 32 3
1 C,D,E 三選項猜對 2 個的機率為 3 =
300×0.1+200×0.2+100×0.3+0×0.4 2 8
=100
(元),故選D C 33 1
2 C,D,E 三選項猜對 3 個的機率為 3 =
2 因抽獎一次需花 120 元,而期望值只有 100 元, 2 8
故對抽獎者而言不利 3 1
故期望值為 ×2.5+ ×5
類題1 8 8
1 購買此彩券一張獎金期望值為 15 10
= +
2 8 10 16 16
50000× +10000× +1000×
2000 2000 2000 25
= (分)
=95(元) 16
2 因購買一張彩券需花 100 元,而期望值只有 95 元 範例 20
顯然購買此彩券是不利的 1 對中最後一碼數字時,車主實際上只支付
類題2 500-200=300 元
設應賠 x 元 500
而原本 500 元可加 =25 公升的汽油
1 3 3 1 20
則期望值 E=10× +6× +2× -x× =0
8 8 8 8 300
∴每公升值 =12 元
c 34-x=0 ∴x=34 25
故應賠 34 元 故選C
197 範例 19 2 情況 對中最後一碼 沒對中最後一碼
1 確定答案的 16 題部分得分之期望值為 1 9
機率
16×4=64(分) 10 10
2 確定 2 個選項的 6 題部分得分之期望值為 每公升油錢 12 元 20 元
1 2 由上表可知,汽油每公升的期望值為
6× 4× +(-1) × =4(分)
3 3 1 9 192
12× +20× = =19.2(元)
3 亂猜的 3 題部分得分之期望值為 10 10 10
1 4 199 類題1
3× 4× +(-1) × =0(分)
5 5 C 21C 11 2
甲、乙兩隊在不同組的機率為 4 2 = ,
由1、2、3知得分之期望值為 64+4+0=68(分) C 2C 2 3
類題1 2!
1 五選一的單選題 1 1
( 2 )= 4
2
又甲勝且乙勝的機率為
1 4
猜對機率 ,猜錯機率
5 5 2 1 1
故所求機率為 × =
設答對得 x 分,答錯倒扣 y 分 3 4 6
1 4 類題2
則 x- y=0
5 5 15 5

c y= x
1
500×80 %-
( 2300 ×10000+ 2300 ×30000)
4 3000
=400- =400-130.43……=269.56……
1 23
故答錯應倒扣原題分的
4 故期望值 E=269.56……×1000000~269000000
2 至少有一個答案的複選題, 約 2.7 億元,故選D
共有 25-1=31 種猜答方式

猜對機率
1
,猜錯機率
30 3-4
31 31
設答對得 x 分,答錯倒扣 y 分 基礎題

1 30 1 200 1 {
(正﹐2)﹐(正﹐4)﹐(正﹐6)}
則 x- y=0 c y= x 共 3 個樣本點
31 31 30
1 2 與 A 互斥的事件有 ∅,{3},{4},{3﹐4},
故答錯應倒扣原題分的 共有 4 個
30
互動式教學講義.數學(2) 解答篇 53
頁碼 頁碼
3 6=A+5=2+4=3+3 q 三球編號之和大於 14 的情形有
C 41C 41+C 41C 41+C 42 1+6+8=15,2+6+8=16,3+6+8=17
所求機率為
C 52
2 共3種
16+16+6 19 3 3
= = 故所求機率為 5 = ,故選B
26×51 663 C3 10
4 1,2,3,4,5 由小排到大的情形只有 1 種 P 12
3 12×11×10 55
w 所求機率為 3 = =
1 1 12 12×12×12 72
故所求機率為 =
5! 120 13×12
13
5 出現正面次數機率最大的為 2 次, C2 2 1
203 e 1 52 = =
4! C2 52×51 17
2!2! 3 2
其機率為 =
24 8 C 13
1 C 4
2 13×6 1
2 52 = =
故選C C2 52×51 17
201 6 1 7=1+6=2+5=3+4 2
3×2! 6 1 C 90
2 89
故所求機率為 2 = = r 所求機率為 =
6 36 6 90×90 180
2 11=5+6,12=6+6 t 出現點數 1 2 3 4 5 6

1×2!+1 3 1 所得獎金 -1 2 -3 4 -5 6
故所求機率為 = =
62 36 12 1
故期望值為 ×(-1+2-3+4-5+6)
7 樣本空間的樣本點個數為 3,3,8,9 的排列數 6
4! 3
即 =12(種) = =0.5(元)
2! 6
而成功的情形只有 1 種 y 取 1 球時,取得紅球個數的期望值為
1 5 5
故一次就成功的機率為 ×1= (個)
12 9 9
C 41C 51 5 ∴任取 2 球,取得紅球個數的期望值為
8 A ○: =
C 92 9 5 10
×2= (個)
B ×:可能取出 (紅﹐白) 或(白﹐紅) 9 9
4 5 40 204 u 得款(元) 1000000 100000 10000 1000
∴所求機率為 × ×2=
9 9 81 1 9 90 900
4 5
C 1C 1×2! 5 機率
C ○: = 106 106 106 106
P 92 9 1 9
5 E=1000000× +100000× 6
D ○:任何時刻取到白球的機率皆為 106 10
9 90 900
E ×:白球先取完表示最後一球為紅球 +10000× 6 +1000× 6
10 10
4 =1+0.9+0.9+0.9=3.7(元)
即所求機率為
9 故預期損失 10-3.7=6.3(元)
故選ACD i 情況 三正 兩正一反 一正兩反 三反
202 9 P(A∪B)=P
(A) +P(B)-P
(A∩B) 得款(元) 8 3 1 -x
1 1 1 2 1 3 3 1
= + - = 機率
3 2 6 3 8 8 8 8
0 P(A∪B∪C) =P
(A) +P
(B) +P(C)
-P(A∩B) 1 3 3 20
-P(B∩C)-P (C∩A) 1 期望值 E= ×8+ ×3+ ×1=
8 8 8 8
+P(A∩B∩C) =2.5(元)

(A∩B∩C)f(A∩B) 2 若欲使賭局公平,則期望值為 0
而P(A∩B)
=0 設出現三個反面應賠 x 元
∴P
(A∩B∩C)
=0 1
1 1 1 1 5 則 2.5+ ×(-x)=0 c x=20
故所求為 + + -0-0- +0= 8
4 4 4 8 8 故應賠 20 元
54 互動式教學講義.數學(2) 解答篇
頁碼 頁碼
o 某甲有 25-1-C 51=32-1-5=26 種選法 E ×:1 白球為最後一球且紅球比綠球先取完
∵期望值為 0 分 5 4 5
c × =
1 25 1 12 7 21
∴ ×5- ×S=0 c S=
26 26 5 2 綠球為最後一球且紅球比白球先取完
進階題 4 5 5
c × =
205 p A ×B ○:S 共有 25 個子集, 12 8 24
即有 25=32 個事件 5 5 25
由1、2得所求機率為 + =
(A)
C ○:n =2 ∴A 事件共有 C 52=10 個 21 24 56
DE ○:S 與 ∅ 均各表一事件 故選BCD
故選BCDE 207 f 任兩點之距離如下:
a 樣本空間 n(S)
=30×30=900 1 AB=1,有 6 組
1 2 AC=a3 ,有 6 組
A ○:
(16﹐16),所求機率為
900 3 AD=2,有 3 組
B ○:(1﹐1),(2﹐2) ,(3﹐3),……, 6 6 3
∴期望值 E= 6 ×1+ 6 ×a3 + 6 ×2
30 1 C2 C2 C2
(30﹐30),所求機率為 =
900 30 6+6a3 +6 12+6a3 4+2a3
= = =
1 29 15 15 5
C ○:所求機率為 1- =
30 30 g 移動 3 次欲落在陰影區域的情形有兩類:
D ○:(1﹐24),(2﹐23),……, (24﹐1), 1 往某方向移動 2 次,往反方向移動 1 次
24 2 3!
所求機率為 = 2!× ×2
900 75 2! 3
機率為 =
E ×:(1﹐3),(2﹐4),……, (28﹐30); 43 16
(3﹐1),(4﹐2),……, (30﹐28) C 43×3! 3
2 4 個方向任取 3 個,機率為 =
28 14 43 8
所求機率為 ×2=
900 225 3 3 9
由1、2可得所求機率為 + =
故選ABCD 16 8 16
C 22 1 歷屆試題
s 所求機率為 8 =
C2 28 h 可能為一男兩女或兩男一女,則所求為
3×1×1 1 C 20
1 C2
15
C 20
2 C1
15
2100+2850 4950 90
206 d A ×:甲獨贏機率為 = + = = =
33 9 C3 35
C3 35
6545 6545 119
C 21×3×1 2 208 j 設半糖且去冰的有 x 人
甲與另一人同贏機率為 3 =
3 9 xN28
c c 15NxN28
1 2 1 37+(28-x)N50
∴甲贏的機率為 + =
9 9 3 15 x 28
c N N
35×34×33×32×31 50 50 50
1 35
C 1C 5 1×2×3×4×5 x
B ○: = c 0.3N N0.56
C 36
6 36×35×34×33×32×31 50
1×2×3×4×5×6 故選BC
6 1 k 跳 4 步後恰回到原點有
= =
36 6 “右右左左”,“上上下下”,“上下左右”三
C ○:1-P(沒出現 6 點) 種類型
53 125 91 4! 4!
=1- 3 =1- = 共有 + +4!=6+6+24=36 種情形
6 216 216 2!2! 2!2!
P 40
2 40×39 36 9
D ○:p1= 90 = , 故所求機率為 4 =
P2 90×89 4 64
40×39 l 所求機率為 1-P(兩人休假皆不同天)
C 40
2 2 40×39 C 72C 52 10 11
p2= 90 = = ∴p1=p2 =1- 7 7 =1- =
C2 90×89 90×89 C 2C 2 21 21
2 故選E
互動式教學講義.數學(2) 解答篇 55
頁碼 頁碼
209 ; A ×:和大於 10 者有 7+6,7+5,7+4,6+5

故所求機率為
4
7 =
4
第4章
C2 21
B ×:和小於 5 者有 1+2,1+3
4-1 直角三角形的邊角關係
2 2
故所求機率為 7 = 212 範例 1
1 a2
C2 21
C ○:1,2,3,4,5,6,7 中有 4 個奇數、 +
sin 30n+cos 45n 2 2 1+a2
1 a2
3 個偶數 = =
∴和為奇數必定 1 奇 1 偶 sin 30n-cos 45n 1-a2

C 41C 31 12 4 2 2
故所求機率為 7 = = (1+a2 ) 2
C2 21 7 =
D ×:差為偶數有(奇-奇) 與(偶-偶)共 2 種 (1-a2 ) (1+a2 )
C 2+C 32
4
6+3 9 3+2a2
故所求機率為 = = = =-3-2a2
7
C2 21 21 -1
213 類題1

6 a3
3

7 1 ∵cos 30n= = c x=4a3
E ○:積為奇數必為 (奇×奇) x 2
y 1
6 a3
C 42 6 2 ∵tan 30n= = c y=2a3
故所求機率為 = =
C 72 21 7
故選CE 4
2 ∵tan 45n= =1 c x=4
x
4 a2
z 取出三數最小為 1,2,3,乘積 1×2×3=6
最出三數最大為 7,8,9,乘積 7×8×9=504 ∵sin 45n= = c y=4a2
而 6<32<222<504,完全平方數可能有 y 2
42=1×2×8,62=1×4×9=2×3×6, 4 1
3 ∵cos 60n= = c x=8
82=2×4×8,122=2×8×9=3×6×8 x 2
共 6 種可能 y
∵tan 60n= =a3 c y=4a3
6 6 1 4
故所求機率為 9 = = 類題2
C3 84 14
sin230n+cos260n+tan 60ncos 30n
a3
210 x 每次「十連抽」抽到金卡張數的期望值為
1 2 1 2
0.02×9+0.1×1=0.18+0.1=0.28(張)
23000

( )( )
2

2
+a3 ×
2
又 =15.33…… 1 1 3
1500 = + + =2
最多可進行 15 次「十連抽」 4 4 2
∴15 次「十連抽」抽到金卡張數的期望值為 範例 2
0.28×15=4.2(張) ∵△ABC 是直角三角形且∠C=90n

c AB=al32+42 =5,如右圖
c 設藍色球有 x 顆 又 AC=3,BC=4
C x2 1 x x-1)

則 10 = c C x2=3 c =3 BC 4
C2 15 2 ∴sin A= = ,
c x2-x-6=0 c(x-3) (x+2) =0 AB 5
c x=3 或-2(不合) ∴x=3 AC 3 BC 4
cos A= = ,tan A= =
又設綠色球有 y 顆 AB 5 AC 3
214 類題
C y2 2 y y-1)

則 10 = c C y2=10 c =10 如右圖,作直角三角形 ABC
C2 9 2

則 BC=al132-52 =12
c y2-y-20=0 c(y-5)(y+4) =0 斜邊與一股分別為 13 與 5
c y=5 或-4(不合) ∴y=5
∴黃色球有 10-3-5=2(顆) 12 12
∴sin A= ,tan A=
則取兩球為相異顏色的機率為 13 5
C 31C 51+C 51C 21+C 21C 31 15+10+6 31 5 5
= = sin B= ,tan B=
C 10 45 45 13 12
2
56 互動式教學講義.數學(2) 解答篇
頁碼 頁碼
範例 3 範例 5
24 BD ∵圓 O 為單位圓,且 OA=OF=OC=1
1 在△ABD 中,sin A= = c BD=24
25 25 又∠ODC=∠DOE=q(內錯角)
2 在△BCD 中 AB
A ○:AB= =tan q
24 12 OA
∵tan C= = c CD=10

又 AD=al252-242 =7
CD 5 OE
B ×:OE= =cos q
OF
∴AC=AD+CD=7+10=17 CD 1
C ×:CD= =
215 類題 OC tan q
1 在△ACD 中 ∵∠ADC=90n EF
D ○:EF= =sin q
CD CD 3 OF
∴sin A= = = c CD=12

2 由畢氏定理知 AD=al202-122 =16


AC 20 5 OD 1
E ×:OD= =
OC sin q
3 在△ABC 中 故選AD
3 217 類題
∵∠ACB=90n,且 sin A=
5 AT 3
AT = =tan q = ,
4 OA 4
c cos A=
5 作一示意圖如右
AC 20 4 PQ 3
∴cos A= = = 1 PQ= =sin q =
AB AB 5 OP 5
c AB=25 OQ 4 1
2 QA=OA-OQ=1- =1-cos q =1- =

∴BC=al252-202 =15
4 在△ABC 中,AB=25,AC=20 OP 5 5
218 範例 6
CD 12 4 1 由平方關係 sin2q+cos2q =1 可得
在△BCD 中,cos∠BCD= = =
BC 15 5 15 2 64
範例 4
sin2q =1-cos2q =1-
17

( )
289

∴AD=2,BD=al12+ =al8+4a3
在△ABC 中,BC=1,則 AC=a3 ,AB=2 8 8
c sin q =± (負不合)c sin q =

=al8+2s12 =a6 +a2


2
(2+a3 ) 17 17
2 由商數關係知
BC 1 a6 -a2 8
a6 +a2
1 sin 15n= = =
BD 4 sin q 17 8
a6 +a2
tan q = = =
CD 2+a3 cos q 15 15
a6 +a2
2 cos 15n= = =
BD 4 17
BC 1 類題
3 tan 15n= = =2-a3

c cos q =±al1-a2(負不合)
CD 2+a3 1 cos2q =1-sin2q =1-a2
216 類題

al1-a2
如右圖 sin q a
2 tan q = =
∵AB=3,BC=4 cos q
∴AC=5 219 範例 7
又 AC=CD ∵sin q =2 cos q
q sin q
故∠ADB= c =2 c tan q =2

∴斜邊長 AD=al92+32 =s90 =3s10


2 cos q
作圖如右,
q 3 s10 2 1
a5 a5
故1 sin = = 得 sin q = ,cos q =
2 3s10 10
q 9 3s10 2 1
a5 a5
2 cos = = ∴sin q+cos q = +
2 3s10 10
q 3 1 3 3a5
a5
3 tan = = = =
2 9 3 5
互動式教學講義.數學(2) 解答篇 57
頁碼 頁碼
類題1 2(sin q-cos q)2=1-2 sin q cos q
sin q cos q 9 7
2× - =1-=
2 sin q-cos q cos q cos q 16 16
a7
=
3 sin q+cos q sin q cos q
3× + c sin q-cos q =±
cos q cos q 4
3 3
2 tan q-1 2×2-1 3 3 sin q+cos q
= = =
3 tan q+1 3×2+1 7 =(sin q+cos q)(sin2q-sin q cos q+cos2q)
類題2 5 9 5 23 115

∵cos q =tan q c cos q =


sin q
c cos2q =sin q

4 (
× 1-
32
= ×
4 ) =
32 128
cos q 類題1

又 sin2q+cos2q =1 c sin2q+sin q-1=0 1


1 sin q-cos q =
-1±a5 2
c sin q = (負不合)
2 兩邊平方得
-1+a5 1
∴sin q = sin2q-2 sin q cos q+cos2q =
2 4
220 範例 8 3
c sin q cos q =
1 cos 40n=cos(90n-50n)=sin 50n ∴q 為 50n 8
2(cos240n+cos250n)+cos260n 2 ∵(sin q+cos q)2
=(sin q-cos q) 2
+4 sin q cos q
=(sin250n+cos250n)+cos260n 1 3 7
c(sin q+cos q)2= +4× =
1 2 5 4 8 4
=1+
2 ()

4 a7
c sin q+cos q =± (負不合)
3 sin210n+sin220n+sin230n+sin240n+sin250n 2
+sin260n+sin270n+sin280n a7
∴sin q+cos q =
=(sin210n+sin280n)+ (sin220n+sin270n) 2
2 2
+(sin 30n+sin 60n)+(sin240n+sin250n) ∴sin 3
q+cos3
q
=(sin210n+cos210n)+ (sin220n+cos220n) =(sin q+cos q) (sin2q-sin q cos q+cos2q)
+(sin230n+cos230n)+(sin240n+cos240n) a7 3 5a7
=1+1+1+1=4

2 (
× 1- =
8 16 )
類題1 222 類題2
2 2
(sin 17n+cos 17n)+(sin 73n-cos 73n) 1
a3
∵sin q-cos q =
=sin 17n+cos 17n+2 sin 17n cos 17n+sin273n
2 2

+cos273n-2 sin 73n cos 73n 1


兩邊平方得 sin2q-2 sin q cos q+cos2q =
=sin217n+cos217n+2 sin 17n cos 17n+sin273n 3
+cos273n-2 cos 17n sin 17n 1 1
c 1-2 sin q cos q = c sin q cos q =
=(sin217n+cos217n)+ (sin273n+cos273n) 3 3
2
=1+1=2 又 sin 及 cos 為 2x +px+q=0 的兩解
q q
類題2 c 2x2+px+q=2(x-sin q)(x-cos q)
2
cos 63n sin 27n =2〔x -(sin q+cos q)x+sin q cos q〕
= =tan 27n ∴q 為 27n
sin 63n cos 27n p ................
sin q+cos q =- 1
221 範例 9 2
可知
5 q .......................
1 ∵sin q+cos q = ,兩邊平方得 sin q cos q = 2
4 2
2
(sin q+cos q) =sin2q+cos2q+2 sin q cos q q 1 2
由2得 = c q=
5 2 2 3 3
c
()4
=1+2 sin q cos q 2
由1知(sin q+cos q)=(sin q-cos q)
2
2
+4 sin q cos q
25 9 p 1 4 20
( )( )
2

a3
c 2 sin q cos q = -1= ∴ - = + c p2=
16 16 2 3 3
9 20 16 4
c sin q cos q = ∴p2-8q= - =
32 3 3 3
58 互動式教學講義.數學(2) 解答篇
頁碼 頁碼
類題3
範例 12
2
由題意知 sin q 及 cos q 為 25x +kx+12=0 的兩解 1 sin 65n>sin 55n
c 25x2+kx+12=25(x-sin q) (x-cos q) 2 cos 65n<cos 55n
=25〔x2-(sin q+cos q)x+sin q cos q〕 3 sin 46n>sin 44n=cos 46n
12 k 1
可知 sin q cos q = ,sin q+cos q =- 4 sin 35n>sin 30n=
25 25 2
2 49 a3

(sin q+cos q) =1+2 sin q cos q = 5 cos 35n<cos 30n=
25 2
7 k 7 6 tan 46n>tan 45n=1
c sin q+cos q = ,故- = ,得 k=-35
5 25 5 類題

223 範例 10 A ×:當 0n<q <45n時,sin q <cos q


1(sin q+cos q)2=sin2q+cos2q+2 sin q cos q ∴sin 37n-cos 37n<0
=1+2 sin q cos q,故得證 B ○:1=tan 45n
2
1+sin q cos q (1+sin q) +cos2q ∴tan 53n>tan 45n=1 ∴tan 53n-1>0
a3
2 + =
cos q 1+sin q cos q(1+sin q)
2 2 C ○: =cos 30n
1+2 sin q+sin q+cos q 2+2 sin q 2
a3
= =
cos q(1+sin q) cos q(1+sin q)
∴cos 25n- =cos 25n-cos 30n>0
2 1+sin q)
( 2 2
= = ,故得證
cos q(1+sin q) cos q D ×:sin 61n<1,而 tan 61n>tan 45n=1
1 sin q cos q sin2q+cos2q ∴sin 61n-tan 61n<0
3 tan q+ = + =
tan q cos q sin q sin q cos q 1
E ○: =cos 60n
1 2
= ,故得證
sin q cos q 1
∴ -cos 61n=cos 60n-cos 61n>0
類題 2
1 左式=(sin2q+cos2q) 2
-2 sin2q cos2q 故選BCE
=1-2 sin2q cos2q =右式 226 範例 13
sin2q 2 sin2q-sin2q cos2q
2 左式= -sin q =
cos2q cos2q
sin q(1-cos q) sin q×sin2q
2 2 2

= =
cos2q cos2q 如上圖,令機場中心位於 O 點
=tan2q sin2q =右式 設大樓蓋 n 層樓,則大樓高度 AB=3n 公尺
224 範例 11 ∵OA=3 公里=3000 公尺
如右圖,設 A 表小孩的位置, AB
則 tan q = <tan 8n~0.1405
C 表風箏的位置, OA
且風箏高度 BC=h 公尺 c 3n<0.1405×3000
則 h=50×sin 42n c n<140.5,即 nN140,故最多可蓋 140 層
~50×0.6691 類題

~33.5 如右圖,令竹竿長為 AB=6 公尺,


故風箏的高度為 33.5 公尺 牆高 CD=4 公尺,
225 類題 ∠BAE=∠DAC=60n
如右圖 在△ABE 中,AB=6
設 A 表小華的位置,B、C 分別代 1
c AE=AB cos 60n=6× =3
表樹底及樹頂, 2
且樹頂到地面的高度為 h 公尺 在△ADC 中,CD=4
h CD 4 4a3
a3
則 tan 51n= c AC= = =
10 tan 60n 3
c h=10 tan 51n 故所求影長 CE=AE-AC
~10×1.2349 4a3
=3- (~0.69)
(公尺)
~12.3(公尺) 3
互動式教學講義.數學(2) 解答篇 59
頁碼 頁碼
1
229 8 將 cos q =sin q- 代入 cos2q+sin2q =1
4-1 3
1
( )+sin q =1
2
基礎題 c sin q- 2

3
227 1 如右圖,
2 8
AC AC 3 c 2 sin2q- sin q- =0
cos A= = = 3 9
AB 20 5
c 9 sin2
q-3 sin q-4=0
3±al9-4×9×(-4)
BC=alAB -AC
∴AC=12
2 2 1±s17
∴sin q = =
2×9 6
=k400-144
∵sin q >0
=d256 =16
1+s17
故選D ∴sin q =
6
2 在直角△ACD 中,
9 設共需造 n 階,則總高度為 18n 公分
CD
sin∠CAD=cos∠ACB= 且 18nMAB
AC
c 18nMAC×tan 30n
a3
又∠ABC=∠CAD 且∠BAD=∠ACB
CD c 18nM600×
∴sin∠ABC=cos∠BAD=cos∠ACB= 3
AC
100a3
故選ABDE c nM ~19.22
9

al3 -22
AE 2
3 tan B=tan∠ADE= = 2 故需造 20 階
DE
進階題
2 2a5
a5
= = DE
5 0 A ×: =tan A
AD
sin q+5 cos q
228 4 1
2 sin q-3 cos q c AD=DE×
tan A
sin q 1
+5 +5 3
cos q tan q+5 4 =
= = = tan A
sin q 2 tan q-3 1
2× -3 2× -3 B ○:BF=BC-CF
cos q 4
=AB×sin A-3
1+20 21
= =- =5 sin A-3
2-12 10
C ○:∵DE=3
5 sin 25n+sin 65n-tan260n
2 2

∴AE>3 c BE<5-3=2
=sin225n+sin2(90n-25n)- 2
(a3 )
又 EF<BE
=sin225n+cos225n-3
∴EF<2
=1-3=-2
D ×:△ACE 面積為
sin 70n
6 tan 70nsin 20n= ×sin 20n 1 1 15
cos 70n ×AC×3= ×5 cos A×3= cos A
2 2 2
sin 70n
= ×cos 70n E ○:△ABC 面積為
cos 70n
1 1
=sin 70n ×AC×CB= ×5 cos A×5 sin A
2 2
又 sin 70n=cos 20n
25
故選AD = sin Acos A
2
7 ∵0n<q <45n
故選BCE
a3
∴sin q <cos q
(sin q-cos q)2=sin2q-2 sin q cos q+cos2q 230 q sin 85n>sin 60n=
2
1 3
=1-2× = cos 3n=sin 87n>sin 85n
a3 a3
8 4
a3 tan 29n<tan 30n= <
∴sin q-cos q =- 3 2
2
故 a>b>c
60 互動式教學講義.數學(2) 解答篇
頁碼 頁碼
w 設 BM =CM =x 歷屆試題
則 BH =x+5,CH =x-5 8 y
tan a= =
4 9 2-x
∵在△ABH 中,cos B= y 作 CH⊥AB,設 C
(x﹐y)
,則
5 8 y
tan b= =
3 3 x+4
∴tan B=
4
3
∴AH =BH ×tan B=(x+5)×
4
1
a5
又在△ACH 中,cos C=

∴tan C=2 c AH =2(x-5)


3 9y=16-8x 5
得 (x+5)=2(x-5)c x=11 c c(x﹐y)= - ﹐4
( )
(11-5)=12,即 AM =al122+52 =13
4 3y=8x+32 2
∴AH =2 5 5
e ∵∠A+∠B=90n ( ) (
∴C - ﹐4 ,又 D ﹐-8

∴CD=al52+122 =13
2 2
∴cos B=sin A
即 2 cos A+3 sin A=3 c 2 cos A=3(1-sin A) 232 u 在△ODC 中,
兩邊平方得 4 cos2A=9 a3
2
(1-sin A)
c 4-4 sin2A=9-18 sin A+9 sin2A OC=8×cos 30n=8× =4a3
2
c 13 sin2A-18 sin A+5=0 在△OBC 中,OB=OC×cos 15n
c(13 sin A-5) (sin A-1)=0 在△OAB 中,
5 AB=OB×sin 15n=OC×cos 15n×sin 15n
c sin A= 或 sin A=1(不合)
13 a6 +a2 a6 -a2
5 12 =4a3 × × =a3
∴sin A= ,得 cos A= 4 4
13 13 故選D
5 i 四邊形 ABCD 為正方形
sin A 13 5 5
故 tan A= = = 且 tan∠AEB=
cos A 12 12 2a6
13 令 AB=5
231 r 依題意作圖如右
c CE=alBC +BE =al52+(2a6 )
c BE=2a6 ,BC=5,CD=5
取 AC=3,BC=5 2 2 2
=7
則 BD=2,CD=3
> > 在△CED 中 ∵∠ECD=90n
延長 AD 並作 BE⊥AD 於 E 5
∵∠ADC=45n ∴tan∠CED=
7
∴∠BDE=45n o 設∠COD=q
c DE=a2 ,BE=a2
∴CD=al262-242 =10
∵OC=26,OD=24
而 AD=3a2
∴△ABE 中,
a2
10 5
BE 1 c sin q = =
tan q = = = 26 13
AE 3a2 +a2 4 5 120
t 設 OP=h 公尺, ∴AB=24 sin q =24× =
13 13
則 OA=h, 233 p 設 BC=x
OB=a3 h, 由右圖可知∠ABC=62n
OC=2h c∠C=28n,在△ABC 中
1 h-20 2.85
tan q = = sin 28n= ~0.4695
3 2h x
c 2h=3h-60 2.85
c h=60 c x~ ~6.07~6.1
0.4695
故大樓高度 OP 為 60 公尺 故迴轉半徑 BC 為 6.1 公尺
互動式教學講義.數學(2) 解答篇 61
頁碼 頁碼
a 設梯長 AB=x 公分 235 f 取 OP 中點 M
1
則 BC=AB cos 60n= x
2
3
又 sin q =0.6=
5
4
∴cos q =
5
4 ∵OA=AP ∴AM⊥OP
c CF=EF cos q = x
5 可得 OP=2OM=2 cos q
4 1 故選D
∴ x=51+ x
5 2 g 如右圖
3 150
c x=51 c x=170(公分) =cos 22n
10 繩長
234 s ∵△ADE 面積 150
c繩長=
9 cos 22n
= ×△ABC 面積
16 故選D
3
∴DE= BC
4
3 4-2 廣義角與極坐標
= ×16=12
4 237 範例 1
1 1 ∵980n=260n+360n×2
h=BC×sin 30°=16× =8
2 ∴所求同界角 q 為 260n
1 2 ∵-700n=20n+360n×(-2)
梯形 DBCE 面積為 (12+16)×8
2 ∴所求同界角 q 為 20n
=112(平方公尺) 類題
故△ABE 面積 設 q =490n+360n×n,n 為整數
=△ADE 面積+△BDE 面積 則-1900n<490n+360n×n<1000n
9 1 c-6.6……<n<1.4……
= ×梯形 DBCE 面積+ ×DE×h
7 2 ∴n 可為-6,-5,-4,-3,-2,-1,0,1,
9 1 共8個
= ×112+ ×12×8=192(平方公尺)
7 2
範例 2
d ∵OC⊥DE A ×:2115n-45n=2070n不是 360n的整數倍
∴∠OCE=∠OCD ∴不是 45n的同界角
=90n B ○:-2115n-45n=-2160n是 360n的整數倍
又∠DOE=90n ∴是 45n的同界角
∴∠DOC=∠OEC=q C ×:1215n-45n=1170n不是 360n的整數倍
且 OA=OB=OC=1 ∴不是 45n的同界角
OC D ×:-1215n-45n=-1260n不是 360n的整數倍
A ×: =sin q,
OE ∴不是 45n的同界角
OC 1 E ○:405n-45n=360n是 360n的整數倍
OE= =
sin q sin q ∴是 45n的同界角
B ×:OC=1 故選BE
OC OC 1 238 類題1
C ×: =cos q,OD= =
OD cos q cos q A ○:-333n-27n=-360n是 360n的整數倍
OC OC 1 ∴是 27n的同界角
D ×: =tan q,CE= =
CE tan q tan q B ×:927n-27n=900n不是 360n的整數倍
CD ∴不是 27n的同界角
E ○: =tan q,CD=OC tan q =tan q
OC C ○:-1773n-27n=-1800n是 360n的整數倍
故選E ∴是 27n的同界角
62 互動式教學講義.數學(2) 解答篇
頁碼 頁碼
D ×:1773n-27n=1746n不是 360n的整數倍 類題

∴不是 27n的同界角 1 由定義作圖如下:


E ○:1467n-27n=1440n是 360n的整數倍
∴是 27n的同界角
故選ACE
類題2

793n=73n+360n×2
∴73n是 793n的最小正同界角
又 73n-360n=-287n -1
tan 225n= =1,
∴-287n是 793n的最大負同界角 -1
故數對 (a﹐b)=(73n﹐-287n) -a3
cos 150n= ,

取 r=OP=al(-12)
239 範例 3 2
2
+52 -1
sin 330n=
=13 2
y 5 a3 -1
∴sin q = =
r 13
故所求為 1- -
( )( )
2

2
x -12 1 a3
cos q = = = +
r 13 2 2
12 2 ∵sin 0n=0,cos 270n=0,cos 180n=-1,
=-
13 tan 180n=0,sin 270n=-1
y 5 5 ∴所求為 0+0+(-1)-0×(-1)=-1
tan q = = =-
x -12 12 範例 5
類題 ∵P(sin q﹐cos q)位於第四象限
y y c sin q >0 且 cos q <0
∵tan q = = =2
x -5 c q 為第二象限角
∴y=-10 c tan q <0
cP(-5﹐-10) ∴Q(cos q﹐tan q)位於第三象限
241 類題1
=al(-5)
取 r=OP
2 2
+(-10) 1 ∵sin q <0,-cos q <0
=5a5 c sin q <0,cos q >0
y -10 ∴q 為第四象限角
∴sin q = =
r 5a5 2 由 tan q >0 知 q 可能為第一或第三象限角,
2 而 cos q <0 知 q 為第二或第三象限角,
a5
=-
得 q 為第三象限角
2a5 ∴sin q <0,又 sin q +cos q <0
=-
5 ∴Q(sin q﹐sin q +cos q)在第三象限
x -5 1 a5 類題2

a5
cos q = = =- =-
r 5a5 5 ∵180n<q <270n

故 alsin2q +al(1+sin q)2


240 範例 4 ∴-1<sin q <0 c 1+sin q >0
在 q =210n的終邊上取一點

OP=al(-a3 )
P
(-a3﹐-1) =|sin q|+|1+sin q|
2 2
+ (-1) =-sin q +1+sin q =1
=2 範例 6
-1 1 作示意圖如右得
∴sin 210n= =-
2 2 4
a3
sin q =- ,
-a3 5
cos 210n= =-
2 2 4
a3
tan q =-
-1 1 3
a3
tan 210n= = =
-a3 3
互動式教學講義.數學(2) 解答篇 63
頁碼 頁碼
類題 類題3

∵cos q <0 且 tan q <0 a=tan 500n=tan


(500n-360n)=tan 140n
∴q 為第二象限角 =tan(180n-40n)=-tan 40n
作示意圖如右得 b=sin 108n=sin(180n-72n)=sin 72n
3 4 c=cos 1040n=cos(1040n-720n)=cos 320n
sin q = ,cos q =-
5 5 =cos(360n-40n)=cos 40n=sin 50n
4 3 ∵sin 72n>sin 50n>-tan 40n ∴b>c>a

5 5 範例 8
∴所求為 +
3 4 1
( ) ( )
1- -
4
2- -
5
1 sin(-150n)=-sin 150n=-sin 30n=-
2
4 3 tan 225n=tan
(180n+45n)=tan 45n=1

5 5 1
= + cos
(-300n)=cos(-300n+360n)=cos 60n=
7 14 2
4 5 1
a2
sin 135n=sin 45n=
16 3 17
=- + =-
35 14 70 1
a2
cos 315n=cos(270n+45n)=sin 45n=
243 範例 7
1 sin 150n=sin(180n-30n) ∴所求為
1 1 1 1 1 1 1 3
=sin 30n=
2 ( )- ×1× -
2 2 a2
×
a2
=- - =-
4 2 4
2 cos 225n=cos(180n+45n) 2 cos(-720n)=cos(-720n+720n)=cos 0n=1
a2 ∴所求為 1+0-0+1=2
=-cos 45n=-
2 245 類題1

3 tan
(-240n) =-tan 240n=-tan
(180n+60n) sin(-1560n)=sin(-1560n+1800n)
=-tan 60n=-a3 a3
=sin 240n=-sin 60n=-
類題1 2
1 sin 1380n=sin(1380n-1080n)=sin 300n tan(-510n)=tan(-510n+720n)
=sin(270n+30n)=-cos 30n a3
a3
=tan 210n=tan 30n=
3
=-
2 1
cos(-240n)=cos 240n=-cos 60n=-
2 cos 1200n=cos(1200n-1080n)=cos 120n 2
=cos(90n+30n) a3 a3 1 1 1

=-sin 30n=-
1
∴所求為 -
(2
×
3 )
+ - =- - =-1
2 2 2 ( )
2 類題2

3 sin
(-930n)=sin(-930n+1080n)=sin 150n -sin q -tan q cos q
原式化為 - +
=sin(180n-30n) sin q tan q -cos q
1 =-1+1+(-1)=-1
=sin 30n=
2 範例 9
4 tan
(-570n)=tan(-570n+720n)=tan 150n 作示意圖如右
=tan(180n-30n)=-tan 30n 1
a3
1 sin(180n-q)=sin q =
1 3
a3
=- =-
3 2 cos(180n+q)=-cos q
244 類題2 2a2
=-
∵tan 570n=tan(570n-360n)=tan 210n 3
=tan(180n+30n)=tan 30n 1 a2
3 tan(180n+q)=tan q = =
又 tan
(-150n)=-tan 150n 2a2 4
=-tan(180n-30n) 4 cos
(q -270n)=cos(q -270n+360n)
=tan 30n 1
=cos(q +90n)=-sin q =-
故選ACE 3
64 互動式教學講義.數學(2) 解答篇
頁碼 頁碼
246 類題 範例 12
如右圖,在 q 的終邊上取一點 P, 1 ∵(sin q +cos q)2=sin2q +2 sin q cos q +cos2q

則 y=al32-22 =a5
使得 OP=3,P 點的坐標為 (2﹐y) =1+2 sin q cos q
289
∴ =1+2 sin q cos q
∴P
(2﹐a5 ) 625
a5 168
(180n+q)
∴tan =tan q = c sin q cos q =-
2 625
範例 10 2 ∵(sin q -cos q)2=sin2q -2 sin q cos q +cos2q
2 =1-2 sin q cos q
∵tan q =- ,q 為第二象限角,
3 168 961
作示意圖如右
∴(sin q -cos q)2=1-2 -
( 625)=
625

且 OP=al(-3)+22 =s13
在 q 終邊取一點 P (-3﹐2) 31
2 c sin q -cos q =±
25
2 -3 但 90n<q <180n
s13 s13
由右圖得 sin q = ,cos q =
∴sin q >0,cos q <0 得 sin q -cos q >0
∴所求為 3 (-sin q) (-sin q)-2 cos q(-cos q) 31
2 2 故 sin q -cos q =
=3 sin q +2 cos q 25
4 9 30 248 類題1
=3× +2× =
13 13 13 1(sin q +cos q)2=sin2q +2 sin q cos q +cos2q
類題 12 25
=1+ =
cos
(q -450n)=cos((q -450n)+360n×2) 13 13
=cos(270n+q)=sin q 5
s13
c sin q +cos q =± ,但 180n<q <225n
-2a2
又 cos q =
3 ∴sin q <0,cos q <0 得 sin q +cos q <0
且 90n<q <180n 5 5s13
s13
故 sin q +cos q =- =-
∴sin q =zx1-
-2a2 1 13
( )
2
= 2
3 3 2(sin q -cos q)=1-2 sin q cos q
1 12 1
故所求為 =1- =
3 13 13
247 範例 11 ∵180n<q <225n
cos 130n=cos
(90n+40n) ∴sin q >cos q
=-sin 40n=k 1 s13
s13
c sin q -cos q = =
c sin 40n=-k(其中 k<0), 13
作示意圖如右 類題2

1 sin(-220n)=sin 140n 2 sin q +1=cos q c 2 sin q =cos q -1


=sin(180n-40n) 兩邊平方得 4 sin2q =cos2q -2 cos q +1
=sin 40n=-k c 4(1-cos2q)=cos2q -2 cos q +1
2 tan 230n=tan(180n+50n) c 5 cos2q -2 cos q -3=0
c(5 cos q +3)
al1-k2 al1-k2
=tan 50n (cos q -1)=0
3
= =- c cos q =- 或1
-k k 5
類題 ∵q 是第三象限角
sin 100n=k c sin(180n-80n)=k 3 4
∴cos q =- ,tan q =
c sin 80n=k 5 3
作示意圖如右 249 範例 13
∴tan 260n=tan(180n+80n) 1 a3 x-y+2=0
=tan 80n c y=a3 x+2

al1-k2
k ∴直線的斜率 tan q =a3

c q =60n
互動式教學講義.數學(2) 解答篇 65
頁碼 頁碼
2 L1:y=a3 x+2,L2:y=-x+2 251 範例 15
1 設 A 點的極坐標為 [r﹐q ]
則 r=al(-5) 2
+02 =5
-5
又 cos q = =-1,
5
0
sin q = =0
5
設 L1、L2 的斜角分別為 q1、q2 ∴q =180n
則 tan q1=a3 且 tan q2=-1 故 A 點的極坐標為 [5﹐180n]
2 設 B 點的極坐標為 [r﹐q ]
則 r=al(-3)
由1得 q1=60n,又 q2=-45n
2 2
∴L1 與 L2 的夾角為 +(-3)
q1-q2=60n-(-45n)=105n =3a2
或 180n-105n=75n -3
又 cos q =
故 L1 與 L2 的銳夾角為 75n 3a2
類題 1
a2
=- ,
設 L1、L2 的斜角分別為
q1、q2 -3
sin q =
1 3a2
a3
則 tan q1=
1
a2
=-
且 tan q2=1
∴q1=30n且 q2=45n ∴q =225n
故 L1 與 L2 的銳夾角為 45n-30n=15n 故 B 點的極坐標為 [3a2 ﹐225n]
250 範例 14 類題

1 設 P 點的極坐標為 [r﹐q ],
則 r=al02+(-4)
1 設 A 點的直角坐標為
2
(x﹐y) ,如右圖 =4
∵r=4,q =300n 0
又 cos q = =0,
∴x=r cos q =4 cos 300n 4
=2 -4
sin q = =-1
y=r sin q =4 sin 300n 4
=-2a3 ∴q =270n
∴A 點的直角坐標為(2﹐-2a3 ) 故 P 點的極坐標為 [4﹐270n]
2 設 Q 點的極坐標為 [r﹐q ],
則 r=al(-a2 )
2 設 B 點的直角坐標為(x﹐y),
2 2
如右圖 +(a6 )
∵r=2,q =270n =2a2
∴x=r cos q =2 cos 270n=0 -a2 1
又 cos q = =- ,
y=r sin q =2 sin 270n=-2 2a2 2
∴B 點的直角坐標為(0﹐-2) a6 a3
sin q = =
類題 2a2 2
1 如右圖 ∴q =120n
∵x=4 cos 225n 故 Q 點的極坐標為 [2a2 ﹐120n]
=-2a2 252 範例 16
y=4 sin 225n 設小啟繞圓心 O 走了角 q
=-2a2 ∵小啟與小榮之速度比為 3:2
∴P 點的直角坐標為 2
∴小榮繞圓心 O 走了角 q
(-2a2 ﹐-2a2 ) 3
2 如右圖 此時小啟所在位置的廣義角為 q,

∴AB=al22+42
∵∠AOB=90n 2
而小榮所在位置的廣義角為 180n-
( 3 )
q

=2a5
66 互動式教學講義.數學(2) 解答篇
頁碼 頁碼
2 sin q

當兩人相遇時,q 與 180n- q 為同界角
3 ) 254 3 tan q =
cos q
=2 c sin q =2 cos q

2 代入 sin2q +cos2q =1 c 4 cos2q +cos2q =1


( 3 )
c q - 180n- q =360n×n,n 為整數
1 1
a5
c cos2q = ∵cos q >0 ∴cos q =
3 5
c q = (180n+360n×n),n 為整數
5 ∴2 sin q -3 cos q =2(2 cos q)-3 cos q
第五度相遇時,將 n=4 代入上面等式 1
a5
=cos q = ,故選E
3
得 q = (180n+360n×4)=972n=252n+360n×2
5 4 A sin(-69n)=-sin 69n
為第三象限角,因此在第三象限相遇 B sin 111n=sin(180n-69n)=sin 69n
類題 C sin 249n=sin(180n+69n)=-sin 69n
∵甲、乙兩星公轉之週期比為 2:7 D sin 339n=sin(270n+69n)=-cos 69n
∴甲、乙兩星公轉之角速度比為 7:2 E sin 789n=sin(69n+360n×2)=sin 69n
設甲星所在位置的廣義角為 q, 故選BE
2 5 sin 300ntan 150n+sin 150ncos 120n
則乙星所在位置的廣義角為 q
7 =sin(360n-60n)tan(180n-30n)
2 +sin(180n-30n)cos(180n-60n)
當甲、乙兩星與恆星成一直線時,q 與 q 為同界角
7 =-sin 60n×(-tan 30n)+sin 30n×(-cos 60n)
2 a3 1 1 1
c q-
7
q =360n×n,n 為整數 =-
2 (
× -
a3 ) ( )
+ × -
2 2
5 1 1 1
c q =360n×n = - =
7 2 4 4
取 n=1 得 q =504n=144n+360n×1 255 6 如右圖,令∠COD=a,
∵90n<144n<180n 則 q =180n+a
∴兩行星與恆星連成一直線時,行星在第二象限 在直角三角形 OCD 中,
故選B CD
tan a= =CD
OC
4-2 ∴CD=tan a
=tan(q -180n)=tan q,故選C
基礎題
4 sin(270n-q) 4(-cos q)
253 1 ∵q 與 10q 為同界角 7 =
3 cos(90n+q) 3(-sin q)
∴10q-q =360°×n,nlZ 4 4 5
c 9q =360°×n,nlZ = = =
3 tan q 8 2
c q =40°×n,nlZ 3×
15
又 q 為銳角 y 1
a3
∴q =40°或 80° 8 tan q = = ,
-a3
故選B 得 y=-1
2 ∵P(3﹐-4)為標準位置角 q 終邊上一點 cP(-a3 ﹐-1),
3
=al(-a3 )
∴作圖如右得 cos q = , r=OP
5 2 2
+(-1) =2
4 1
sin q =- ∴sin q =-
5 2
3 1
2× +3 sin(180n+q)=-sin q =
2 cos q +3 5 2
∴ =
sin q -1 4 256 9 k=tan 200n=tan
(180n+20n)
- -1
5 =tan 20n,作示意圖如右
6+15 7 cos 110n=cos(90n+20n)
= =-
-4-5 3
alk +1
k
故選C =-sin 20n=- 2
互動式教學講義.數學(2) 解答篇 67
頁碼 頁碼
0 設 L1、L2 的斜角分別為 q1、q2 258 t cos(q -450n)=cos((q -450n)+360n×2)
=cos(270n+q)
=sin q
-2a2
又 cos q = 且 90n<q <180n
3

∴sin q =zx1-
-2a2 2 1
( 3 ) =
3
1
故所求為 sin q =
1 3
a3
則 tan q1= 且 tan q2=a3
-1
a3
y L2 之斜率為 ,
∴q1=30n 且q2=60n

q r=al42+
∴L1 與 L2 的銳夾角為 60n-30n=30n 故斜角為-30n
2
(-4a3 ) =8, 又 L1 之斜率為
4 1 4 1
a3
且 cos q = = - >- ,
8 2 7
-4a3 a3 得-30n<q <0n
sin q = =-
8 2 ∵L3⊥L1
∴q =300n ∴由右圖可知,
故數對 (r﹐q)=(8﹐300n) 夾角 f=〔30n-(-q)〕+90n
進階題 =120n+q >90n
257 w 在直角△ADE 中, 故所夾的銳角為 180n-f=60n-q

DE=alAD +AE =g16+9 =5


令∠AED=a y
2 2 u 由 tan∠AOM=a2 = ,其中 M(x﹐y)
x
AE 3 c M(x﹐y)=M(x﹐a2 x)
cos a= =
DE 5 B+A
又 M=
3 2
∴cos q =cos(180n-a)=-cos a=-
5 c B=2M-A=2(x﹐a2 x)-(6﹐0)
e ∵sin q,-cos q 是 a3 x -x+k =0 的兩解
2
=(2x-6﹐2a2 x)
則 a3 x2-x+k=a3(x-sin q) (x+cos q) OB=2a6
2
2
=a3〔x - (sin q-cos q)x-sin q cos q〕 c OB =24
1 k c(2x-6) 2 2
+(2a2 x) =24
a3 a3
∴sin q -cos q = 且-sin q cos q =
c 12x -24x+12=0
2

1 c x2-2x+1=0
( )
2

a3
2
(sin q -cos q) =
c(x-1) 2
=0
1 c x=1
c sin2q -2 sin q cos q +cos2q =
3 2a2 x 2a2 ×1 2a2
故 tan∠AOB= = =
k 1 2 1 2x-6 2×1-6 -4
a3 a3 a2
c 2× = -1=- ∴k=-
3 3
=-
r 設 OA 為 q 的終邊 2
且 OA=5, 歷屆試題
3 4 259 i A ○:∠A=∠B=∠C=60n
則 cos q = ,sin q =
5 5 B ○:A,B,C 中一個大於 150n,另兩個角小
∵OA' 為 90n+q 的終邊 於 30n即可
設 A'
(x'﹐y') C ×:此條件要三個角均介於 60n至 120n之間才可
x' 4 1
則 =cos(90n+q)=-sin q =- ∴x'=-4 D ×:sin q = ,q =30n或 150n,三個角相加
5 5 2
y' 3 不可能 180n
=sin(90n+q)=cos q = ∴y'=3
5 5 E ○:∠A=∠B=30n,∠C=120n
故 A' 坐標為(-4﹐3) 故選ABE
68 互動式教學講義.數學(2) 解答篇
頁碼 頁碼
o 作 BE⊥CD,AF⊥BE,
則∠BAF=q -90n
4-3 面積公式與正餘弦定理
∠DBE=90n-∠ABF 261 範例 1
=90n-(90n-∠BAF) 如右圖,作 AH ⊥BC 於 H
=∠BAF 則 BH =4a2 cos 45n
∵AB=a a2
=4a2 × =4
∴CE=AF 2
=a cos∠BAF=a cos(q -90n) a2
且 AH =4a2 sin 45n=4a2 × =4
=a cos(90n-q)=a sin q 2
∵BD=b 又△ACH 中,AH =4
∴DE=b sin∠DBE=b sin(q -90n) AH a3 4
∴tan 30n= c =
=-b sin(90n-q)=-b cos q CH 3 CH
c CD=CE+DE=a sin q -b cos q ∴CH =4a3
故選B ∴BC=BH +HC=4+4a3
1 1 類題1
260 p A ×:cos q1= < =cos 60nc q1>60n
3 2 如右圖,作 AH ⊥BC 於 H
B ○:如圖1,單位圓上 q1=a 1
則 BH =4 cos 60n=4× =2
∴q1+q2=180n 2
且 AH =4 sin 60n
a3
=4×
2
=2a3
又△ACH 中,AH =2a3
AH 2a3
∴tan 45n= c =1 c CH =2a3
CH CH
圖!
∴BC=BH +HC=2+2a3
1
C ○:如圖2,cos q3=- 262 類題2
3
如右圖,作 AH ⊥BC 於 H
19
則 BH =5 cos B=5×
35
19

7
5 30
CH =6 cos C=6× =
7 7
圖@ 19 30
∴BC=BH +CH = + =7
2a2 7 7
D ×:如圖2,sin q4=-
3 範例 2
E ×:如圖2,q4=q3+2b(b<30n) △ABC 面積為
故選BC 1
×AC×BC×sin C
a sin 73n 2
sin 146n=sin(180n-146n)=sin 34n 1 1 a3
= ×12×10×sin 120n= ×12×10× =30a3
sin 219n=sin(180n+39n)=-sin 39n 2 2 2
sin 292n=sin(270n+22n)=-cos 22n 類題
=-sin 68n △ABC 之面積為
sin 365n=sin(360n+5n)=sin 5n 1 1
×AB×AC×sin A= ×6×3×sin A
由以上可知 2 2
sin 73n>sin 146n>sin 365n>sin 219n>sin 292n =9 sin AN9
c中位數為 sin 365n (∵0n<∠A<180n ∴0<sin AN1)
故選E 故△ABC 面積之最大值為 9
互動式教學講義.數學(2) 解答篇 69
頁碼 頁碼
263 範例 3 265 範例 5
設 AD=x BC AC 2 a3 +1
由正弦定理知 = c =
∵△ABC 面積 sin A sin B sin 45n sin B
=△ABD 面積+△ACD 面積 a2 1
c sin B=(a3 +1)× ×
1 2 2
a6 +a2
∴ ×12×6×sin 60n
2

1 1 4
= ×12×x×sin 30n+ ×6×x×sin 30n
2 2 c∠B=75n或 105n
3 1 當∠B=75n時
c 18a3 =3x+ x
2 ∠C=180n-75n-45n
9 =60n
c x=18a3 c x=4a3
2 2 當∠B=105n時
故 AD=4a3 ∠C=180n-105n-45n
類題 =30n
△ABC 面積=△ABD 面積+△ACD 面積 類題

1 a3 +1 a6
c ×a2 ×1×sin 75n 由正弦定理知 =
2 sin 75n sin C
a6 +a2
c sin C=a6 ×
1 1
= ×a2 ×AD×sin 45n+ ×1×AD×sin 30n
2 2 4
a6 +a2 1 1 a3
a3 +1
ca2 × =AD+ AD × =
4 2 2
3 a3 +1 a3 +1 c∠C=60n或 120n(不合)
c AD= ∴AD=
2 2 3 ∠A=180n-75n-60n=45n
264 範例 4 a6 BC a2 2
a3
又 = c BC=a6 × × =2
BC AC sin 60n sin 45n 2
由正弦定理知 = =2R
sin A sin B 範例 6
6 AC 3 4
c = =2R ∠A=180n× =45n,∠B=180n× =60n,
sin 45n sin 30n 3+4+5 3+4+5
6 sin 30n 5
1 AC= ∠C=180n× =75n
sin 45n 3+4+5
1 ∴由正弦定理知
=6× ×a2 =3a2
2 BC:AC:AB
3 =sin A:sin B:sin C=sin 45n:sin 60n:sin 75n
a2 a3 a6 +a2
2 外接圓半徑 R= =3×a2 =3a2
sin 45n
= : :
類題 2 2 4
∠A=180n-60n-75n=45n =2a2 :2a3 :(a6 +a2 )
由正弦定理知 266 類題1

2 AC AB ∵(a+b):(b+c):(c+a)=4:5:6
= =
sin 45n sin 60n sin 75n a+b=4k .............................1
=2R ∴令 b+c=5k .............................2
2 sin 60n c+a=6k .............................3
1 AC=
sin 45n 1+2+3得 2(a+b+c)=15k
a3 15k ..........................
=2× ×a2 =a6 c a+b+c= 4
2 2
2 sin 75n a6 +a2 5 3 7
2 AB= =2× ×a2 4-2得 a= k,4-3得 b= k,4-1得 c= k
sin 45n 4 2 2 2
=a3 +1 由正弦定理知 sin A:sin B:sin C=a:b:c
1 5 3 7
3 外接圓半徑 R= =a2 = k: k: k=5:3:7
sin 45n 2 2 2
70 互動式教學講義.數學(2) 解答篇
頁碼 頁碼
類題2 268 類題2

5a-2b-c=0 .................. 1 1 如右圖,


a+2b-2c=0 .................. 2 ∠AEB=∠EBC-∠EAC
1+2得 6a-3c=0 c c=2a =60n-30n=30n
3 c AB=BE=2
代入2得 a+2b-4a=0 c b= a
2 △ABE 為等腰三角形
由正弦定理知 sin A:sin B:sin C ∴AE=2×2 cos 30n
3 =2a3
=a:b:c=a: a:2a=2:3:4
2 又△BCE 中,∠BCE=90n,∠EBC=60n
範例 7 且 BC=1 ∴CE=a3
∵A、B、C、D 四點共圓 ∵∠DAE=45n=∠DCE
∴△ABD 與△BCD 有相同的外接圓 ∴A、C、E、D 四點共圓
設外接圓半徑為 R DE CE 2 a2
c = c DE=a3 × × =a6
6 AD sin 45n sin 30n 1 2
由正弦定理知 =2R=
sin 30n sin 45n ∴2 分鐘內移動的距離 DE=a6(公里)
a2 故選D
1 AD=6× ×2=6a2
2 2
1
2 外接圓半徑 R=6×2× =6
2
267 類題

1 ∵∠B=90n
∴CA 是直徑,即 2R=13 ∵∠ADE=∠ACE=90n,令抵達岸邊時交 GAC 於 F
13 ∴∠AFD=180n-75n-90n=15n
a3 a6 -a2
c 外接圓半徑 R=
2
∴△CEF 中, =sin 15n=
2 在△ABD 中, EF 4
由正弦定理知 c EF=3a2 +a6
BD 又快艇每 2 分鐘走 a6 公里,
a6
=2R
sin 120n
即 1 分鐘走 公里
13a3 2
c BD=
2 3a2 +a6
a6
∴抵達岸邊需 =2+2a3(分鐘)
範例 8
∠ACB=180n-102n-48n=30n 2
100 AC 故需再行駛 2+2a3 分鐘才能抵達岸邊
由正弦定理知 =
sin 30n sin 102n 269 範例 9
c AC×sin 30n=100×sin 102n 1 由餘弦定理知
2
100 sin 102n BC
∴AC=
sin 30n =32+52-2×3×5×cos 120n
100×0.9781 9781 1
~
0.5

50
=9+25-2×3×5× -
2 ( )
=195.62~195.6(公尺) =34+15=49
類題1 ∴BC=s49 =7
∠C=180n-76n-74n=30n 2 由餘弦定理知
BC 5 72+82-132
由正弦定理知 = cos B=
sin 76n sin 30n 2×7×8
c BC×sin 30n=5 sin 76n 56
=-
5 sin 76n 5×0.9703 112
c BC= ~
sin 30n 0.5 1
=-
=9.703 2
~9.7
(公里) ∴∠B=120n
互動式教學講義.數學(2) 解答篇 71
頁碼 頁碼
270 類題1 271 範例 10
1 由餘弦定理知 令 AD=x,由餘弦定理可得
2
BC =32+62-2×3×6×cos 60n 在△ABD 中,
1 72+72-x2
=9+36-2×3×6× cos B=
2 2×7×7
=27 72+152-132
在△ABC 中,cos B=
∴BC=s27 =3a3 2×7×15
2 ∵s37 =6.…… 72+72-x2 72+152-132
c =
∴AB>AC>BC 2×7×7 2×7×15
由大邊對大角性質知 c 1470-15x =735
2

∠C 最大 c x2=49
2 2 2
BC +AC -AB c x=±7(負不合)
∴cos C=
2×BC×AC 故 x=7
9+16-37 類題1

2×3×4 設 AC=x ∵∠ADB+∠ADC=180n
1 ∴cos∠ADB=-cos∠ADC
=-
2 由餘弦定理知
∴∠C=120n 42+52-72 52+62-x2
=-
類題2 2×5×4 2×5×6
設 AB=x -8 61-x2
c =-
4 6
c x2=49 c x=7,故 AC=7
類題2

設 AB=x,AD=x
由內角平分線性質知
由餘弦定理知 AB BD 3 1
2 2 2 = = =
BC =AB +AC -2×AB×AC×cos A AC CD 6 2
c(2a7 ) 2
=x2+42-2×x×4×cos 60n 設 AB=x,AC=2x
c 28=x +16-4x
2
∵cos∠BAD=cos∠CAD
c x2-4x-12=0 由餘弦定理知
c(x-6)(x+2)=0 x2+x2-9 x2+(2x) 2
-36

c x=6 或 x=-2(不合) 2×x×x 2×x×2x
故 AB=6 2x2-9 5x2-36
c =
類題3 1 2
c x =18
2

2x2-9 27 3
∴cos∠BAD= = =
2x2 36 4
272 範例 11
a2 ab a2
由上圖可知△ABC 之圖形有兩種
a2+b2-c2
A ×:∵∠B 無法確定 由餘弦定理知 cos C= = =
2ab 2ab 2
∴cos B 無法確定
∴∠C=45n
B ○:∠C 可能為 q 或 180n-q
類題
∴sin C 可以確定 2
(b+c) -a2=bc
C ×:無法確定
c b2+c2+2bc-a2=bc
D ×:無法確定
c b2+c2-a2=-bc
E ○:由正弦定理知
b2+c2-a2 -bc 1
BC ∴由餘弦定理知 cos A= = =-
=2R 外接圓半徑可確定 2bc 2bc 2
sin A
∴∠A=120n
故選BE
72 互動式教學講義.數學(2) 解答篇
頁碼 頁碼
2 設最近距離為 OC,則△OAB 面積為
1 AC=al32+52-2×3×5×cos 60n
範例 12
1 1
×OA×OB×sin 120n= ×AB×OC
2 2
a3
=s19
2 設 AD=x 1 1
c ×2×1× = ×a7 ×OC
2 2 2
s21
∵ABCD 是圓內接四邊形
∴對角互補 c∠D=120n 故最近距離 OC= (浬)
由餘弦定理知 7
2
AC =x2+22-2×2×x×cos 120n 類題

=32+52-2×3×5×cos 60n 由餘弦定理知


2
c x +2x-15=0 c(x+5)
2
(x-3)=0 AB =802+502-2×80×50×cos 60n
∴x=-5 (不合) 或 x=3,故 AD=3 =6400+2500-4000=4900
類題1 c AB=f4900 =70
1 ∵ABCD 為圓內接四邊形 ∴對角互補 故 A,B 兩點間之距離為 70 公尺
設∠B=q,則∠D=180n-q,由餘弦定理知 274 範例 14
2
AC =32+52-2×3×5×cos q
=32+82-2×3×8×cos(180n-q)
c 34-30 cos q =73+48 cos q c 78 cos q =-39
1
c cos q =- c q =120n,故∠B=120n
2
2 2 2
2 AC =3 +5 -2×3×5×cos 120n
=9+25+15=49 c AC=7
由正弦定理得知
AC 7 7a3
=2R c =2R c R=
sin B sin 120n 3
273 類題2 立體圖 平面圖

∴BD=al42-12 =s15
連 BD,則∠ABD=90n 如上圖,在△AOC 中,AO=100
在△BOC 中,BO=100a3
令 CD=x,則由餘弦定理知 在△AOB 中,∠AOB=90n
15=12+x2-2×1×x×cos C 2 2 2
c AB =AO +BO =1002+(100a3 )
2

1 =1002(1+3)=4×1002
但 cos C=-cos A=- (∵圓內接四邊形對角互補)
4 ∴AB=2×100=200,故 A 與 B 相距 200 公尺
1 類題1
c 15=1+x2+ x c 2x2+x-28=0 c(2x-7)
(x+4)=0
2
7 7 7
c x= 或 x=-4(不合) ∴x= ,故 CD=
2 2 2
範例 13
1

立體圖 平面圖

AB=al12+22-2×1×2×cos 120n
由餘弦定理得
如上圖,設飛機 C 的高度 h 公尺,在△AOC 中
h h
=zx1+4-4× -
1 c OA=
( )
2
=a7
∵tan 30n=
OA tan 30n
=a3 h

a7
在△BOC 中
且用了 20 分鐘,故時速為 =3a7(浬/時) h h h
a3
1 ∵tan 60n= c OB= =
OB tan 60n
3
互動式教學講義.數學(2) 解答篇 73
頁碼 頁碼
在△AOB 中 ∵∠AOB=90n 故△AOB 的面積為
2 2 2
∴OA +OB =AB 1
×AO×BO×sin∠AOB
h 2 10 2 2
c 2
(a3 h) +
( )
a3
=20002 c
3
h =20002
1
= ×150a3 ×50a3 ×
2a2

故飛機 C 的高度 h=2000×zx


3 2000s30 2 3

10 10 =7500a2(平方公尺)
=200s30(公尺) 276 範例 15
275 類題2 a+b+c 5+6+7
1 令 s= = =9
如右圖所示 2 2

△ABC 面積=al(
設塔高為 由海龍公式得
s s-a)
=al9×(9-5)×(9-6)×
OC=h 公尺 (s-b)
(s-c)
在△AOC 中 (9-7)
h =6a6
=tan 30n
OA abc
2 △ABC 面積=
1 4R
a3
= 立體圖
5×6×7 35a6
c 6a6 = c外接圓半徑 R=
c OA=a3 h 4R 24
在△BOC 中 3 △ABC 面積=rs
h 2a6
=tan 45n=1 c 6a6 =r×9 c內切圓半徑 r=
OB 平面圖 3
c OB=h 類題

在△OAB 中,由餘弦定理知 1
2 2 2 △ABD 的面積= ×4×10×sin 60n=10a3
AB =OA +OB -2OA×OB cos 30n 2
a3 1
c 3600= (a3 h) 2
+h2-2×a3 h×h× 在△BCD 中 ∵s= (6+10+14)=15
2 2
c 3600=3h2+h2-3h2 ∴由海龍公式得
c h2=3600 c h=60
=al15×(15-10)×(15-14)×(15-6)
△BCD 的面積
故塔高為 60 公尺
類題3 =al15×5×1×9 =15a3
150 故四邊形 ABCD 的面積
tan 30n=
AO =△ABD 的面積+△BCD 的面積
150 =10a3 +15a3 =25a3
c AO= =150a3
1
a3 4-3
150
tan 45n= c CO=150 基礎題
CO
150 150 277 1 如右圖,作頂角
a3
tan 60n= c BO= =50a3 之角平分線 AD 垂
BO
設 BC=AC=x,觀察△AOB,由中線定理知 直平分底邊 BC
(150a3 )2+(50a3 ) 2
=2(1502+x2) ∠BAD=70n,∠ABD=20n,AB=2,
c 67500+7500=2(22500+x2) c 37500=22500+x2 BD
cos 20n=
c x2=15000 c x=50a6 ∴AB=100a6 AB
在△AOB 中,由餘弦定理可知 ∴BD=2 cos 20n,BC=2BD=4 cos 20n,故選E
(150a3 ) 2
+ 2
(50a3 )- 2
(100a6 ) 2 如右圖,設 AB=c,BC=a
cos∠AOB= ∠C=180°-45°-60°=75°
2×150a3 ×50a3
15000 1 a c
= = 由正弦定理知 =
sin A sin C
a6 +a2
45000 3
2a2 a
c sin∠AOB= c =
3 sin 45° sin 75°
74 互動式教學講義.數學(2) 解答篇

a6 +a2
頁碼 頁碼
9 如右圖,由餘弦定理知
又 sin 75°= 2 2 2
4 BD =AB +AD
a a6 +a2 -2×AB×AD×cos A
a2 a6 +a2
c = =4 2 2
=BC +CD
2 4 -2×BC×CD×cos C
a2 c 1 +42-2×1×4×cos A
2

∴a=4× =2a2 ,故 BC=2a2


2 =22+32-2×2×3×cos(180n-∠A)
3 1
3 ∠A=180n× =45n, c 17-8 cos A=13+12 cos A,得 cos A=
3+4+5 5
4 2 1 77
∠B=180n× ∴BD =17-8× =
3+4+5 5 5
=60n, d385
得 BD=
5 5
∠C=180n× =75n
3+4+5 0
BC 1
由正弦定理知 =
sin 45n sin 75n
BC 1
a2 a6 +a2
c = c BC=a3 -1

2 4
故選ABCD
278 4 令 b+c=5t,c+a=6t,a+b=7t,t≠0
得 a=4t,b=3t,c=2t,由正弦定理知
立體圖
sin A:sin B:sin C=a:b:c=4:3:2
如上圖,直升機位於 C 點,在地面之投影點為 O
5 設圓之半徑為 R
點,OC=600(公尺)
AB CD
則由正弦定理知 =2R= ∠CAO=60n,
sin 45n sin 30n
a2
∠CBO=30n,
3a2
c AB= × =6,故選C ∠AOB=180n-60n
1 2
=120n
2
600
a3
6 在△ABC 中 則 AO=
AC=2,∠CAB=68n,∠CBA=30n
=200a3 ,
∴∠ACB=180n-30n-68n=82n 平面圖
BO=600a3
AB 2
由正弦定理知 = 由餘弦定理知
sin 82n sin 30n 2 2 2
AB =AO +BO -2×AO×BO×cos 120n
2 sin 82n 2×0.9903
c AB= ~ =(200a3 )2+(600a3 )
2

sin 30n 0.5


1
=3.9612~3.96(公里)
7 由餘弦定理知
-2×200a3 ×600a3 × -
2( )
2 =120000+1080000+360000=1560000
AB =82+102-2×8×10×cos 60n
∴AB=200s39(公尺)
=64+100-80
進階題
=84
280 q A ○:sin2A+sin2B=sin2C
∴AB=s84
a b c
( 2R ) ( 2R ) ( 2R )
2 2 2
=2s21 c + =
279 8(a+b+c)(a-b+c)=ac
c a2+b2=c2 c∠C=90n
c(a+c)-b2=ac
2

1
c a2+c2-b2=-ac B ×:sin A= c∠A=30n或 150n
2
a2+c2-b2 -1
由餘弦定理知 cos B= = C ○:cos A<0 c 90n<∠A<180n
2ac 2
∴∠A 為鈍角
∴∠B=120n
互動式教學講義.數學(2) 解答篇 75
頁碼 頁碼
D ○:a+b>c r 如右圖
c 2R sin A+2R sin B>2R sin C AD AC 10 5
∵ = = =
c sin A+sin B>sin C BD BC 6 3
E ×:由正弦定理知 5 15 9
∴AD=12× = ,BD=
c b 5+3 2 2

sin C sin B 在△ACD 與△ACB 中,由餘弦定理知
a2 ×sin 30n a2 15 2
c sin C=
1

2
cos A=
( )
102+
2
-CD
2


102+122-62
15 2×10×12
2×10×
2
225

c 100+
4 ) 2
-CD ×8=(100+144-36)×5
2
c∠C=45n或 135n c 800+450-8CD =1040
故選ACD 2 210 105 d105
∴CD = = ,得 CD=
w △ABC 面積=△ABD 面積+△ACD 面積 8 4 2

BC=alAB +AC =h64+36 =10,


1 282 t 在直角△ABC 中,
c ×6×10×sin 120n 2 2
2
1 1 1
= ×6×AD×sin 30n+ ×AD×10×sin 90n BE=CD= BC=5
2 2 2
a3 1 AC 3
c 30× =3AD× +5AD×1 令∠ABC=q,則 sin q = =
2 2 BC 5
30a3 在△ABE 中,由餘弦定理知
∴AD= 2 2 2
13 AE =AB +BE -2×AB×BE×cos (q+90n)
281 e 由正、餘弦定理及三角形的面積公式可得 =64+25-2×8×5×(-sin q)
a c 3
A ×: = =89+80× =89+48=137
sin A sin C 5
3a2 ∴AE=d137
ca6 ×sin C=3×sin 45n=
2 y 由餘弦定理知
a3
2 2 2
AC +AB -BC
c sin C= cos∠CAB=
2 2×AC×AB
∴∠C=60n或 120n 57600+22500-44100 1
= =
即∠B=75n或 15n 2×240×150 2
a3 a3
B ○:由A得 sin C= c sin∠CAB= ,又 AD=BD=CD
2 2
2 2 2
C ×:a =b +c -2bc cos A ∴AD 為△ABC 外接圓之半徑
c(a6 ) 2
=b2+32-2b×3×cos 45n BC 210
a3
由正弦定理知 2AD= = =140a3
c b2-3a2 b+3=0 sin∠CAB
3a2 ±a6 2
∴b=AC=
2 得 AD=70a3(公尺)
D ×:△ABC 的面積為 u 在△ABC 中,由餘弦定理知
a6 ±a2
2 2 2
1 1 AB +AC -BC
ac sin B= ×a6 ×3× cos∠CAB=
2 2 4 2×AB×AC
9±3a3 16+25-49 -8 1
= = = =-
4 2×4×5 40 5
E ○:△ABC 的外接圓半徑為 在△AEH 中,AE=4,AH=5,
a a6 ∠EAH=180n-∠BAC
= =a3 2 2 2
2 sin A 2 sin 45n EH =AE +AH -2×AE×AH×cos∠EAH
故選BE =16+25-2×4×5×(-cos∠BAC)
=41-8=33 ∴EH=s33
76 互動式教學講義.數學(2) 解答篇
頁碼 頁碼
283 i 設∠CBD=q,∠BCA=90n-q,圓半徑 r=4 s 令 AC=x,∠ADC=q,
由正弦定理得 則∠ABC=180n-q
CD AB 由餘弦定理可得
= =2r
sin∠CBD sin∠BCA 在△ACD 中,
6 AB 52+72-x2
c = =8 cos q =
sin q sin(90n-q) 2×5×7
3 在△ABC 中
∴sin q = ,
4 12+52-x2
a7
cos(180n-q)=
2×1×5
AB=8 sin(90n-q)=8 cos q =8× =2a7
4 ∵cos(180n-q)=-cos q
o 如右圖 12+52-x2 52+72-x2
c =-
A、C 重合後之摺痕為 AC 的 2×1×5 2×5×7
2 2
中垂線 26-x 74-x
c =
所求之摺痕長即為 EF 長 1 -7
26 c x2=32 c AC=x=s32 =4a2
令 AF=x,則 cos∠OAF=
x d ∵APQR 為平行四邊形
由餘弦定理知 ∴∠CRQ=∠A
332+522-252 =60n(同位角)
cos∠OAF=
2×33×52 ∴△CRQ 為正三角形
3168 12 設 CR=x,則 RQ=AP=x
= =
2×33×52 13 ∴四邊形 APQR 面積為
12 26 169 x×(13-x)×sin 60n=20a3
a3
c = c x=
13 x 6
c( x 13-x)× =20a3 c x=5 或 8
5 OF 2
在△AOF 中,sin∠OAF= =

PR=al52+82-2×5×8×cos 60n
13 x 在△APR 中,由餘弦定理得
5 169 65
c OF= × =
13 6 6 =s49 =7
65 65 65 285 f 在△ABE 中,
故 EF= ×2= ,即摺痕長為
6 3 3 由正弦定理得
歷屆試題 1 BE

p 在△ABC 中,由正弦定理知 sin 120n sin 15n
AC 11 AE
= =
sin 70n sin 30n sin 45n
c AC~11×0.940×2 3a2 -a6 a6
∴BE= ,AE=
=20.68 6 3
~20.7(公尺) 又∵△ABEw△BCF (AAS 全等)
故選C ∴BF=AE
284 a 設 A、B、C、D 分別表示 a6 3a2 -a6
甲、乙、丙、丁四鎮的所
∴EF=BF-BE=
3 (

6 )
在位置 3a6 -3a2 a6 a2
= = -
在△BCD 中,由正弦定理 6 2 2
CD 20 g
知 =
sin 120n sin 45n
a3
20×
2
a2
c CD= =10a6

2 A ○:在△ADE 中,AD=al22+22 =2a2,


~10×2.449=24.49~24.5(公里) 且∠DAE=45n
故選A B ×:∠DAB=105n-45n=60n
互動式教學講義.數學(2) 解答篇 77

287 k A ○:AB=al
頁碼 頁碼

BC=al
2 2
C ×:在△ABD 中,由餘弦定理知 (0-1) +(2-0) =a5
2

AC=al
2 2 2 2
BD =(a6 +a2 ) +(2a2 ) (1-4) +(0-1) =s10
-2(a6 +a2 )×2a2 ×cos 60n 2 2
(0-4)+(2-1) =s17
=8+4a3 +8-4a3 -4 ∴AC 最長
=12
c BD=2a3
D ○:在△ABD 中,由正弦定理知
BD AD

sin A sin B
2a3 2a2 B ×:由 AC>BC>AB 可得∠B>∠A>∠C
c =
sin 60n sin B 又∠A、∠C 必為銳角
a2 ∴sin A>sin C
c sin B= 2 2 2
2 AB +BC -AC
C ×:cos B=
c∠ABD=45n或 135n(不合) 2 AB×BC
2 2 2
E ×:∵△BCD 的三邊長為 2,2a3,4 (a5 ) +(s10 ) -(s17 )

∴△BCD 是∠B 為直角的直角三角形 2×a5 ×s10
1 1
c△BCD 的面積為 ×2×2a3 =2a3 =- <0
2 5a2
故選AD ∴∠B 為鈍角,故△ABC 為鈍角三角形
286 h A ○:cos A 確定c∠A 確定 1
D ○:∵cos B=-
c△ABC 確定 (SAS) 5a2

∴sin B=zx1- -
B ○:cos B 確定c∠B 確定 1 7
( )
2

c∠C 必為銳角 (∵∠B 必大於∠C) 5a2 5a2
∴△ABC 確定 7a2

C ×:cos C 確定 10
c∠C 確定(必為銳角) E ×:由正弦定理
但∠B 可能是銳角、 AC s17 10s17
2R= = =
鈍角或直角 sin B 7a2 7a2
∴△ABC 不確定 10
1 10s16 10a8 20a2
D ×:△ABC 的面積為 ×4×6×sin A > = =
2 7a2 7 7
△ABC 面積 20×1.414
c sin A= ~ =4.04>4
12 7
c∠A 可能有兩解 ∴R>2
6 〔另解〕
E ×:sin B= ,但不確定∠B 為銳角或鈍角
2R 如下圖
故選AB
j

∵△ABC 為鈍角三角形,
且∠B 為鈍角
∴2R>AC
s17 s16
作圖如上,所求為
1
(正方形面積) +(4 個小三角形面積) 得 R> AC= > =2
2 2 2
1
= 2
(2+a2 )

+4× ×1×1
2 ) 故選AD

=6+4a2 +2=8+4a2(平方公尺)
78 互動式教學講義.數學(2) 解答篇
頁碼 頁碼
二、多選題
第一次段考模擬試題
a1+9d=20 a1=29
290 6 A ×: c
288 1B 2C 3E 4D 5A 6BCD a1+19d=10 d=-1
3 B ○:承A
7ABD 8CD 9ACDE 0CD q-
2 C ○:a15=a1+14d=29+14(-1)=15
10 D ○:a100=a1+99d
w e 13 r2 t 19800 y 60
21 =29+99×(-1)=-70
6 E ×:a30=0,a31=-1
u(24﹐15) i Q=6P-50 o1E;2 y= x+24
5 故選BCD
------------------------------ 詳解 ----------------------------- 7 A ○:a7=S7-S6<0,a6=S6-S5>0 ∴d<0
一、單選題 a1+a11
B ○:∵a6= ∴S11=11a6>0
288 1 a2=-2a1,a3=-2a2,……,a9=-2a8 2
則〈an〉為等比數列,且公比為-2 C ×:∵a6+a7=S7-S5>0
因此 a8=a1r7=1×(-2) 7
=-128,故選B 12(a1+a12)
∴S12= =6(a6+a7)>0
2 a1=S1=3+b 2
a2=S2-S1= (32+b)- (3+b)=6 a1+a13
3 D ○:∵a7= ∴S13=13a7<0
a3=S3-S2= (3 +b)- (32+b)=18 2
a3 a2 18 6 E ×:∵a6>0,a7<0 ∴S6 為最大項
c公比為 = c = c 3+b=2
a2 a1 6 3+b 故選ABD
∴b=-1,故選C 8 A ×:原算術平均數為 6
3 設 xi:1,2,3,4,5 若捨去 6 後,算術平均數仍為 6
yi:6,7,8,9,10 B ×:原中位數為 6
zi:2,4,6,8,10 若捨去 6 後,中位數仍為 6
vi:1,1,3,5,5 CD ○:必改變
ui:2,2,3,4,4 E ×:若捨 2 或 4 或 6 或 8 或 10 皆不改變
可知 yi=xi+5,zi=2xi c sy=sx,sz=2sx 故選CD
c sx=sy<sz 291 9 A ○:my=-3mx+64=-3×12+64
又 mx=mv=mu=3 =-36+64=28
且 2,2,3,4,4 此組數據最為集中 B ×:sy=|-3|sx=3×4=12
1,1,3,5,5 此組數據最為分散 C ○:(xi﹐yi)均在 yi=-3xi+64 上,
所以 su<sx<sv,故選E 其斜率為-3,為完全負相關
1+2+3 3+5+k 8+k 故 x 與 y 的相關係數為-1
289 4 mx= =2,my= =
3 3 3 1
D ○:sx=zx 〔(x1-mx)2+(x2-mx)2+……+(xn-mx)2〕
7 8+k 7 n
∵y= x-1 必過(mx﹐my) ∴ = ×2-1
2 3 2 1
c 4=zx ×320
c 8+k=18 c k=10,故選D n
5 設公差為 d,由 3a4=7a7 320
c 16= c n=20
c3(a1+3d)=7(a1+6d) n
c 3a1+9d=7a1+42d E ○:迴歸直線即為 y=-3x+64,
4 故必過點(0﹐64)
c d=- a1,其中 a1>0,d<0
33 故選ACDE
若 Sn 有最大值,表示 anM0 0 A ×:若各數據點(xi﹐yi)均落在一鉛垂線或一
4 水平線上時,表示 X 與 Y 為零相關
c an=a1+(n-1)d=a1+ (n-1)-

33
a1 M0
) B ×:|r|愈大,表示 X 與 Y 的相關程度愈高
4
( )
C ○
c 1+ (n-1)- M0(同除以 a1>0)
33 rsY r2sY
D ○:a= c ar= M0
4 33
( )
sX sX
c(n-1)- M-1 c n-1N
33 4 E ×:r(Z﹐W)=-r(X﹐Y)=-r
c nN9.…… ∴n=9,故選A 故選CD
互動式教學講義.數學(2) 解答篇 79
頁碼 頁碼
三、填充題 x
3 當 x>74 時,中位數為 74 c +60=74
a1+a3+a5+a7=-6 ....................1 5
292 q 由題意可得
a2+a4+a6+a8=9 ........................2 ∴x=70(不合)
2 9 3 3 故數學成績為 60 分
c r= =- ∴公比為-
1 -6 2 2 18+21+22+a+b
294 u =20 c a+b=100-61=39
1 5
-1
a1-1 3 1-3 2 182+212+222+a2+b2
w a2= = = = -202=10
4a1-3 1 4-9 5 5
4× -3
3 c a2+b2=410×5-(324+441+484)=801
2 將 b=39-a 代入得
-1
a2-1 5 2-5 3 a2+(39-a) 2
=801
a3= = = =
4a2-3 2 8-15 7 c 2a2-78a+720=0
4× -3
5 c a2-39a+360=0
3 c(a-24) (a-15)=0
-1
a3-1 7 3-7 4 c a=24 或 15
a4= = = =
4a3-3 3 12-21 9 當 a=24 時,b=39-24=15
4× -3
7 當 a=15 時,b=39-15=24

∵a>b ∴數對(a﹐b)=(24﹐15)
n i 迴歸直線必過(mx﹐my)=(3﹐5)
c an=
2n+1 由 L 過(3﹐5)、(2﹐1),
10 10 可知迴歸直線 L 為 4x-y=7
∴a10= =
2×10+1 21 1
x= (P-5)
e a4=a1+3d=60 .................1 P=2x+5 2
c
a8=a1+7d=32 .................2 Q=3y+1 1
y= (Q-1)
2-1得 4d=-28,d=-7 代入1得 a1=81 3
an=a1+(n-1)d=81+(n-1)× (-7)<0 1 1
代入 4x-y=7 得 4 (P-5)- (Q-1)=7
c-7n+88<0 c 7n>88 c n>12.…… 2 3
∴取 n=13,故第 13 項開始為負數 1 1
c 2(P-5)- (Q-1)=7 c (Q-1)=2P-17
293 r a1+a2+……+a15=0 c 15a8=0 c a8=0 3 3
又 an=a8+(n-8)d=(n-8)d c Q=6P-50
c|a1|+|a2|+……+|a15| 四、混合題
=|-7d|+|-6d|+……+|-d|+|0|+|d| 26+28+30+32+34
295 o mx= =30,
+……+|6d|+|7d|=112 5
c 56d=112 c d=2 58+54+66+50+72
my= =60
t a50=a48+482=a46+482+462=…… 5
=a2+482+462+……+22 x (yi-my)(xi-mx)2 (yi-my)2
y xi-mx yi-my (xi-mx)
=200+4(12+22+……+242) 2658 -4 -2 8 16 4
24×25×49 2854 -2 -6 12 4 36
=200+4× =200+19600=19800 3066 0 6 0 0 36
6
y 設數學成績為 x 分, 3250 2 -10 -20 4 100
3472 4 12 48 16 144
則五科成績為 x,70,72,74,84
合計 48 40 320
x+70+72+74+84 x
其算術平均數為 = +60 1 氣溫與冰淇淋銷售數量的相關係數為
5 5
48 48 3 3a2
s40 ×d320
x = = =
1 當 x<72 時,中位數為 72 c +60=72 80a2 5a2 10
5
故選E
∴x=60
48 6 6
x 2 y-60= (x-30)= (x-30)= x-36
2 當 72NxN74 時,中位數為 x c +60=x 40 5 5
5
6
∴x=75(不合) c y= x+24
5
80 互動式教學講義.數學(2) 解答篇
頁碼 頁碼
8!
第二次段考模擬試題 E ×: =3360(組),故選AB
2!3!
296 1D 2C 3C 4C 5B 6AB 7AB 8 A={(1﹐5)﹐(2﹐4)﹐(3﹐3)﹐(4﹐2)﹐(5﹐1)}
8AD 9ACDE 0 BCDE q 8 B={(2﹐6)﹐(3﹐5)﹐(4﹐4)﹐(5﹐3)﹐(6﹐2)}
1 3 127 C={(1﹐1)﹐(2﹐2)﹐(3﹐3)﹐(4﹐4)﹐
w 1024 e r t y 130 u 35
2 28 924 (5﹐5)﹐(6﹐6)}
i 70 o1ABD;2 2 A ○:A∩B=∅ ∴A、B 為互斥事件
------------------------------ 詳解 ----------------------------- B ×:B∩C={(4﹐4)}
一、單選題 5 5
C ×:P(A)= 2 =
296 1 將數學年鑑看成 A,數學講義看成 B, 6 36
未領到的看成× 5 5
D ○:P(B)= 2 = ∴P(A)=P(B)
則形成 AABBBB××共 8 件物品分給 8 個人 6 36
8! 6×5
∴有 =420(種),故選D E ×:P(C')= 2
2!4!2! 6
2 1 當甲、乙互斥時,和事件機率最大 故選AD
2 1 11 C 31×52 75 C 32×5 15
此時和事件機率為 + = 299 9 A ○: 3 = ;B ×: 3 =
3 4 12 6 216 6 216
3
2 當乙發生,甲必發生時,和事件機率最小 5 125
C ○: 3 =
2 6 216
此時和事件機率為
3 D ○: 情況 恰 1 顆 6 點 恰 2 顆 6 點 恰 3 顆 6 點
2 11 75 15 1
由1、2得 NPN ,故選C 機率
3 12 216 216 216
3 所求為 金額 216 432 648
n(任意兩人握手)-n(夫婦握手)-n(女人和女人握手)
所求期望值為
=C 20 10
2 -10-C 2 =190-10-45=135
(次) 75 15 1
故選C ×216+ ×432+ ×648
216 216 216
297 4 1 若己選手未入選,則必是由一位右手持拍選
=75+30+3=108(元)
手與一位左手持拍選手搭配,有 3×2=6(種)
E ○:期望值為 108<120 ∴對玩家不利
2 若己選手入選,則只要從其他 5 位選手任選
故選ACDE
1 人與己搭配即可,有 5 種 0 A ×:由巴斯卡公式得 C 99 99 100
8 +C 9 =C 9
由1、2可得共 6+5=11(種) ,故選C B ○:由巴斯卡公式得 C 34 34 35
9 +C 10=C 10
3 3
2! 1 (C 1×C 1)×2! 18 1 C ○:P 100 95
5 P1= 2 = ,P2= = = 5 ×P 25= (100×99×98×97×96)
2 2 62 36 2 ×(95×94×……×71)
4!
=P 10030
2!2! 6 3 D ○:C 48 48 48
P 3= = = ,得 P1=P2>P3,故選B 10+2×C 9 +C 8
4
2 16 8 48! 48! 48!
二、多選題 = +2× +
10!38! 9!39! 8!40!
6 A ○:物理成績不及格則無法滿足第二個條件
48! (39×40+2×10×40+9×10)
B ○:兩個條件皆符合 =
10!40!
C ×:只要國文成績不低於 80 分即可領
48!×2450 48!×49×50 50!
D ×:要看物理成績是否及格 = = =
10!40! 10!40! 10!40!
E ×:國文、英文成績皆低於 80 分或物理成績 =C 50 10
不及格,故選AB 〔另解〕
298 7 A ○:P74=7×6×5×4=840(種)
C 50
10 表示由 50 個人取 10 人的取法數
B ○:P74=840(種) 若 50 人中有兩人:甲及乙
C ×:n(甲排首) -n(甲排首且乙排尾) 1 不含甲、乙兩人有 C 48 10 種取法
=6!-5!=720-120=600(種) 2 恰含甲、乙之一人有 C 21×C 48 9 種取法
D ×:將甲乙綁起與其他人排列,再將甲乙排 3 含甲、乙兩人有 C 48 種取法
8
列得 6!×2!=720×2=1440(種) 得 C 50 =C 48
+2×C 48
+C 48
10 10 9 8
互動式教學講義.數學(2) 解答篇 81
頁碼 頁碼
E ○:P 80
50 表示由 80 人中取 50 人排成一列的排法 y 1 若個位數字為 0,則有 6×5=30(個)
若 80 人中有兩人:甲及乙 2 若個位數字為 2,4,6 或 8,但百位數字不
1 不含甲、乙兩人有 P 78 50 種排法 為 0,則有 5×5×4=100(個)
2 恰含甲、乙之一人有 由1、2可得共有 30+100=130(個)
C 21×P 78 50 78
49×C 1 =100×P 49 種排法 1
(x + x )展開式中一般項為
7
3
u
3 含甲、乙兩人有
P 78 49 50 78
48×C 1 ×C 1 =2450×P 48 種排法 1
( x )=C x x =C x
r
7 3 7-r 7 21-3r -r 7 21-4r
C (x )
r r r
得 P 80 78 78
50=P 50+100×P 49+2450×P 48
78

故選BCDE 令 21-4r=5 c r=4


三、填充題 7×6×5
∴展開式中 x5 項係數為 C 74=C 73= =35
300 q A={1﹐3﹐5},若事件 B 與 A 互斥,則 B∩A=∅ 1×2×3
而 S= {1﹐2﹐3﹐4﹐5﹐6} 302 i 每一盒的燈泡中有 7 個好的,1 個壞的
∴B 事件的樣本點為由 2、4、6 中取得 C 72 21 3
∴每一盒賣出的機率為 8 = =
共有 C 30+C 31+C 32+C 33=1+3+3+1=8 種選法 C2 28 4
故 B 事件有 8 個 ∴獲利期望值為
w 所求為 C 11 11 11 11
6 +C 7 +C 8 +C 9 +C 10+C 11
11 11
3 1
×(360-200)+ ×(-200)
又 C 11 11 11 11 11 11
6 =C 5 ,C 7 =C 4 ,C 8 =C 3 ,C 9 =C 2 ,
11 11
4 4
11 11 11 11
C 10=C 1 ,C 11=C 0 =120-50=70(元)
1 四、混合題
故所求為 (C 11 11 11 11
0 +C 1 +C 2 +C 3 +C 4 +C 5
11 11

2 o 1 1 3 次皆往右,
+C 11 11 11 11
6 +C 7 +C 8 +C 9 +C 10+C 11)
11 11
則 P 停在數字 1+3=4
1 2 3 次中 2 次往右,1 次往左,
= ×211=210=1024(種)
2 則 P 停在數字 1+(2-1)=2
1 3 3 次中 1 次往右,2 次往左,
e ∵P (A'∩B') =P ( (A∪B) ')=1-P (A∪B)=
3 則 P 停在數字 1+(1-2)=0
2 4 3 次皆往左,
∴P(A∪B) =
3 則 P 停在數字 1+(-3)=-2
又 P(A∪B) =P (A) +P (B) -P (A∩B) ∴P 停留的數字可能為 4,2,0 或-2
2 1 1 2 1 1 1 故選ABD
c =P(A) + - cP (A) = - + =
3 3 6 3 3 6 2 2 ∵P 往右的機率是往左機率的 2 倍
301 r 原中位數為 85 分 2 1
∴P 往右的機率為 ,往左的機率為
任意選出 3 人刪除其成績後變成 6 人 3 3
新中位數為由小至大的第 3 位與第 4 位成績的算 停留
4 2
84+86 數字
術平均數,即 =85
2 3! 2 1
2! ( 3 ) ( 3 )
2
× ×
則刪完的結果可能為 2 8
( 3 )= 27
3
機率
○,○,84,86,○,○ 12

50,60,77 擇一刪除 27
且 89,95,100 擇一刪除 停留
0 -2
C 31C 31 9 3 數字
故所求機率為 9 = =
C3 84 28 3! 2 1
( )( )
2
× ×
t 至少 50 元
(含 50 元)的情形如下 2! 3 3 1 1
( 3 )= 27
3
機率
10 元 6 5 5 4 6

5元 0 1 0 2 27
1元 0 0 1 0 所求期望值為
6 6 4
C +C C +C C +C C
6 5 1
6
5
2
1
6
4
4
2
8 12 6 1
所求機率為 ×4+ ×2+ ×0+ ×(-2)
C 12
6
27 27 27 27
1+24+12+90 127 32+24+0-2 54
= = = = =2
924 924 27 27
82 互動式教學講義.數學(2) 解答篇
頁碼 頁碼
5 sin 270n=-1,cos 270n=0,
第三次段考模擬試題
sin(270n-q)=-cos q,cos(270n-q)=-sin q
303 1A 2C 3D 4E 5B 6BDE (-1)+0+sin q+cos q+(-cos q)+(-sin q)
且 m=
3 4s41 6
7AC 8ACDE 920n 0- q
5 41 1
s31
=-

al1-k2
81 k 6
w e r- t 6a3
17 4 ∴標準差為

u s73 i zx 6〔(-1)+0 +sin q+cos q+(-cos q)+(-sin q)〕- - 6


3a3 3s30 1 1
( )
2
2 2 2 2 2 2
y o1C;2 6 小時
2 5
s17
=zx -
------------------------------ 詳解 ----------------------------- 3 1
= ,故選B
一、單選題 6 36 6
303 1 A sin 27n 二、多選題
B cos 27n=sin 63n>sin 27n 305 6
sin 27n
C tan 27n= >sin 27n
cos 27n
D sin 54n>sin 27n
E tan 54n>sin 54n>sin 27n
故選A
2 如下圖,∠A=∠BCD=40n -5a2
A ×:tan q= =-a2 c x=5
x

B ○:OP=al52+(-5a2 )
∴P(5﹐-5a2 )
2
=5a3
5 a3
c cos q= =
5a3 3
∴CD=BC×cos 40n=AB×sin 40n×cos 40n -5a2 a6
C ×:sin q= =-
=c sin 40ncos 40n 5a3 3
故選C a3
D ○:cos(180n+q)=-cos q=-
3 如下圖,知 D 距離漁船最近 3
a6
E ○:sin(180n+q)=-sin q=
3
故選BDE
1
7 A ○:cos A= c∠A 唯一,條件為 SAS
3
∴△ABC 唯一
1
故選D B ×:sin B= c∠B=30n或 150n
2
304 4 1 當 x 為第一象限角時,
1 1
3 sin x 2 cos x tan x 又 sin A= <
f
(x) = + + =6 3 2
sin x cos x tan x
∴∠A<30n ∴△ABC 有兩種可能性
2 當 x 為第二象限角時,
1 1
3 sin x -2 cos x tan x C ○:cos A= ,cos B=
f
(x) = + + =0 3 2
sin x cos x -tan x
c∠A、∠B 唯一,條件為 AAS
3 當 x 為第三象限角時,
∴△ABC 唯一
3 sin x -2 cos x tan x
f
(x) = + + =-4 D ×:∵AC>BC
-sin x cos x tan x
∴∠B>∠A,且∠A 為銳角
4 當 x 為第四象限角時,
∴△ABC 有兩種可能性,如下圖
3 sin x 2 cos x tan x
f
(x) = + + =-2
-sin x cos x -tan x
∴f(x)之值不可能為 4,故選E
互動式教學講義.數學(2) 解答篇 83
頁碼 頁碼
1 2a2 q ∵∠BED=q c∠AED=180n-q
E ×:cos A= c sin A=
3 3 又△AED 中,AD=4,AE=5 c DE=s41
2 4 4a2 AD 4
s41
∴ = c sin B= >1 (180n-q)
∴sin∠AED=sin = = =sin q
2a2 sin B 3 DE
3 4 4s41
s41
故 sin q= =
∴∠B 不存在,即△ABC 不存在 41
故選AC w 令 AD=BD=x
2 2 2 2
306 8 A ○:AB +AC =2(AM +BM ) 92+92-32
2 cos B=
c AM=7
c 72+92=2(AM +42) 2×9×9
B ×:如下圖 92+x2-x2

2×9×x
153 81 81
c = c x=
9 x 17
81
故 BD=
17
令 AE=AF=x,BD=BE=y,CD=CF=z 1
308 e 大正方形的邊長為 1,小正方形的邊長為
則 x+y=7,y+z=8,z+x=9 4
解之得 y=3=BD DH=AE=sin q,DE=cos q
72+82-92 32 2 1 1 2
又 cos B=
2
2×7×8
2 2

112 7
= ∴cos q-sin q= c(cos q-sin q)
4
2

() 4
∴AD =AB +BD -2 AB×BD cos B 1 15
c 1-2 sin q cos q= c sin q cos q=
2 16 32
=72+32-2×7×3× =46
7 31
∴(sin q+cos q)2=1+2 sin q cos q=
c AD=s46 16
s31
al12(12-7)
C ○:△ABC 面積為
故 sin q+cos q=
(12-8) (12-9)=12a5 4
AB+BC+AC r sin 1014n=sin(-66n)
D ○:△ABC 的面積=rs,其中 s=
2 =-sin 66n=k
∴12a5 =r×12 c r=a5 c sin 66n=-k,如右圖
2 3a5 AC ∴tan 246n=tan(180n+66n)
E ○:cos B= c sin B= c =2R
7 7 sin B =tan 66n

al1-k2
9 21 21a5 k
c =2R c R= = =-
3a5 2a5 10
7 t 設外接圓半徑為 R
故選ACDE AB 6
c =2R=
三、填充題 sin 60n sin 30n
9 sin A=-cos B c cos B<0 ∴∠A 為銳角,∠B 為鈍角 c AB=6a3
又 sin A=cos(90n-∠A)=-cos B=cos(180n-∠B) 309 y 如右圖
∴90n-∠A=180n-∠B c∠A-∠B=-90n a2+b2-c2
cos C=
∠A+∠B=130n 2ab
∴ c∠A=20n
∠A-∠B=-90n 2 +32-(a7 )
2 2


307 0 2 sin q+cos q=-2 c cos q=-2 sin q-2 2×2×3
c cos2 q=(-2 sin q-2) 2
1

c 1-sin2 q=4+8 sin q+4 sin2 q 2
c 5 sin2 q+8 sin q+3=0 c(5 sin q+3)
(sin q+1)=0 a3
c sin C=
3 2
∴sin q=- 或-1(不合 ∵q 為第三象限角)
5 ∴△ABC 的面積為
3 1 1 a3 3a3
故 sin q=- ab sin C= ×2×3× =
5 2 2 2 2
84 互動式教學講義.數學(2) 解答篇
頁碼 頁碼
u cos∠EAF=cos(180n-∠BAC) 四、混合題
=-cos∠BAC 310 o 設颱風下午 2 時在 B 處
52+62-72 1 經過 x 公里後到達 C 處
=- =-
2×5×6 5 此時 A 地進入暴風半徑的範圍內
又 AE=5,AF=6
2
c EF =52+62-2×5×6×cos∠EAF
1
( )
=61-60× - =73
5
故 EF=s73
i 令司令臺位於 O 點
直升機一開始位於 B 點
3 分鐘後位於 D 點,
如右示意圖
由題意知∠BOA=60n,
∠DOC=30n ∵∠ABC=60n-15n=45n
且∠AOC=90n ∴x2+(200a2 )2
-2×x×200a2 ×cos 45nN2502
3 c x2+80000-400xN62500
∴BD=AC=120× =6 公里
60 c x2-400x+17500N0
令 AB=CD=h 公里 c(x-50)(x-350)N0
h c 50NxN350
a3
c OA= 公里,OC=a3 h 公里
1 50÷50=1
2 2 2
∴AC =OA +OC ∴下午 3 時 A 地會開始進入暴風半徑的範圍內
h 2 故選C
( )
c 6 2=
a3
+ (a3 h) 2

2(350-50)÷50=6
10 2 54 ∴A 地在暴風半徑範圍內的時間共有 6 小時
c h =36 c h2=
3 5
3s30
c h= (公里)
5
3s30
故飛行高度為 公里
5
Notes

- 85 -
Notes

- 86 -
Notes

- 87 -

You might also like